Phương trình hàm trên tập rời rạc

Những bài toán về chủ đề phương trình hàm hiện nay đã trở nên khá phổ biến đối với các bạn học sinh yêu thích môn Toán, vì chúng đã xuất hiện thường xuyên hơn trong các đề thi học sinh giỏi môn Toán các cấp cũng như kì thi chọn đội tuyển HSG Toán cấp quốc gia, VMO hay các kì thi khu vực và quốc tế.

CHINH PHC OLYMPIC TOÁN
NGUYN MINH TUN
DOÃN QUANG TIN
TÔN NGC MINH QUÂN
PHƯƠNG TRÌNH HÀM
TRÊN TP RI RC
TẠP CHÍ VÀ TƯ LIỆU TOÁN HC
Omaths
Littited
Edition
Chuyên đề
Bồi dưỡng
Hc sinh gii
Phương trình hàm
Trên tp ri rc
TẠP CHÍ VÀ TƯ LIU TOÁN HC
TẠP CHÍ VÀ TƯ LIỆU TOÁN HC
Copyright © 2019 by Tap chi va tu lieu toan hoc.
All rights reserved. No part of this book may be reproduced or distributed in any form
or by anymeans, or stored in data base or a retrieval system, without the prior written
the permission of the author.
TẠP CHÍ VÀ TƯ LIỆU TOÁN HC
HCHC
Nh nên rt ph bii vi các bn
ht hi thi hc sinh gii
các ci tuyn quc gia, VMO hay các thi khu vc
quc t c bic bit, trong các lp d
dp ri rc là mt mc ít các hc sinh cý
ti b c tip xúc nhing thi ngoài vic s dt
x n chúng ta còn phi s dng các tính cht s hc rt
c sc ca tp ri rt, tính cht ca s nguyên t, ca s
rong ebook này chúng tôi s mang ti cho bc tuyn tp các
p ri rc mt s 
khác hay khó vi nhng li gic sc nhm giúp bc th
nhiu cách nhìn khác v mng toán nng thn b cho các
hc sinh gii, olympic.
Mình xin gi li ci
1. Thy Hunh Kim Linh  Khánh Hòa 
giúp bn mình v phn ni dung.
2. Bn La Th  i hc Hoa Sen ã giúp bn mình chnh sa bn
th hoàn thi
Mt ln na gi li cn, các thng h và theo dõi fanpage sut
thi gian qua. Hy vng ebook này s c cho mi. Thank you!
Nhóm tác gi
Nguyn Minh Tun
Doãn Quang Tiến
Tôn Ngc Minh Quân
LI GII THIU
PHƯƠNG TRÌNH HÀM TRÊN TẬP RI RC
TẠP CHÍ VÀ TƯ LIỆU TOÁN HC
CHINH PHC OLYMPIC TOÁN
Chinh phc olympic toán| 1
Phương trình hàm trên tp ri rc
CHINH PHC OLYMPIC TOÁN
Chuyên đề
PHƯƠNG TRÌNH HÀM TRÊN TẬP RI RC
Tạp chí và tư liệu toán hc
Để gii quyết các bài toán phương trình hàm trên tp ri rc th gii bng các tính
cht s hc thì nên lưu ý đến mt s du hiu sau:
Nếu xut hin các biu thc tuyến tính chứa lũy thừa, th nghĩ đến các i toán
liên quan đến cp ca phn tử, các phương trình đc biệt như phương trình Pell
hay phương trình Pythagore,<hay đưa v vic x các phương trình đnh
nghim nguyên.
Nếu hàm s đã cho hàm nhân tính, ta thường hay xét đến giá tr hàm s ti các
đim là s nguyên t hoc dãy vô hn các s nguyên t.
S dụng các đẳng thc và bất đẳng thc s hc.
đặc bit nht, trong mt s bài toán, h số đếm th dùng để xây dng
nhiu dãy s tính cht s hc thú v. Trong h số
10
chúng ta th rt khó
nhn ra quy lut của dãy, nhưng nếu chọn được h số phù hp thì i toán
th gii quyết đơn giản hơn rất nhiu.
Nếu
2, ,gg
vi
g
số đếm, thì mi s nguyên dương
M
đều biu din
duy nhất dưới dng:

12
1 2 1 2 1
... ...
nn
n n n
g
M a a a a g a g a g a
vi
1
1 1;0 1, 2, .
i
a g a g i n
Cơ số đếm mà hay được s dụng trong các bài toán phương trình hàm trên tập ri rc là
2
3.
Sau đây, chúng tôi sẽ đề cập đến các bài toán phương trình hàm sử dng các tính cht
cũng như các phương pháp trong s hc để gii, nhm giúp bạn đọc hiểu rõ hơn và có một
cái nhìn mi m hơn về các phương pháp khác để giải phương trình hàm, bên cạnh đó
chúng tôi cũng s gii thiu cho bạn đọc các bài toán phương trình hàm khó trong tài
liu này. Nào cùng bắt đầu nhé!
2 | Tạp chí và tư liệu toán hc
Bồi dưỡng hc sinh gii
TẠP CHÍ VÀ TƯ LIỆU TOÁN HC
I. ĐỀ BÀI
Câu 1. Tìm tt c các hàm s
:f
thỏa mãn điều kin sau:
3 2 ,f n f f n n n
Câu 2. Tìm tt c các hàm s
:f
thỏa mãn điều kin sau
22
, , 1m n f m n mf n nf m m n
Câu 3. Cho hàm s
**
:f
thỏa mãn điu kin sau:
*
1,f n f f n n
Chng minh rng
*
,.f n n n
Câu 4. Tìm tt c các hàm s
**
:f
thỏa mãn điều kin sau:
2 2 *
, , x f y f x y x y
Câu 5. Tìm tt c các hàm s
**
:f
thỏa mãn điều kin sau:
2
2 2 *
, , *f m f n m n m n
Câu 6. Tìm tt c các hàm
:f
tha mãn tn ti s
k
và s nguyên t
p
sao cho
vi mi
,n k f n p f n
và nếu
mn
thì
1 1.f m f n
Câu 7. Cho
p
là s nguyên t l. Tìm tt c các hàm
:f
thỏa mãn đồng thi các
điu kin:
i)
f m f n
vi
modm n p
ii)
,,f mn f m f n m n
Câu 8. Tìm s nguyên không âm
n
nh nht sao cho tn ti hàm s
: 0,f
khác
hng s thỏa mãn đồng thời các điều kin:
i)
,,f xy f x f y x y
ii)
22
2 0,1,..., , ,f x y f x f y n x y
Vi s
n
tìm được, hãy tìm tt c các hàm s tha mãn.
Câu 9. Gi s hàm s
*
:f
thỏa mãn các điều kin sau:
11f








1
1 2 1
2
1 2
2
n
f if n m
fn
n
f if n m
Tìm các giá tr ca
n
sao cho
2019.fn
Câu 10. Tìm tt c các hàm s
**
:f
thỏa mãn các điều kin sau:
Chinh phc olympic toán| 3
Phương trình hàm trên tp ri rc
CHINH PHC OLYMPIC TOÁN

1 1, 3 3
2
4 1 2 2 1
4 3 3 2 1 2
ff
f n f n
f n f n f n
f n f n f n
vi mi s nguyên dương
n
.
Câu 11. Cho hàm s

:f
thỏa mãn đồng thời các điều kin:
fn
là ước ca
2018
n
vi mi
n
.f a f b f c
vi mi
,,a b c

2 2 2
a b c
a) Chng minh rng nếu
n
l hoc
4n
thì
1fn
b) Gi
A
là tp hp giá tr có th có ca
2 2018ff
. Tính
A
Câu 12. Có tn ti hàm s
:f S S
thỏa mãn điều kin
22
, , ,f a f b f a b a b S a b
không, trong đó
*
\1S
?
Câu 13. Tìm tt c các hàm s
**
:f
thỏa mãn điều kin
2
2*
1,n f n f f n n n n
.
Câu 14. Tìm tt c hàm s
:f
thỏa mãn đồng thời hai điều kin sau:
i)
x f y f x y f x f y
vi mi
,xy
;
ii) Tp hp




, , ,
f x f y
I x y x y
xy
là mt khong
Câu 15. Tìm các hàm s
**
:f
tha mãn
2
2 2 *
,,f m f n m n m n
Câu 16. Cho hàm
,f x y
tha mãn các điu kin:
0, 1; 1,0 ,1f y y f x f x
1, 1 , 1,f x y f x f x y
Vi mi s nguyên không âm
,xy
. Tìm
4,1981f
?
Câu 17. Cho hàm

:f
thỏa mãn các điều kin sau:
i)
1;f n f n n
ii)
3 , .f f n n n Z
Hãy tính
2003 .f
Câu 18. Cho
fn
là hàm s xác định vi mi
*
n
và ly giá t không âm tha mãn tính
cht:
*
,:n m f m n f m f n
ly giá tr 0 hoc 1
20f
30f
.
9999 3333f
.
4 | Tạp chí và tư liệu toán hc
Bồi dưỡng hc sinh gii
TẠP CHÍ VÀ TƯ LIỆU TOÁN HC
Tính
2000f
.
Câu 19. Cho
,fg
là các hàm xác định trên thỏa mãn điều kin
2 . , ,f x y f x y f x g y x y
Chng minh rng nếu
0fx
1,f x x
thì

0
1g y a
Câu 20. Cho hàm s
:f
thỏa 2 điều kin
i)
1;f x x x
ii)
. ; ,f x y f x f y x y
Chng minh rng không th tn ti hai s
;ab
.0f a f b
Câu 21. Cho
2003
, y cos 2 cos
2
f x x y a x y
vi
,a
.
Chng minh rng

22
min , max , 2003.f x y f x y
Câu 22. Cho hàm s
2
1
, 0.
2
x
f x x
x
Gi s
0
f x x
*
1
,0
nn
f x f f x n x
.
Chng minh
2
1
1
, 1,0,1 1
1
1
n
n
n
fx
nx
fx
x
f
x



Câu 23. Cho hàm s

* * *
:f
là hàm s thỏa mãn đồng thời các điều kin sau:
i)
1,1 2f
ii)
*
1, , , ,f m n f m n m m n
iii)
*
, 1 , , ,f m n f m n n m n
Tìm tt c các cp s
,pq
sao cho
, 2019.f p q
Câu 24. Tìm tt c các hàm s
:f
thỏa mãn các điều kin sau:
i)
2
0,f x x n
ii)
f x f y
chia hết cho
xy
vi mi
,,x y x y
Câu 25. Tìm tt c các hàm s

**
:f
mà tp
*
0xx
tha mãn:
*
2 , , 1
f xy
f x f y xyf xy x y
f x y
Câu 26. Cho hàm
:f
là mt hàm s tha mãn vi mi
1n
thì có mt s nguyên
t
p
là ước ca
n
sao cho:




1
n
f n f p f
p
2018 2019 2020
3 5 7 2017.f f f
Chinh phc olympic toán| 5
Phương trình hàm trên tp ri rc
CHINH PHC OLYMPIC TOÁN
Hãy tính giá tr ca biu thc
2018 2019 2020
2018 2019 2020G f f f
Câu 27. Tìm tt c các hàm s
**
:f
tha mãn:
3 2 2 2 2 *
2 , ,f m n f m f n f m f n m n
Câu 28. Gi s
:f
là hàm liên tc và gim sao cho vi mi
,xy
ta có
f x y f f x f y f y f x
Chng minh rng
f f x x
.
Câu 29. Cho song ánh
:f
. Chng minh rng tn ti vô s b
,,a b c
vi
,,a b c
tha mãn
a b c
2 f b f a f c
.
Câu 30. Có bao nhiêu hàm
:f
**
tho mãn đồng thời các điều kin sau
a)
11f
b)
2
2 9 1 1997,f n f n f n
 *.n
Câu 31. Tìm tt c các hàm s
**
:f
sao cho.
a)
22f
b)
..f m n f m f n
vi mi
*
,mn
,
,1UCLN m n
c)
f m f n
*
,,m n m n
.
Câu 32. Tìm tt c các hàm s
:f
tha mãn
1, ,f m n f mn f m f n m n
Câu 33. Tìm tt c các hàm s
:f
tha mãn
02f
2 2 2 , ,f x f x y f x f y x y
1
Câu 34. Tìm tt c hàm s
:f
sao cho
2 3, 1f f n f n n n
Câu 35. Chng minh rng tn ti duy nht hàm s
: * *f
tha mãn
, , *f m f n n f m b m n b
i
Câu 36. Hãy xác định tt c hàm s
**
:f
thỏa mãn đẳng thc:
1 2 . 3f n f n f n f n a
1
Vi a là s t nhiên tha mãn
1a
là s nguyên t.
Câu 37. Tìm tt c các hàm s
**
:f
tha mãn
1.
t
f n a f n an t a k
vi
...
t
t
f n f f f n
vi
, at
là s t nhiên tùy ý tha mãn
2 1 1k t a
.
Câu 38. Cho hàm s
:f
tha mãn:
2 1 2 1 2 1 2 1 3 1 2
,
2
f n f n f n f n f n
n
f n f n
6 | Tạp chí và tư liệu toán hc
Bồi dưỡng hc sinh gii
TẠP CHÍ VÀ TƯ LIỆU TOÁN HC
Tìm n sao cho
2009fn
.
Câu 39. Tìm tt c các hàm s
:f
tho mãn:
1 1 1
, , ,
3 3 9
f xy f xz f x f yz x y z
.
Câu 40. Cho
2nn
và hàm s
:f
sao cho:
1
;,
nn
f x y x f x f f y x y
*
a) Gi s rng
2002 0.f
Tính
2002 .f
b) Tìm hàm s
f
.
Câu 41. Tìm tt c các hàm s
:f
tha mãn
2 3 2 3
,,f x y z f x f y f z x y z
Câu 42. Cho hàm s
**
:f
thỏa mãn đồng thời hai điều kin:
a)
,,f ab f a b f a b
vi mi
*
,,a b a b
; trong đó
, , ,a b a b
lần lượt là bi
chung nh nhất, ước chung ln nht ca hai s nguyên dương
,ab
;
b)
f p q r f p f q f r
vi mi s nguyên t
,,p q r
.
Tính giá tr ca
2013f
? Kí hiu
*
là tp hp tt c các s nguyên dương.
Câu 43. Đặt
: 0,1 0,1Ff
2.n
Tìm giá tr nh nht ca c thỏa mãn điều kin
11
00
n
f x dx c f x dx

Vi
fF
f
là hàm liên tc.
Câu 44. Tìm tt c các hàm
: 1,1f 
liên tc, tha mãn:
2
2
1
x
f x f
x



, 1,1x
Câu 45. Có th tn ti hay không mt hàm s
:f
, liên tc trên và thỏa mãn điều
kin: Vi mi s thc
x
, ta có
fx
là s hu t khi và ch khi
1fx
là s vô t.
Câu 46. Tìm tt c các hàm s
:f
thỏa mãn điều kin
f x f t f y f z
vi
mi s hu t
x y z t
, , ,x y z t
theo th t lp thành cp s cng.
Câu 47. Gi s
,rs
là hai s cho trước. Tìm tt c các hàm s
:f
thỏa mãn điều
kin
,,f x f y f x r y s x y
?
Câu 48. Tìm tt c các hàm s
:f
sao cho vi tt c các s nguyên
,,a b c
tha mãn
0a b c
, đẳng thức sau là đúng:
2 2 2
2 2 2f a f b f c f a f b f b f c f c f a
Câu 49. Tìm tất cả các hàm
,: fg
có đạo hàm trên
thỏa mãn
'
gx
fx
x

;
'
fx
gx
x

x

Chinh phc olympic toán| 7
Phương trình hàm trên tp ri rc
CHINH PHC OLYMPIC TOÁN
Câu 50. Tìm tt c các hàm
*
:f
có đạo hàm trên
*
tha mãn
*
,f xy f x f y x y
1
Câu 51. Tìm tt c các hàm
:f
tha mãn
1f f n n b n
trong đó
b
là s nguyên dương chẵn.
Câu 52. Tìm tt c các hàm
:f

tha mãn:
i)
,f x f y yf x x y
1
ii)
lim 0
x
fx

Câu 53. Chng minh rng tn ti song ánh
:f

sao cho
3 4 ,f mn m n f m f n f m f n m n
Câu 54. Tìm tt c các hàm
:f
tha:
3 2 6 ,f f f n f f n f n n n
Câu 55. Tìm tt c các hàm s
: 0; 0;f  
thỏa mãn điều kin:
, 0; 1f f x yf yf x x y 
Câu 56. Chng minh rng tn ti duy nht mt hàm s
f
xác định trên tp các s thc
dương, nhận giá tr thực dương và thỏa mãn
6.f f x x f x
Câu 57. Hàm s
:f
thỏa mãn đồng thời các điều kin sau:
: , 1
: 2 2 , 2
: 0 1 3
i f f n n n
ii f f n n n
iii f
Tìm giá tr
1995 , 2007ff
Câu 58. Tìm
: 0,1f
tha mãn
, , 0,1f xyz xf x yf y zf z x y z
Câu 59. Tìm tt c các hàm
f
xác định trên và tha mãn đồng thời các điều kin sau:
2 2 3 ,
11
f n f k n f k n f n f k k n
f
Câu 60. Tìm tt c các hàm s
**
:f
thỏa mãn đồng thời hai điều kin sau:
**
*
2 , ,
1 ,
f f n n k n k
f n f n n
Câu 61. Tìm tt c các hàm s
:f
thỏa mãn đồng thời hai điều kin sau:
2013 2016
4,
f
f f n n n
Câu 62. Tìm tt c các hàm s
*
:f
thỏa mãn điều kin sau:
8 | Tạp chí và tư liệu toán hc
Bồi dưỡng hc sinh gii
TẠP CHÍ VÀ TƯ LIỆU TOÁN HC
1 1 . 3 , 1f n f n f n f n n
Câu 63. Tìm tt c các hàm
:f

tha mãn:
f x f y f x y f y
Câu 64. Tìm s nguyên dương
m
nh nht sao cho tn ti hàm s
*
: \ 1;0;1f 
thỏa mãn đồng thời các điều kin sau
i)
2015 , 1 2016 ;f m f f m f
ii)
1
, 1, 2,....
1
fn
f n m n
fn
Câu 65. Xác định hàm s
fx
liên tc

thỏa mãn đồng thời các điều kin:
22f x f x
vi mi
x
,
1
32
11
fx
x
f f x e x e f x
vi mi
x
,
2
1 1 1f e e f
,
3
fk
là s nguyên dương với mi s nguyên dương
k
,
4
Câu 66. Tìm tt c các hàm
:f
thỏa mãn đồng thời hai điều kin sau:
Vi mi cp a, b nguyên dương không nguyên tố cùng nhau, có
.f a f b f ab
Vi mi b a, b nguyên dương tn ti mt tam giác không suy biến độ dài ba
cnh là
,f a f b
1f a b
.
Câu 67. Tìm các hàm s
: 1;f 
tho mãn điều kin:
f x f y y x f xy
vi mi
,1xy
1
Câu 68. Tìm tt c các hàm
**
:f
thỏa mãn đẳng thc:
2 2 2 2
22f f m f n m n
, vi mi
*
,.mn
Câu 69. Tìm tt c các s nguyên không âm
n
sao cho tn ti mt hàm
: 0;f
khác
hng thỏa mãn đồng thời 2 điều kin sau
i)
,,f xy f x f y x y
ii)
22
2 0;1; 2;...; .,y f x f y x yf x n 
Câu 70. Tìm tt c các hàm s
: * *f
tho mãn điều kin:
3
2 2 2 2 *
2 . . , ,f m n f m f n f n f m m n
Câu 71. Tìm tt c các hàm s
:f
tho mãn điều kin:
0fc
31
1 , *
3
fn
f n n
fn
1
Chinh phc olympic toán| 9
Phương trình hàm trên tp ri rc
CHINH PHC OLYMPIC TOÁN
Câu 72. Tìm tt c các hàm s
:f
tha:
2 2 ,f a f b f f a b a b
Câu 73. Có tn ti hay không hàm s
:f
sao cho
, , 1f m f n f m n m n
Câu 74. Cho hàm s
:f
là hàm s thỏa mãn các điu kin sau:
i)
,,f mn f m f n m n
ii)
mn
là ước ca
f m f n
vi mi
,mn
Chng minh rng tn ti mt s t nhiên l
k
sao cho
,.
k
f n n n
Câu 75. Tìm tt c các hàm s
**
:f
thỏa mãn đồng thời các điều kin sau:
i)
0 0, 1 1ff
ii)
0 1 2 ...f f f
iii)
2 2 2 2 *
,,f x y f x f y x y
Câu 76. Tìm tt c các hàm s
:f
thỏa mãm các điều kin sau:
i) Nếu
ab
thì
f a f b
ii)
22
,,f ab f a b f a f b a b
Câu 77. Tn ti hay không hàm s
: 1,2,...,fn
thỏa mãn điều kin:
i)
f
là hàm đơn ánh
ii)
f ab f a f b
vi mi
, 1, 2,...,a b n
ab n
Câu 78. Gi s Josephus
1n
người bn,
n
người này đúng thành một vòng tròn
đánh số t
1
đến
n
theo chiều kim đồng h, t sát theo nguyên tắc, người th nht cm
dao đếm
1
ri t sát, ngưi th hai đếm
2
ri t sát,<Quá trình dng li khi còn mt
người. Gi
fn
là hàm s biu th v trí cùa người sống sót đó. Câu hỏi đặt ra là, hãy tính
fn
?
Câu 79. Cho hai hàm s
**
,:fg
là hai hàm s thỏa mãn đồng thời các điu kin:
i)
g
là hàm s toàn ánh
ii)
2 2 2
2,f n n g n n
Nếu
2019 ,f n n n n
thì
f
có vô s đim bất động.
Câu 80. Tìm tt c các hàm s
**
:g
thỏa mãn điều kin sau:
g g 1 3 ,g n n n n g n n
Câu 81. Cho ba s thc
,,a b c
không âm, phân bit sao cho tn ti hàm
,:fg
tha
mãn
x
af xy bf cf x g y
y



vi mi s thực dương
xy
.
10 | Tạp chí và tư liệu toán hc
Bồi dưỡng hc sinh gii
TẠP CHÍ VÀ TƯ LIỆU TOÁN HC
Chng minh rng tn ti hàm
:h
sao cho:
2 , 0
x
f xy f f x h y x y
y



Câu 82. Tìm tt c hàm s
:f
tha mãn:
! ! ! ! , ,n f m f n f m m n
Câu 83. Tn ti hay không hàm s
**
:f
thỏa mãn điều kin sau:
*
3 2 ,f f n n f n n
Câu 84. Tìm tt c các hàm s tăng thực s
**
:f
thỏa mãn điều kin sau:
*
2,f n f n f n n
Câu 85. Tìm tt c các toàn ánh
:f
sao cho vi mi
,mn
tha mãn:
f m f n m n
Chinh phc olympic toán| 11
Phương trình hàm trên tp ri rc
CHINH PHC OLYMPIC TOÁN
II. LI GII.
Câu 1. Tìm tt c các hàm s
:f
thỏa mãn điều kin sau:
3 2 ,f n f f n n n
Li gii
Gi s
f
là hàm s thỏa mãn điều kin bài toán.
Đặt
,.g n f n n n
Khi đó, thì ta được


2 2 , 1g f n f f n f n f n n g n n
Áp dng liên tiếp
1
ta được
2
2 2 ... 2 ... ... ,
m
g n g f n g f f n g f f f n
trong đó có
m
du
.f
Như vậy thì
gn
chia hết cho
2 , 0,
m
m g n n
hay
,f n n n
Th li thì thy hàm s
,f n n n
tha mãn yêu cầu đề bài.
Vy tt c các hàm s thỏa mãn đề bài là:
,.f n n n
Nhn xét. Việc đặt hàm ph
,g n f n n n
giúp ta đưa phương trình hàm ban
đầu v dng mới đẹp hơn. Và khi đó ta phát hiện ra thêm được các tính cht ca hàm mi
gn
để t đó ta áp dụng liên tiếp các tính cht ykết hp vi các tính cht s hc chia
hết để suy ra được hàm tha mãn yêu cầu đề bài.
Câu 2. Tìm tt c các hàm s
:f
thỏa mãn điều kin sau
22
, , 1m n f m n mf n nf m m n
Li gii
Gi s
f
là hàm s thỏa mãn điều kin bài toán.
Kí hiu
,P u v
là phép thế
,uv
vào
1
thì ta được:
2
0, 0 ,P n nf n nf n
Do đó
2
0 , .f n f n
Đặt
0 , .g n f n f n
Khi đó, ta thay vô
1
ta được
22
, , 2m n g m n mg n ng m m n
Hơn nữa, ta còn có
00g
2
0,g n n
Kí hiu
,Q u v
là phép thế
,nm u v
vào
2
thì
2
, 2 2 2 ,Q n n ng n ng n n
Do đó ta được
2
2,g n g n n
12 | Tạp chí và tư liệu toán hc
Bồi dưỡng hc sinh gii
TẠP CHÍ VÀ TƯ LIỆU TOÁN HC
2 2 2 4 2
2 , 3 5 ,Q n n n g n n g n n
T đó suy ra
4
3 5 ,g n g n n
T đây ta áp dụng liên tc các tính chất trên, thì ta đó ta suy ra
*
3 , ,
k
g n n k
Suy ra:
0,g n n
hay
0 , .f n f const n
Th li thì ta thy hàm này tha mãn yêu cu bài toán.
Vy tt c các hàm s tha mãn yêu cu bài toán là
0 const, .f n f n
Nhn xét. Cũng tương tự như i toán 1 ta nhìn phương trình hàm ban đầu dưới mt hàm
ph khác, bng các phép thế bản ta phát hiện ra được mt s tính chất sơ khai ban đầu.
bằng phép đặt
,g n f n n n
ta được một phương trình hàm dạng y chang
phương trình hàm ban đầu, nhưng ta lại được thêm các điều kin ràng buc là
00g
2
0,g n n
nên t đó ta đã được thêm các ràng buc, thun li cho vic giải phương
trình. Phép đặt này rt hay, va bảo toàn phương trình hàm dạng y chang ban đầu
kèm theo các điều kin rang buộc phương trình hàm ban đu không có. T đấy,
tương t bài toán 1, ta phát hin các tính cht ca hàm
gn
và s dng liên tc chúng
kết hp cùng vi các tính cht chia hết để suy ra hàm s cn tìm.
Câu 3. Cho hàm s
**
:f
thỏa mãn điu kin sau:
*
1,f n f f n n
Chng minh rng
*
,.f n n n
IMO 1977
Li gii
Gi s
f
là hàm s thỏa mãn điều kin bài toán.
Đặt

*
min , ,d f n n
theo nguyên lý cc hn thì
d
tn ti và duy nht.
Gi
*
m
sao cho:
.f m d
Nếu
1m
thì
1,d f m f f m
mâu thun.
Do đó
fn
đạt giá tr nh nht duy nht ti
1n
Lp luận tương tự thì ta có
*
2 min , , 2f f n n n
Và lp lun lại quá trình tương tự như trên ta được:
1 2 3 ... ...f f f f n
Ta có
11f
nên
*
,f n n n
Nếu tn ti
*
0
n
00
f n n
thì

00
1.f n n
Chinh phc olympic toán| 13
Phương trình hàm trên tp ri rc
CHINH PHC OLYMPIC TOÁN
Suy ra

00
1,f f n f n
mâu thun
Do đó,
*
,,f n n n
th li thy tha mãn yêu cu bài toán.
Vy tt c các hàm s tha mãn yêu cu bài toán là:
*
,.f n n n
Nhn xét. Đây một bài toán phương trình hàm trong thi Toán Quc Tế - IMO năm
1977, mt bài toán phương trình hàm với điều kin rang buc dng bất đẳng thc, rt
l mới. Làm ta nãy ra ý tưởng s dng nguyên cc hạn để đánh giá để có điều
và suy ra được hàm s tha mãn yêu cầu đề bài.
Câu 4. Tìm tt c các hàm s
**
:f
thỏa mãn điều kin sau:
2 2 *
, , x f y f x y x y
Li gii
Gi s
f
là hàm s thỏa mãn điều kin bài toán.
Trong
ta thế
1xy
ta được
2
1 1 1 1 1 1f f f
Trong
ta thế
1x
ta được
2 * *
1 1 1 , , 1f y f y y y y f y y
Trong
ta thế
1y
ta được
2 2 * 2 2 * *
1 1, , 1 1, , , 2x f f x x y x f x x y f x x x
T
1
2
ta suy ra
*
,,f x x x
th li ta thy tha mãn yêu cu bài toán.
Vy tt c các hàm s tha mãn yêu cu bài toán là
*
,.f x x x
Nhn xét. Đây một bài toán phương trình hàm trên tp ri rạc, cho i dng chia
hết. Bng các phép thế đơn giản cùng với các đánh gsố hc không quá khó khan, ta
th nhanh chóng đánh giá được biên ca hàm
f
để t đó ta suy ra được hàm s tha
mãn đề bài.
Câu 5. Tìm tt c các hàm s
**
:f
thỏa mãn điều kin sau:
2
2 2 *
, , *f m f n m n m n
IMO Shortlist 2004
Li gii
Gi s
f
là hàm s thỏa mãn điều kin bài toán.
Trong
*
ta thế
1mn
ta được:
2
22
1 1 1 1 4 1 1,f f f
do
1f
11f
Trong
*
ta thế
1m
ta được:
2
2
2 2 * *
1 1 , , 1 1 ,f f n n m n f n n n
14 | Tạp chí và tư liệu toán hc
Bồi dưỡng hc sinh gii
TẠP CHÍ VÀ TƯ LIỆU TOÁN HC
Trong
*
ta thế
1n
ta được:
22
2 2 * 2 2 *
1 1 , 1 1 ,f m f m m f m m m
Vi
p
là mt s nguyên t bt kì thì:
Trong
*
ta thế
1, 1m n p
ta được:

2
11f p p
2
11
11
f p p
f p p
Trường hp 1.
22
1 1 1 1.f p p f p p
Ta thế
1, 1m p n
vào
*
ta được:
22
2
22
22
1 1 1 1 1 1 1 1f p p p p
Mà ta lại có đánh giá sau đây:
2
22
2 2 2 2
2 2 2
1 1 1 1 1 1 1 ,p p p p p p p p
mâu thun
Do đó, ta phải xảy ra trường hp còn li.
Trường hp 2.
1 1 1 1,f p p f p p
vi mi
p
là s nguyên t
Hay tn ti
k
sao cho
.f k k
Vi mi
k
như thế và s t nhiên
0n
bt kì thì ta có:
2
2
22
2 2 2
1 1 2k f n k n k f n p f n p n f n f n n
Khi ta chn
k
là mt s đủ ln thì ta bt buc phi có:
*
,,f n n n
th li tha.
Vy tt c các hàm s tha mãn yêu cu bài toán là:
*
,.f n n n
Nhn xét. Cũng tương tự như bài toán 4, đây một bài phương trình hàm trên tập ri
rc dng chia hết. Cũng tương t bài trên, ta cũng thế bng các phép thế đơn giản để
phát hin mt s tính cht của đề bài. Nhưng bài toán 5 này khó hơn bài toán 4 rt
nhiu, t các tính cht ta m được, ta không th chặn được khong ca hàm
f
để ri
suy ra
*
,f n n n
như bài toán trên được. thế ta phi xét giá tr ca hàm s
f
ti các gtr s nguyên t để x bài toán bng mt s kiến thức đơn giản v
gii hn ta có th suy ra được
*
,f n n n
mt cách d dàng, t đó kết thúc bài toán.
Chinh phc olympic toán| 15
Phương trình hàm trên tp ri rc
CHINH PHC OLYMPIC TOÁN
Câu 6. Tìm tt c các hàm
:f
tha mãn tn ti s
k
và s nguyên t
p
sao
cho vi mi
,n k f n p f n
và nếu
mn
thì
1 1.f m f n
Iran TST 2005
Li gii
Gi s
f
là hàm s thỏa mãn điều kin bài toán.
Gi s
nk
p
không chia hết cho
1n
thì khi đó tồn ti
k
sao cho
1.n n kp
Suy ra ta được
1f n f n kp
Mt khác ta li có
f n f n kp
nên
1 1 1f n f n f n f n
Vi
1n
bt kì thì
1 1 1 1 2n n kp f n f n kp
Do đó với
1n
thì ta có:
1,2 .fn
Bây gi ta s xét hai trường hp sau:
Trường hp 1.
2,f n n k
1.pn
Xác định
nk
p
không chia hết cho
1n
khi đó tồn ti
m
sao cho:
1nm
1.pm
Suy ra
13f n f m
hay
1fn
Ta xác định hàm
f
như sau:
2,f n n k
1.pn
1,f n n k
p
không là ước ca
1.n
,.f i f i p i k
Trường hp 2.
1,f n n k
1.pn
Trong trường hp này
1,f n n k
nếu gi s
2S a f a
thì s không tn ti
,m n S
tha mãn
1.mn
Ta xác định hàm
f
như sau
1, 2 , .f n n
Vi
S
là mt tp con vô hn ca sao cho không tn ti
,m n S
tha mãn
1mn
và vi
1n
thì
2 ; 1,f n n S f x
vi các giá tr
1x
còn li và
1f
mt s bt
xác định bi
2 1 1.ff
T đây ta thử lại đề bài và thy tha mãn nên ta hoàn thành bài toán.
Nhn xét. Đây một bài toán phương trình hàm trên tp ri rạc khó điều kin ràng
buc khá khó chu. bng các phép thế để tìm ra các tính cht ca hàm, cùng vi c
thuật x rất khó khan, chúng ta đã x được bài toán. Đây một bài toán khó, các
bạn đọc cn nghiên cứu và đọc thật kĩ.
16 | Tạp chí và tư liệu toán hc
Bồi dưỡng hc sinh gii
TẠP CHÍ VÀ TƯ LIỆU TOÁN HC
Câu 7. Cho
p
s nguyên t l. m tt c các hàm
:f
thỏa mãn đồng thi c
điu kin:
i)
f m f n
vi
modm n p
ii)
,,f mn f m f n m n
USA TST
Li gii
Gi s
f
là hàm s thỏa mãn điều kin bài toán.
Vi
,k
thì ta có
1 1 0f p k f pk f p f k f k
Bây gi ta s xét hai trường hp sau
Trường hp 1.
0fp
D thy nếu
10f
thì
0, ,f n n
mâu thun vi
0.fp
Xét riêng khi
1 1.f
Vi mi
x
p
khong chia hết cho
x
ta có
y
sao cho
1 mod .xy p
Do đó ta có
1 1, ,f x f y f xy f x y
Suy ra:
1fn
p
không chia hết cho
.n
Mt khác ta li

22
1f n f n
vi
p
không chia hết cho
n
nên
1,fm
nếu
m
mt s chính phương mod
p
p
không chia hết cho
.m
Nếu không tn ti
,i
vi
p
không chia hết cho
i
sao cho
1fi
thì ta ngay
1,f n n
p
không chia hết cho
.n
Xét
i
mt s không chính phương mod
p
k
mt s không chính phương mod
p
p
không chia hết cho
k
bt kì thì ta suy ra
ik
là s chính phương mod
.p
Mt khác ta li có
1f k f i f k f ik
Hay
1,fx
nếu
x
là mt s chính phương mod
p
p
không chia hết cho
x
1,fx
nếu
x
là mt s không chính phương mod
p
Xét s
0
x
sao cho

0
1.fx
Bây gi t điu kin ii) ta thay

0
,m x n p
ta được:

00
f p f px f p f x
hay
1fp
Suy ra:
1,fx
nếu
x
là s chính phương mod
p
1,fx
nếu
x
là mt s không chính phương mod
p
Trường hp 2.
0fp
suy ra
0, .f n p n
Kh năng 1. Nếu
10f
thì
0, .f n n
Chinh phc olympic toán| 17
Phương trình hàm trên tp ri rc
CHINH PHC OLYMPIC TOÁN
Kh năng 2. Nếu
10f
Gi s tn ti
0
x
sao cho
0
0fx
p
không chia hết cho
0
x
Suy ra
0
0,f nx n
Ta có dãy
0 0 0
,2 ,..., 1x x p x
là mt h thặng dư đầy đủ mod
p
Suy ra
1 0,f
điu này mâu thun.
Vy ta có
0f x p x
1,fx
vi các giá tr
x
còn li.
T các kết qu trên đây, ta thấy có
4
hàm s tha mãn yêu cu bài toán:
0,f n n
0
1
if p n
fn
if n
p
1,f n n
1 mod
1 mod
if n la mot so chinh phuong p
fn
if n khong la mot so chinh phuong p
Vậy đây là tất c các hàm tha mãn yêu cu bài toán.
Nhn xét. Đây một bài toán khó, với điều kin hàm rt kx lý, mt bài toán khó
trong thi chọn đội tuyn IMO ca M, vic ng dng sâu sc các kiến thc S Hc
tng hp trong li gii, v khá phc tp. Mong bạn đọc suy nghĩ đọc thật kĩ,
mong bạn đọc có mt li gii khác ngn gọn và hay hơn cho bài toán.
Câu 8. Tìm s nguyên không âm
n
nh nht sao cho tn ti hàm s
: 0,f
khác
hng s thỏa mãn đồng thời các điều kin:
i)
,,f xy f x f y x y
ii)
22
2 0,1,..., , ,f x y f x f y n x y
Vi s
n
tìm được, hãy tìm tt c các hàm s tha mãn.
Li gii
Vi
1n
xét hàm
f
được xác định như sau:
0
1
if p x
fx
if x
,
p
vi
p
là s nguyên t có dng
43k
Hin nhiên hàm s trên tha mãn yêu cu bài toán.
Gi s vi
0n
thì cũng tồn ti hàm s
f
tha mãn yêu cu bài toán.
Khi đó thì ta có:
2 2 2 2
2 0, , 2 , , f x y f x f y x y f x y f x f y x y
T điu kin i) ta thế
0xy
ta được:
2
0 0 0 0f f f
hoc
01f
Trường hp 1.
01f
18 | Tạp chí và tư liệu toán hc
Bồi dưỡng hc sinh gii
TẠP CHÍ VÀ TƯ LIỆU TOÁN HC
Ta thế
0y
vào
thì ta được
2
2 0 1,f x f x f f x x
22
,f x f x x
nên ta suy ra:
22
2 2 1, 1, ,f x f x f x x f x x
do
0,f x x
Điu này li trái vi gi thiết
f
khác hng s.
Trường hp 2.
00f
Ta thế
0y
vào
thì ta được
2
2 0 ,f x f x f f x x
22
,f x f x x
nên ta suy ra
22
2 2 ,f x f x f x x
Suy ra vi mi
x
thì ta phi có
0fx
hoc
1
.
2
fx
Nếu tn ti
0
x
sao cho
0
1
.
2
fx
Ta thế

0
xyx
vào
thì ta được:
22
0 0 0 0 0 0 0
2 2 2 2 2 2 f x f f x f x x f x f x f x
T
ta thay
1, 0xy
thì ta được:
1 0.f
T
ta thay
1, 1xy
thì ta được:
2 0.f
T đây ta thay
20f
vào

thì ta được:
0, ,f x x
điu này li mâu thun vi
f
khác hng s.
Vy t đây ta khẳng định được rng
1n
là giá tr nh nht tha mãn yêu cu bài toán.
Khi ta tìm được
1n
ta s quay li vic gii quyết bài toán đ bài.
Tìm tt c các hàm s
: 0,f
khác hng s thỏa mãn đồng thời các điều kin:
i)
,,f xy f x f y x y
ii)
22
2 0,1 , ,f x y f x f y x y
Gi s
f
là hàm s thỏa mãn điều kin bài toán.
Ta d dàng chứng minh được rng:
0 0, 1 1.ff
Trong i) ta thế
yx
thì ta được
22
,f x f x x
Trong ii) ta thế
0y
thì ta được
22
2 2 0,1 0,1f x f x f x f x f x
Trong i) ta thế
1xy
thì ta được
2
1 1 1 1 1f f f
Trong i) ta thế
1,x y x
thì ta được:
Chinh phc olympic toán| 19
Phương trình hàm trên tp ri rc
CHINH PHC OLYMPIC TOÁN
1 , ,f x f f x x f x f x x
Trường hp 1. Tn ti s nguyên t
p
sao cho
0.fp
Gi s cũng tồn ti s nguyên t
qp
sao cho
0.fq
Trong ii) ta thế
,x p y q
thì ta được
2 2 2 2
2 0 0f p q f p f q f p q
Do đó với mi
,ab
thì ta luôn có:
22
2 2 2 2 2 2 2 2
2 2 2 0f a b f p q f a b p q f ap bq aq bp
Lưu ý rằng.
22
2 2 2 2
a b p q ap bq aq bp
đẳng thc Brahmagupta
Fibonacci ni tiếng, cũng đã được đề cập đến trong nhiu cun sách, mong bạn đọc u ý
chi tiết này để gii toán.
22
02f x f y f x y
nên
0.f aq bp
Do
,1pq
nên tn ti
,ab
sao cho
1.aq bp
Suy ra được
1 1 0,f f aq bp
điu này là vô lý.
Vy tn ti duy nht s nguyên t
p
sao cho
0.fp
Kh năng 1. Nếu
p
s nguyên t dng
4 1,kk
thì tn ti
a
sao cho
2
1pa
hay

2
1 0.fa
Lưu ý rằng. Kết qu này các bn có th tham kho trong phần chuyên đề Thặng dư bình phương.
Mt khác, trong ii) ta thế
1,x y a
thì ta được:
2 2 2 2
2 1 1 2 1 1 1 1 1,f a f f a f a f f a f a
Điu này là mu thun.
Vy t đấy ch xy ra kh năng còn lại.
Kh năng 2. Nếu
p
là s nguyên t có dng
43k
thì
T đó ta có
0f x p x
1fx
vi các giá tr
x
còn li.
Trường hp 2.
1fp
vi mi s nguyên t
.p
Khi đó
1, \ 0f x x
Vy t đó có hai hàm số tha mãn yêu cu bài toán là:
0
1
if p x
fx
if x
,
p
trong đó
p
là mt s nguyên t bt kì có dng
4 3, .kk
0 0
1 0
if x
fx
if x
Nhn xét. Đây là một bài toán phương trình hàm trên tp ri rc phi nói là rt rt khó, s
dng rt nhiu kiến thc trong S Học cũng như năng phán đoán biến đổi thun
20 | Tạp chí và tư liệu toán hc
Bồi dưỡng hc sinh gii
TẠP CHÍ VÀ TƯ LIỆU TOÁN HC
thc. S dng rt nhiu các mng kiến thức liên quan đến s nguyên t, thặng bình
phương hay các đẳng thc rt ni tiếng trong Toán hc. Thc s đây một bài hàm liên
quan đến s hc tng hp, rt hay và thú v, mong bạn đọc nghiên cu thậtcàng và cẩn
thn bài toán này.
Câu 9. Gi s hàm s
*
:f
thỏa mãn các điều kin sau:
11f








1
1 2 1
2
1 2
2
n
f if n m
fn
n
f if n m
Tìm các giá tr ca
n
sao cho
2019.fn
Li gii
T cách xác định ca hàm
f
ta d dàng tính được:
2 3 2; 4 5 6 7 3f f f f f f
Bây gi ta s viết dưới dng nh phân như sau:
222
2 2 2
1 1 1; 2 10 2; 3 11 2
4 100 3; 5 101 3; 6 110 3;...
f f f f f f
f f f f f f
T cách việt dưới dng nh phân như trên, ta dự đoán
fn
s ch s trong biu din
nh phân ca s
.n
Ta s chng minh d đoán này bng quy nạp như sau.
Tht vy, ta thy khẳng định đúng với
1, 2.nn
Gi s khẳng định đúng đến
.n
Ta s chng minh khẳng định đúng đến
1.n
Nếu
n
là s chn thì
1 1 2
... 0 1.
kk
n a a a f n k
Khi đó thì ta có

1 1 2
1 ... 1
kk
n a a a
T đấy ta
11
2
... 1 1,
2 2 2
kk
n n n
a a a f k f n f k
tc bng s ch s
trong biu din nh phân ca s
.n
Nếu
n
là s l thì bằng cách làm tương tự ta cũng được kết qu tương tự.
Vy theo nguyên lý quy np ta suy ra
fn
là s ch s trong biu din nh phân ca
.n
Ta đó ta suy ra nếu
2019fn
thì biu din ca
n
trong h nh phân chứa đúng
2019
ch s.
Vy t đó ta suy ra:

2018 2019
2 2 .n
Nhn xét. Đây là một bài toán khá hay, với tư tưng giảiđưa về h nh phân. Bng cách
biu th bình thường thì ta không th tìm ra được tính cht ca dãy, bởi điều kin xen
k vi tính chn l, rt khó chu phc tp. ch bằng cách đưa v h nh phân ta đã
Chinh phc olympic toán| 21
Phương trình hàm trên tp ri rc
CHINH PHC OLYMPIC TOÁN
nắm được quy lut ca dãy s tác giá đã ẩn đi trong bài toán, từ đó ta nảy ra ý
ng gii và ch việc đi triển khai, cuối cùng ta thu được kết qu ca bài toán.
Câu 10. Tìm tt c các hàm s
**
:f
thỏa mãn các điều kin sau:

1 1, 3 3
2
4 1 2 2 1
4 3 3 2 1 2
ff
f n f n
f n f n f n
f n f n f n
vi mi s nguyên dương
n
.
IMO 1988
Li gii
Gi s
f
là hàm s thỏa mãn điều kin bài toán.
Mt s nguyên dương
k
ch có th có mt trong bn dng sau:
4 , 4 1, 4 2, 4 3; ,k n k n k n k n k n
Do đó, từ gi thiết ca bài toán, hàm s
f
đưc xác định mt cách duy nht. Ta s s
dng biu diễn cơ số
2
để tìm biu din ca hàm s
.f
Ta có các nhận xét sơ bộ như sau:
2 2 2 2
2 2 2 2
1 1 1 1 , 10 2 1 01
11 3 3 11 , 100 4 1 001 ,...
f f f f
f f f f
T đấy ta thấy được quy luật như sau:
Quy lut. Biu din ca
fn
trong h số
2
chính cách viết ngược li ca biu din
ca
n
trong h cơ số
2
tc là

1 1 0 0 1 1
22
... ... .
k k k k
f a a a a a a a a
Bây gi ta s chng minh d đoán này bằng quy nạp như sau.
Chng minh.
Vi
1, 2,3, 4n
thì hiển nhiên đúng, do ta đã thử kim tra trên.
Gi s tính chất đã đúng cho với mi
.kn
Ta s chng minh tinh chất cũng đúng vi
.n
Trường hp 1. Nếu
2nm
thì theo gi thiết ta có
.f m f n
2nm
nên nếu
m
đưc
biu din trong h cơ số
2
i dng
1 1 0
2
...
kk
m a a a a
thì
1 1 0
2
... 0 .
kk
n a a a a
Mà theo gi thiết quy np thì ta có:
1 1 0 1 1 0 0 1 1 0 1 1
2 2 2 2
... 0 ... ... 0 ...
k k k k k k k k
f a a a a f n f m f a a a a a a a a a a a a
T đây, trong trường hp này, tính chất được chng minh.
Trường hp 2. Nếu
41nm
vi
1 1 0
2
...
kk
m a a a a
thì
1 1 0
2
... 01
kk
n a a a a

1 1 0
2
2 1 ... 1 .
kk
m a a a a
Mà theo gi thiết quy np thì ta có:
22 | Tạp chí và tư liệu toán hc
Bồi dưỡng hc sinh gii
TẠP CHÍ VÀ TƯ LIỆU TOÁN HC







1 1 0
2
1 1 0 1 1 0
22
0 1 1 0 1 1
22
0 1 1 0 1 1
22
0 1 1 0 1 1
2 2 2
0 1 1
22
... 01 4 1 2 2 1
2 ... 1 ...
2 1 ... ...
1 ... 0 ...
10...0 ... 0 ...
10...0 ... 10
kk
k k k k
k k k k
k k k k
k k k k
kk
f a a a a f n f m f m f m
f a a a a f a a a a
a a a a a a a a
a a a a a a a a
a a a a a a a a
a a a a a
0 1 1
2
... .
kk
a a a
T đây, trong trường hp này, tính chất được chng minh.
Trường hp 3. Nếu
43nm
vi
1 1 0
2
...
kk
m a a a a
thì
1 1 0
2
... 11
kk
n a a a a

1 1 0
2
2 1 ... 1 .
kk
m a a a a
Mà theo gi thiết quy np thì ta có:

1 1 0
2
0 1 1 0 1 1 0 1 1
2 2 2
0 1 1
22
0 1 1
2
... 11 4 3 3 2 1 2
2 1 2 2 1 2
1 ... 1 ... 0 ... 0
1 ... 10...0
11 ... .
kk
k k k k k k
kk
kk
f a a a a f n f m f m f m
f m f m f m
a a a a a a a a a a a a
a a a a
a a a a
T đây, trong trường hp này, tính chất được chng minh.
Vy theo nguyên lý quy np thì quy lut của chúng ta đã được chng minh.
Vy tt c các hàm
fn
thỏa mãn đề bài là:

1 1 0 0 1 1
22
... ... ,
k k k k
f a a a a a a a a
trong đó
1 1 0
2
...
kk
n a a a a
là biu din ca s
n
trong h cơ số
2.
Nhn xét. Cũng với ý tưởng tương tự như bài toán 9 s dng h nh phân để tìm ra
quy lut ca dãy số. Nhưng bài toán 10 thì khó hơn các bài trước rt nhiều, do điều
kin của đề bài khá nhiu, làm ta khá hoang mang, và khá nhiều trường hp cn xét ca
n
nên li gii v phc tp. bài này, các bn cn mt cách nhìn tổng quan để khám
phá ra quy lut và kim nghim s chính xác ca nó, t đó đi chứng minh phỏng đoán
đó bằng phương pháp quy nạp, bên cạnh đó cũng cần s biến đổi điêu luyện và tht chính
xác thì mới thu được kết qu ca bài toán.
Chinh phc olympic toán| 23
Phương trình hàm trên tp ri rc
CHINH PHC OLYMPIC TOÁN
Câu 11. Cho hàm s

:f
thỏa mãn đồng thời các điều kin:
fn
là ước ca
2018
n
vi mi
n
.f a f b f c
vi mi
,,a b c

2 2 2
a b c
a) Chng minh rng nếu
n
l hoc
4n
thì
1fn
b) Gi
A
là tp hp giá tr có th có ca
2 2018ff
. Tính
A
Li gii
a) Nếu
1 mod 2n
ta chn

2018
2 2 2 2
1 1 1 1
, , , , | |
2 2 2 2
n n a a
a b c n f n f
Ta có








2018
22
2018 2018 2018
11
| |gcd , gcd , 1 1
22
nn
f n n f n n n f n
Nếu
0 mod 4n
ta chn
2018
2 2 2 2
, , 4 , 4 1, 4 1 4 | 4 1 | 4 1a b c k k k f k f k k
Ta có
2018
2018 2018
2
4 | 4 4 |gcd 4 , 4 1 1f k k f k k k
b) Theo câu a, ta có

2018 0 1 2018
2 2 2 2 , 2 ,...,2ff
Vi

2
, , 2018,1018080,1009 1a b c
ta có
2018
22
2018
2018 1009 1 1009 1
2018 2018
ff
f
2018 2 2018 0 1 2018
2018 gcd 2018,1009 1 2 2018 2 ,2 ,...,2ff
Vi
,,a b c
tho

2 2 2
*
gcd , , 1
a b c
a b c
Nếu
,ab
l,
c
chn.
Do
2 2 2
2 2 2
2 2 2a b c a b c
nên
1 2 2 2f c f a f b
.

:f
nên
2 2 2 1f a f b f c
.
Nếu

1 mod2
0 mod2
ac
b
.
Khi đó:
2 2 2 2 , , 1 f c f a f b f a a c
l tha mãn (*).
Do đó
2 2 2
2 2 * * : , , 1f c f a b a b c a c
.
Ta
2018f
tha
**
vi b



2
2018 1018080 1009 1
, , , ,
2 2 2
a b c
2f
không tha
**
do
,1ac
nên
2 2018ff
D dàng chứng minh được
2 2 2 2
a b c d
vi
,,a b c d
bng cách chia c hai vế cho
2
vi
min , , ,a b c d
nên
2
2019
TC
.
24 | Tạp chí và tư liệu toán hc
Bồi dưỡng hc sinh gii
TẠP CHÍ VÀ TƯ LIỆU TOÁN HC
Câu 12. Có tn ti hàm s
:f S S
thỏa mãn điều kin
22
, , ,f a f b f a b a b S a b
không, trong đó
*
\1S
?
Li gii
Gi s tn ti hàm s tha mãn yêu cu bài toán. Ta s tìm tính chất đặc bit ca hàm s
f
Xét hai biu thc sau
,f a f d f c f a f b f c
.
Ta có

2 2 4 4 2
f a f d f c f a d f c f a d c

2 2 2 4 4
f a f b f c f a f b c f a b c
.
Ta s chn
,cd
sao cho
4 4 2 2 4 4 2 4 4 2 2 2
a d c a b c a d b c ad b c
1.
Vi
,cd
tha mãn (1) thì
4 4 2 2 4 4
2f a d c f a b c f a f d f c f a f b f c f b f d
Tt nhiên ta cn chn
db
c
tha mãn
1
chng hn

22
,d b c ab
.
Nên t
2
ta được
2
,f b f b b S
(3).
T tính cht
3
ta có
22
, , , , , , 4f a f b f a b a b S a b f a f b f ab a b S a b
S dng tính cht
3 , 4
ta được
22
16 4 4 2 2f f f f f
.
Mt khác
16 2.8 2 8f f f f
.
T hai đẳng thức trên ta được
2 8 2 8 1f f f f
(vô lí.)
Vy không tn ti hàm s
f
tha mãn yêu cu bài toán.
Câu 13. Tìm tt c các hàm s
**
:f
thỏa mãn điều kin
2
2*
1,n f n f f n n n n
.
Li gii
Vi dng toán gi thiết v bất đẳng thc thì vic d đoán được nghiệm đóng vai trò
quan trng trong việc định hướng li gii. Vic d đoán nghiệm thường ta thông qua vic
tính các giá như
1 , 2 , 3 ,...f f f
Thay
1n
vào phương trình ban đầu ta được
0 1 1 2 1 1f f f f
.
Thay
2n
vào phương trình ban đầu ta được
1 2 2 6f f f
.
T bất đẳng thc này suy ra
21f
, nếu
23f
.
T điu kiện ban đầu ta thay
n
bi
fn
thu được
2
2
*
1,f n f f n f f f n f n f n n
(1).
T (1), thay
2n
ta được
2
2
2 1 2 2 2 2f f f f f f f f
Chinh phc olympic toán| 25
Phương trình hàm trên tp ri rc
CHINH PHC OLYMPIC TOÁN
2
4 2 1 2 2 2 2f f f f f f f f
Điu này mâu thun vi
1 2 2 6f f f
. Do đó
22f
.
Do đó ta dự đoán
*
,f n n n
. Ta s chng minh bằng phương pháp quy nạp.
Gi s
1 1, 2 2,..., 1 1f f f n n
. Ta chng minh
f n n
Vi mi s nguyên dương
n
, đặt
f n m
. Ta xét các trường hp sau:
Nếu
2
2
11m n f n f f n mf m m n
vô lí.
Nếu
2
11f n m n n f f n f n f f n n n
f f n n
2
2
.f f f n f f n f f n f f f n f f n n
,
Kết hp vi

2
1f f f n f f n f n
ta được
2
2
11n f n f n n
vô lí.
Do đó
f n n
. Vy
*
,f n n n
.
Câu 14. Tìm tt c hàm s
:f
thỏa mãn đồng thời hai điều kin sau:
i)
x f y f x y f x f y
vi mi
,xy
;
ii) Tp hp




, , ,
f x f y
I x y x y
xy
là mt khong
Li gii
Phân tích. Đim mu chốt để gii bài toán chính là hiu và tn dụng được gi thiết ii).
Tp hp
I
là mt khoảng có nghĩa là nếu
,,a b I a b
thì
,a b I
.
T điu này ta suy ra tính cht quan trng ca tp hp
I
nếu s thc
aI
thì hoc
,x a x I
hoc là
,x a x I
.
Khi đó ta có lời gii sau.
Để ý rng nếu
f x f y x y
thì
xy
( do i) ).
Do đó
1 I
.
Gi s
tI
. Khi đó
1t
và tn ti
,,x y x y
sao cho
f x f y t x y
.
xy
nên
f y f x f x f y

1
1
f y f x f x f y
yx
y x t x y t
y f x x f y
.
Do đó
1
1
I
t
. Mà
I
là mt khong nên mi phn t
x
nm gia
t
1
1 t
cũng thuộc
I
Nếu
1t
thì
1
0
1 t
(mâu thuẫn vì khi đó ta lại có
1 I
)
Do đó
1t
. Tương tự ta cũng có
1
1
1 t
, tc là
0t
.
26 | Tạp chí và tư liệu toán hc
Bồi dưỡng hc sinh gii
TẠP CHÍ VÀ TƯ LIỆU TOÁN HC
Như vậy ta đã chứng minh được rng nếu
tI
thì
1
1
I
t
0t
.
Mà khi
0t
thì
1
0
1 t
nên ta có mâu thun.
Như vậy
I
phi là tp rng mà rõ ràng
I
, chng hn
10f f I
.
Do đó không tồn ti hàm s tha mãn yêu cu bài toán.
Ngoài ra ta cũng có thể giải bài toán này như sau.
Đặt
,
f x f y
F x y
xy
vi mi
,,x y x y
.
Tương tự cách đầu, ta có
, 1, , y , x yF x y x
Nếu tn ti
xy
sao cho
,0F x y
thì
,1F y f x x f x
(vô lý).
Do đó
, 0;1 , , , *F x y x y x y
Suy ra
f
là đơn ánh. Do đó với mi
,,x y x y
ta có
f x f y f y f y f y f x f y f y x y
Hay
, , , 1F x f y y f y F y f x y f y F x y
T đây, kết hp vi
*
ii) ta suy ra
, 1, , ,F x y x y x y
Suy ra
f
đơn điệu tăng trên .
Nếu
xy
thì
f x f y x y
hay
y f x x f y
.
Suy ra
f y f x f x f y
, kết hp vi i), ta được
xy
.
Tương tự nếu
xy
ta cũng có mâu thuẫn.
Vy không tn ti hàm s tha mãn yêu cu bài toán.
Câu 15. Tìm các hàm s
**
:f
tha mãn
2
2 2 *
,,f m f n m n m n
Li gii
Gi s tn ti hàm
f
tha mãn yêu cu bài toán
Ta có
2
1 1 4 1 1f f f
Thay
2
1 1 1 ,m f n n n
Thay
1, 1m n p p P
2
2
11
11
11
f p p
f p p
f p p
Nếu
2
: 1 1p P f p p
.
Thay
2
2
2
1, 1 1 1| 1 1m p n f p p
Suy ra
22
22
1 1 2 2p p p
Chinh phc olympic toán| 27
Phương trình hàm trên tp ri rc
CHINH PHC OLYMPIC TOÁN
22
1 2 2p p p
nên có mâu thun
Vy
1 1,f p p p P
Thay


2
22
1 1 1m p p f n p n
Suy ra
2
22
1 0 mod 1n p p f n
2
2
0 mod 1f n n p f n
Suy ra
2
2
0 mod 1f n n p f n
Nếu
f n n
, khi đó ta có
2
2
( ) 1 , *f n n f n p p P
Mà tp các s nguyên t là vô hn nên
2
1f n p
.
2
f n n
c định vi
n
xác định.
Do đó chỉ cn chn
p
nguyên t đủ ln ta có mâu thun
*
.
Vy
*
,f n n n
(th li tha mãn)
Câu 16. Cho hàm
,f x y
thỏa mãn các điều kin:
0, 1; 1,0 ,1f y y f x f x
1, 1 , 1,f x y f x f x y
Vi mi s nguyên không âm
,xy
. Tìm
4,1981f
Li gii
Ta có
1, 0, 1, 1 1 1, 1f n f f n f n
Do đó
1, 1,0 0,1 2f n n f n f n
Ta li có
2, 1, 2, 1 2, 1 2f n f f n f n
Do đó
2, 2 2,0 2 1, 1 2 3f n n f n f n
Bây gi
3, 2, 3, 1 2 3, 1 3f n f f n f n
Đặt
1
2
nn
uu
0
3,0 3 2,1 3 0u f f
Do vy

33
2 3, 2 3
nn
n
u f n
Ta có


4, 1 3
4
224
4, 3, 4, 1 2 3
4,0 3,1 2 3 13
4, 2 2 3
fn
f n f f n
ff
f
Bng qui np ta chứng minh được

22..24
4, 2 3fn
Trong đó số mũ chứa
2n
ch s 2. T đó

22.24
4, 1981 2 3f
vi s mũ chứa 1983
ch s 2.
28 | Tạp chí và tư liệu toán hc
Bồi dưỡng hc sinh gii
TẠP CHÍ VÀ TƯ LIỆU TOÁN HC
Câu 17. Cho hàm

:f
thỏa mãn các điều kin sau:
iii)
1;f n f n n
iv)
3 , .f f n n n Z
Hãy tính
2003 .f
Li gii
T
1, 1 3 1 2f f f fi ii
Ta có
2 1 3.1 3f f f
3 3 3.2f f f
2
2.3 3 3.3 3f f f
.............
Suy ra
1
2.3 3 , ; 3 2.3 ;
n n n n
f n f n Z
Nên có


11
3 2.3 2.3
n n n
f f f
1 1 1 2
2.3 3 3.3 3
n n n n
f f f
Do đó khẳng định đúng với mi n
Ta
31
n
s nguyên m nm gia
3
n
2.
3
n
do gi thiết
i
1f n f n
nên
31
n
s nguyên m nm gia
3
n
f
2.3
n
f
suy ra
03
n
m
3 2.3 3
nn
f m n
.
Do gi thiết
ii
suy ra
2.3 3 3 3
n n n
f m f f m m
Vy
2.3 3 3
nn
f m m
vi
03
n
m
Suy ra
66
2003 2.3 545 2003 3 3 545 3822nf
.
Câu 18. Cho
fn
hàm s xác định vi mi
*
n
ly giá t không âm tha mãn
tính cht:
*
,:n m f m n f m f n
ly giá tr 0 hoc 1
20f
30f
.
9999 3333f
.
Tính
2000f
.
Li gii
f m n f m f n
ly giá tr 0 hoc 1 nên ta suy ra
f m n f m f n
2 2 1 1 0 3 1f f f f
Ta có
6 3 3 2f f f
9 6 3 3f f f
.................
9999 9996 3 3333f f f
Chinh phc olympic toán| 29
Phương trình hàm trên tp ri rc
CHINH PHC OLYMPIC TOÁN
Vì gi thiết cho
9999 3333f
nên ta có dấu “=” ở các bất đẳng thc trên xy ra, tc
3 , 1, 2,..., 3333f n n n
1998 666, 2001 667ff
Mt khác nếu a

*
,ab
a b f a f b f a b f b
.
666 2000 667 2000 666ff
hoc 667
Gi s
2000 667f
4000 1334 6000 1334 667 2001ff
.
6000 2000f
, mâu thun. Vy
2000 666f
.
Câu 19. Cho
,fg
là các hàm xác định trên thỏa mãn điều kin
2 . , ,f x y f x y f x g y x y
Chng minh rng nếu
0fx
1,f x x
thì

0
1g y a
Li gii
Ta dùng phương pháp phản chng
Gi s li một điểm
00
:1y g y a
Ta ly
00
:0x f x
và xây dng dãy
0,1, 2....
k
xk
như sau:
0 0 0
1
0 0 0
,khi
,khi
k k k
k
k k k
x y f x y f x y
x
x y f x y f x y
Theo gi thiết ta có
1 0 0 0 0
2
k k k k k
f x f x y f x y f x y f x y

0
22
kk
f x g y a f x
Nên
1kk
f x a f x
vi
1; 1, 2,3...ak
Do đó ta có:
0
.
k
k
f x a f x
Nhưng vì
0
0fx
1a
nên có th chn k sao cho
0
1
k
a f x
dó đó
1
k
fx
Mâu thun vi gi thiết.
Vy
1,g y y R
Câu 20. Cho hàm s
:f
thỏa 2 điều kin
iii)
1;f x x x
iv)
. ; ,f x y f x f y x y
Chng minh rng không th tn ti hai s
;ab
.0f a f b
Li gii
Ta s chng minh
0,f x x
Tht vy! Vi
1x
thì theo điều kin
i
ta có ngay
0fx
30 | Tạp chí và tư liệu toán hc
Bồi dưỡng hc sinh gii
TẠP CHÍ VÀ TƯ LIỆU TOÁN HC
Vi
1x
, trước hết ta s chng minh bất đẳng thc:






2
, , 1
2
n
n
x
f x f x n
Vi
0n
thì bất đẳng thức đúng!
Gi s
1
đúng với
0nk
tc






2
2
2
k
k
x
f x f
Ta có
1
2 2 .2
2
1 1 1 1
2 2 2 2 2
kk
k
k k k k k
x x x x x
f f f f
tc
1
đúng với
1nk
Theo nguyên lý quy np toán hc bất đẳng thc
1
đúng.
Bây gi chọn n đủ lớn để
2,
n
xx
tùy ý, khi đó



10
22
nn
xx
f
Do đó






2
0
2
n
n
x
f
tc
0,f x x
Như vậy không th tn ti hai s
;ab
.0f a f b
.
Câu 21. Cho
2003
, y cos 2 cos
2
f x x y a x y
vi
,a
.
Chng minh rng

22
min , max , 2003.f x y f x y
Li gii
Ta có





2003
0,0 ; 2
2 2 2
ff
Nên





2003
max , max 0,0 , , ,
2 2 2
f x y f f x y

2
2003
max ,
2
f x y
Ta li có
2003 2003
; .sin , ; .sin
4 4 2 4 4 2
f a f a
Nên
2003
;;
4 4 4 4 2
ff
.
Suy ra


2003
min , min ; , ; ,
4 4 4 4 2
f x y f f x y

2
2003
min , .
2
f x y
Do đó

22
min , max , 2003.f x y f x y
Chinh phc olympic toán| 31
Phương trình hàm trên tp ri rc
CHINH PHC OLYMPIC TOÁN
Câu 22. Cho hàm s
2
1
, 0.
2
x
f x x
x
Gi s
0
f x x
*
1
,0
nn
f x f f x n x
.
Chng minh
2
1
1
, 1,0,1 1
1
1
n
n
n
fx
nx
fx
x
f
x



Li gii
Đặt



22
1
11
2
nn
n
p x x x



22
1
1 1 , ,
2
nn
n
q x x x x y
Ta có:

22
1
1
0
0
0
2 , ,
,0
1
n n n
n n n
p x p x q x
q x p x q x x y
px
x
f x x x
qx
Gi s
k
k
k
px
fx
qx



2
22
1
1
1
1
2
2.
k
k
k k k
k
k
k k k
k
px
qx
p x q x p x
fx
px
p x q x q x
qx
Do đó
, , 0
n
n
n
px
f x n x
qx
Ta có
, 1,0,1nx
thì có:
1
1
2 2 2 2 1
2 2 2 2 1
1
1 1 1 1
1 1 1 1
n n n n
n n n n
n
n
x x x x
fx
fx
x x x x









11
2
22
22
2 2 1 2 2 1 2
11
2 1 1
1
11
1 1 1 1
1
1
nn
nn
n n n n n
xx
xx
x x x x
x
f
x
Câu 23. Cho hàm s

* * *
:f
là hàm s thỏa mãn đồng thời các điều kin sau:
i)
1,1 2f
ii)
*
1, , , ,f m n f m n m m n
iii)
*
, 1 , , ,f m n f m n n m n
Tìm tt c các cp s
,pq
sao cho
, 2019.f p q
Doãn Quang Tiến
Li gii
Áp dụng điều kin ii) ta có:
32 | Tạp chí và tư liệu toán hc
Bồi dưỡng hc sinh gii
TẠP CHÍ VÀ TƯ LIỆU TOÁN HC
1
, 1, 1 2, 2 1 ... 1,
2
pp
f p q f p q p f p q p p f q
1 1 1
1, 1 1 ... 1,1 2019,
2 2 2
p p q q p p
f q q f
áp dụng điều kin
iii)
T điu kin i) ta có
1,1 4f
nên t đẳng thc trên ta suy ra:

1 1 1 1
1, 1 2 2019
2 2 2 2
11
2017 1 2.2017
22
q q p p q q p p
f
p p q q
p q p q
2017
là s nguyên t
1p q p q
nên có th xảy ra các trường hp sau:
Trường hp 1.
1pq
1 4034pq
thì t đó ta được:
2018, 2017.pq
Trường hp 2.
2pq
1 2017pq
thì t đó ta được:
1010, 1008.pq
Vy tt c các cp s
,pq
sao cho
, 2019pq
là:
, 2018,2017 , 1010,1008 .pq
Câu 24. Tìm tt c các hàm s
:f
thỏa mãn các điều kin sau:
i)
2
0,f x x n
ii)
f x f y
chia hết cho
xy
vi mi
,,x y x y
Li gii
Ta s phải xét hai trường hợp dưới đây.
Trường hp 1.
f
là hàm s hng.
Gi s
const,f x c c
là hng s thuc
.
Cho
0x
thì t i) ta suy ra:
0 0 0 0 0.ff
Vy
0, .f x x
Trường hp 2.
f
không là hàm s hng.
00f
nên khi cho
0y
thì t điu kin ii) ta được:
, \ 0 .x f x x
Vi
1x
thì t điu kin i) ta suy ra:
10f
hoc
1 1.f
Kh năng 1.
1 0.f
Cho
1y
thì t điu kin ii) ta suy ra:
1 , \ 1 .x f x x
Mà ta đã
, \ 0x f x x
, 1 1xx
Nên
2
11x x f x x x x x f x
Mt khác ta có
2
f x x
x f x

2
2
f x x x
f x x
Ta loại trường hp
2
f x x
1 1 0.f
Chinh phc olympic toán| 33
Phương trình hàm trên tp ri rc
CHINH PHC OLYMPIC TOÁN
Vy trong kh năng 1 này ch có hàm s
2
,f x x x x
tha mãn yêu cu bài toán.
Kh năng 2.
1 1.f
Khi
, \ 0x f x x
1 , \ 1x f x x
thì ta ly
k
tùy ý sao cho
,f k mk
vi
m
tùy ý
T điu kin ii)
1 1 1 1 1 1 1k mk k mk mk m m k m
Mà t điu kin i) ta được
22
f k k mk k m k
Mt khác ta li có


11
11
10
km
k m k m
m
Do đó
10m
nên suy ra:
1m
hoc
.mk
Vy t đó ta được
2
,f x x x
hoc
,.f x x x
Th li thì thy các hàm s này tha mãn yêu cu bài toán.
Vy tt c các hàm s thỏa mãn đề bài là
22
0, , , , .f x f x x f x x f x x x x
Câu 25. Tìm tt c các hàm s

**
:f
mà tp
*
0xx
tha mãn:
*
2 , , 1
f xy
f x f y xyf xy x y
f x y
Li gii
Ta s giải bài toán này thông qua ba bước sau đây.
c 1. Ta s chng minh
1 1.f
Tht vy, cho
1y
vào
1
và đặt
1fa
thì ta được
2 1 2
1 1 2
f x f x
f x a xf x f x
f x x f x a
T đó ta suy ra
2
1
1 1 1
4
2 , 3 , 4 *
5
4 4 5 4 7 5 4
4
a
f f f
a a a a
a
Mt khác, ta cho
2xy
vào
1
thì ta được
4
2 2 8 4 1
4
f
ff
f
Mà t
*
ta suy ra

2
11
2. 8. 1 1 1 1.
4 7 5 4
af
aa
T đây ta xong c 1.
c 2. Ta s chng minh

2
, 1, 3
21
fx
f x n n
n nx f x
Ta s dùng phương pháp quy nạp để chng minh khẳng đnh này.
34 | Tạp chí và tư liệu toán hc
Bồi dưỡng hc sinh gii
TẠP CHÍ VÀ TƯ LIỆU TOÁN HC
T
2
suy ra
3
đúng khi
1.n
Gi s đúng đến
.nk
Thì ta có
1
1 2 1
f x k
f x k
x k f x k








2
2
2
21
12
1 2 1 1
1
21
fx
k kx f x
fx
x k f x
k k x f x
k kx f x
T đây theo nguyên lý quy nạp ta suy ra điều phi chng minh.
Vy khẳng định c 2 đưc chng minh.
Vy t đó ta suy ra

2
2
1
1
1
2 1 1
1
f
fn
n n f
n
hay
2
1
, 1,f n n
n
c 3. Ta s chng minh:






2
2
11
, 1, 4
1
f n n
n
n
Tht vy, trong
3
ta thay
1
x
n
thì được











2
1
1
1
21
f
n
fn
n
nf
n
Tiếp theo, ta thay
1
y
x
vào
1
thì được






11
2
1
f x f
x
fx
x
Vy



2
1 1 1
22
11
f n f n
n
f n f
nn
Mà ta có:
2
1
fn
n
nên suy ra:



2
1
.fn
n
Nên t đây c 3 đưc chng minh hoàn toàn.
Th li thì thy hàm s này tha mãn yêu cu bài toán.
Vy tt c các hàm s tha mãn yêu cu bài toán là
*
2
1
,.f n n
n
Chinh phc olympic toán| 35
Phương trình hàm trên tp ri rc
CHINH PHC OLYMPIC TOÁN
Câu 26. Cho hàm
:f
mt hàm s tha mãn vi mi
1n
thì mt s
nguyên t
p
là ước ca
n
sao cho:




1
n
f n f p f
p
2018 2019 2020
3 5 7 2017.f f f
Hãy tính giá tr ca biu thc
2018 2019 2020
2018 2019 2020G f f f
Doãn Quang Tiến
Li gii
Thay
np
vào
1
thì ta được
1
1
2
f
f p f f p f p
Thay
n
bi
n
p
thì ta được

1nn
f p f p f p
Bằng phương pháp quy nạp thì ta chng minh đưc.



2
1 *
2
n
n
f p f
Tht vy, thì
*
đúng với
1.n
Gi s
*
đúng với
1.nk
Ta s chng minh
*
cũng đúng với
1nk
hay ta có







1
1 2 1
2
11
2 2 2
kk
fk
k
f p f p f p f f
T đây ta suy ra
*
cũng đúng trong trường hp
1.nk
Vy theo nguyên lý quy np thì
*
đúng với mi
.n
Khi đó thì ta suy ra
2018 2019 2020
3 5 7 2017
2 2018 2 2019 2 2020
1 1 1 2017
2 2 2
6051 2
1 2017 1 .
23
f f f
fff
ff
T

2
1
3
f
1
2
f
fp
suy ra:

1
.'
3
fp
Khi

1
3
fp
thì
1
đưc viết lại như sau



1
, 2
3
n
f n f n
p
Cho
k
s các tha s nguyên t ca
,n
khi đó số ng tha s nguyên t ca



n
p
1.k
Vi
1k
thì ta chn
2n
thì theo
2
ta được:
1 1 2
2.
33
f
36 | Tạp chí và tư liệu toán hc
Bồi dưỡng hc sinh gii
TẠP CHÍ VÀ TƯ LIỆU TOÁN HC
Vi
2k
thì ta chn
10 2.5n
thì theo .
2
.
ta được:



10 1 1 1 1 2 2
10 5 0
2 3 3 3 3 3
f f f
Vi
3k
thì ta chn
12 2.3.4n
thì theo
2
ta được:
12 1 1 6 1 1 1 1 1 1 1 1 3 2
12 6 2
2 3 3 3 3 3 3 3 3 3 3 3 3
f f f f f
T đó, bằng phương pháp quy nạp ta suy ra được:
2
,
3
k
fn
vi
k
là s các tha s nguyên t ca
* *n
Mà lưu ý rằng
2
2018 2.1009, 2019 3.673, 2020 2 .5.101.
Do đó suy ra:
2018
2018
4036
tha s nguyên t,
2019
2019
4038
tha s nguyên t,
2020
2020
8080
tha s nguyên t.
T đấy theo công thc
**
ta suy ra được:
2018 2019 2020
4036 2 4038 2 8080 2 16148
2018 2019 2020 .
3 3 3 3
G f f f
Vy t đấy ta có kết qu ca bài toán.
Nhn xét. Một điều thú vkhi ta thay
2017, 2018,2019,2020
bng
1, , 1, 2x x x x
thì
ta được kết qu ca

2
1
3
f
không thay đổi.
Câu 27. Tìm tt c các hàm s
**
:f
tha mãn:
3 2 2 2 2 *
2 , ,f m n f m f n f m f n m n
Li gii
Gi s tn ti hàm
f
tha mãn yêu cu bài toán.
Trường hp 1. Nếu
f
là hàm s hng.
Tc là
,f n const c
vi
c
là hng s thì hin nhiên tha mãn yêu cu bài toán.
Trường hp 2. Nếu
f
không là hàm s hng.
Nếu tn ti
*
,mn
sao cho
f m f n
thì ta gi
,ab
là hai s tha mãn:
*
min , , 1f a f b f m f n m n
Gi s
f a f b
thì ta có

3 2 2 3
22f b f a f b f a f b f a
Suy ra
2 2 2 2
f b f a b f a f a b f b f a f b
T đó thì ta suy ra
2 2 2 2
2f a f b f a f b f a b f b f a b f b
Rõ ràng thì ta thy
2
mâu thun vi
1.
Chinh phc olympic toán| 37
Phương trình hàm trên tp ri rc
CHINH PHC OLYMPIC TOÁN
Do đó chỉ
f
là hàm s hng tha mãn yêu cu bài toán.
Vy tt các hàm s thỏa mãn đề bài là:
const ,f n c
vi
c
là hng s.
Câu 28. Gi s
:f
là hàm liên tc và gim sao cho vi mi
,xy
ta có
f x y f f x f y f y f x
Chng minh rng
f f x x
Li gii
Cho
yx
ta được:
2 2 2f x f f x f f x f x
Thay
x
bng
fx
ta có
2 2 2f f x f f f x f f f x f f x
Tr hai phương trình trên ta suy ra:
2 2 2 2f f f x f x f f f x f f x f f x f x
Nếu
f f x x
, vế trái của phương trình trên âm, do đó:
f f x f f x f x f x
f x f f x x f x
là điều mâu thun.
Tương tự, ta cũng có điều mâu thun xy ra khi
f f x x
Vy
f f x x
, điều phi chng minh.
Câu 29. Cho song ánh
:f
. Chng minh rng tn ti s b
,,a b c
vi
,,a b c
tha mãn
a b c
2 f b f a f c
Li gii
Ta xây dng dãy
n
a
như sau:
Trong các s t 0, 1, 2,..., m chn s
1
a
sao cho
11
0;f a f i i a m
Chn
21
aa
sao cho
22
, 0;f a f i i a
Chn
1kk
aa
sao cho
, 0;
kk
f a f i i a
Vy ta có dãy
1 2 1
...
kk
a a a a
1 2 1
...
kk
f a f a f a f a
Trong đó

i
a
0;
ii
f a f j j a
Vì f là song ánh nên
*
1
,
kk
f a f a p p N
c
để

11kk
f c f a p f a
Mt khác

1
11
, 1,
, 1,
ki
kn
a a i k
f a f i i a
38 | Tạp chí và tư liệu toán hc
Bồi dưỡng hc sinh gii
TẠP CHÍ VÀ TƯ LIỆU TOÁN HC
Nên
1k
ca


1
1
1
2
kk
kk
k
p a f a p
f a f a f c
f c f a p
Do cách xây dng, dãy
n
a
dãy hn nên tn ti s b
,,a b c
thỏa điều kiện đã
nêu.
Câu 30. Có bao nhiêu hàm
:f
**
tho mãn đồng thời các điều kin sau
a)
11f
b)
2
2 9 1 1997,f n f n f n
 *.n
Li gii
Gi
D
là tp hp tt c các hàm s
f
tho mãn điều kin bài toán.
Theo gi thiết b) ta có
2
2 1 1997f n f n f n
;
2
1 3 2 1997f n f n f n
Suy ra
22
2 1 1 ( 3 2 1997f n f n f n f n f n f n

2 1 3
, *.
12
f n f n f n f n
n
f n f n
Vì vy ta có
1 3 2 4 2
... ...
2 3 1
f f f f f n f n
f f f n
Đặt
13
2
ff
c
f
1
suy ra
2 1 , * 2f n cf n f n n
Ta chng minh
*.c
Tht vy, nếu
p
c
q
vi
,pq
,1pq
thì t
2
ta có
2 1 , *q f n f n pf n n
Suy ra
1 , *q f n n
hay
2
2 , *q f n f n n
2.n
2
2
1997 2 1 .f n f n f n q
1997
là s nguyên t nên
2
1q
hay
1q
suy ra
*c
Gi
2,fa
do
1
ta có
13ac f
suy ra
2
1 3 1 3 2 1997ac f f f f
2
1 1997ac a
1998a c a
Ta được
1998a
, hay
2f
là một ước dương của 1998.
Ngược li vi mỗi ước dương
a
ca
1998
ta xây dng hàm
: * *f
như sau
1 1;f
2fa
*
2 1 ,f n a b f n f n n
; trong đó

1998
*.b
a
Chinh phc olympic toán| 39
Phương trình hàm trên tp ri rc
CHINH PHC OLYMPIC TOÁN
Ta chng minh
f
tho mãn điều kiện đề bài, nghĩa là
fD
.
Tht vy
22
22
2
2
1 3 2 1 2 1 2
1 2 1 2
2 1 2 1
2 1 , *
f n f n f n f n a b f n f n f n
a b f n f n f n f n
f n a b f n f n f n
f n f n f n n
Suy ra
22
1 3 2 2 1f n f n f n f n f n f n
...
22
3 1 2 3 2f f f f f
T đó ta có
22
2 1 3 2f n f n f n f f
2
2
2 1 2 1a b f f f a b a a
1 1998 1 1997.ab
Vậy ta được
2
1 1 1997f n f n f n
hay
fD
Ta có tương ứng, mi
fD
vi mt giá tr
2 1998f
là mt song ánh gia
D
và tp các
ước dương của
1998
.
Do đó số phn t ca
D
là:
3
1998 2.3 .37 1 1 3 1 1 1 16.D d d
Vì vy có tt c 16 hàm s tho mãn đề bài.
Câu 31. Tìm tt c các hàm s
**
:f
sao cho.
a)
22f
b)
..f m n f m f n
vi mi
*
,mn
,
,1UCLN m n
c)
f m f n
*
,,m n m n
.
Li gii
Chn
1n
, thay vào
. . 1 1f m n f m f n f
.
Ta để ý rng
3 . 5 15 2 . 9 2 . 10 2 . 2 5f f f f f f f f f f
.
Vy
3 2 . 2 4f f f
. Mà
2 2 3 4ff
nên
33f
.
T đó ta tính được
4 4, 5 5, 6 6, 7 7, 8 8, 9 9, 10 10f f f f f f f
.
D đoán
f n n
vi

*
n
.
Gi s
f k k
vi
*
,10k k n
. Ta chứng minh điều khẳng định vẫn còn đúng với
1kn
.
Nếu
k
là s chẵn, ta xét hai trường hp sau:
*
2 2 1 ; ,k l l
.
40 | Tạp chí và tư liệu toán hc
Bồi dưỡng hc sinh gii
TẠP CHÍ VÀ TƯ LIỆU TOÁN HC
Lúc này
2 2 1 2 2 1 2 2 1f k f l f f l l k
.
*
2;k
Lúc này
1 1 1
2 2 2 2 2 1 2 2 1 2 2 1 2f k f f f f k
.
Mt khác
1 1 1 2 2k f k f k f k f k k
.
Do đó
, 1 1f k k f k k
.
Nếu
k
là s l thì
1k
là s chẵn, ta xét hai trường hp sau:
*
1 2 2 1 ; ,k l l
. Khi đó
0 2 ,0 2 1n l n
.
Theo gi thiết quy np
1 2 2 1 2 2 1 2 2 1 1f k f l f f l l k
.
1 1 1 1k f k f k f k k f k k
.
*
1 2 ;k
. Lúc này
1 1 1
1 2 2 2 2 2 1 2 2 1 2 2 1 3f k f f f f k
Mt khác
1 1 1 2 3 3k f k f k f k f k f k k
.
Do đó
, 1 1, 2 2f k k f k k f k k
.
Theo nguyên lí quy np
f n n
vi

*
n
.
Câu 32. Tìm tt c các hàm s
:f
tha mãn
1, ,f m n f mn f m f n m n
1
Li gii
Thay
0mn
vào
1
ta được 2
2
0 0 1 0 1f f f
Thay
1, 1mn
vào
1
ta li có
1 1 1 1f f f
Vy
10f
. hoc
11f
Xét
11f
. Thay
1n
vào
1
ta có :
1 1,f m f m f m m
Suy ra
( 1) 1,f m m
hay
( ) 1,f m m
. Th li tha mãn.
Xét
10f
Thay
1n
vào
1
ta được
1 1,f m f m m
2
Thay
1n
vào
1
ta lại được
1 1 1,f m f m f f m m
Đặt
1af
thì phương trình trên trở thành
1 1,f m f m af m m
3
+ Vi
2a
. Nếu
1a
thì dn ti
11f
(trường hợp này đã giải trên)
Do đó ta xét
1a
.
Khi đó




11
3 1 1 ,
22
f m a f m m
aa
Chinh phc olympic toán| 41
Phương trình hàm trên tp ri rc
CHINH PHC OLYMPIC TOÁN




11
1 1 ,
22
f n a f n n
aa




1
11
1 1 ,
22
n
f n a f n
aa

1
11
,4
2
n
a
f n n
a
Đặt
1ba
1
1
\ 0,1 , ,
1
n
b
b f n n
b
.
Thay vào
2
ta được
1
1
m
b
b
1
1
1,
1
m
b
m
b
1
1 1 1
mm
b b b
,
m
1
1,
mm
b b b m
5
Thay
2m
vào
5
ta được
2
2
1
1 1 1 0 1b b b b b
b
1b
T đó suy ra

1
11
,
2
n
f n n
, hay
0fn
khi n l
1fn
khi n chn.
Th li tha mãn.
+ Vi
2a
thì
12f
.
Thay
1n
vào
1
ta được
1 1,fmmmf
6
T
12f
6
ta dùng phương pháp quy nạp toán hc thì s chứng minh được
 1,f nnn
.
Th li thy tha mãn. Vy các hàm s tha mãn yêu cầu đề bài là

1
1
1
2
,
1
1 ,
n
n fnf nfn
với
n
.
Câu 33. Tìm tt c các hàm s
:f
tha mãn
02f
2 2 2 , ,f x f x y f x f y x y
1
Li gii
Thay
0xy
vào
1
ta được
0 2 0 2 4f f f f
Thay
0x
1y
ta được
2 2 2 4 6f f f f
Ta s chng minh bng quy np rng vi
x
thì
2 2 2f x x
2
Theo trên thì
2
đúng khi
0x
.
Gi s
2
đúng tới
,0xkkk
.
Thay
0, x y k
vào
1
ta được
2 0 2 2 1 2 1 2f f k f f k f k k
Vy
2
cũng đúng khi
1xk
, suy ra
2
đúng với
, 0.xx
Vi
, 0,xx
thay
, 2 ,x y x x
vào
1
ta được
42 | Tạp chí và tư liệu toán hc
Bồi dưỡng hc sinh gii
TẠP CHÍ VÀ TƯ LIỆU TOÁN HC
2 0 4 2 , , 0
2 1 2 2 2 , , 0
2 1 2 2 2 2 2 , , 0
2 2 2, , 0
2 2 2, , 0
(2 ) 2 2,
f x f f x f x x x
f x f x f x x x
x x f x x x
f x x x x
f x x x Z x
f x x x Z
Dó đó (1)
1 2 2 2 4, ,f x f x y x y x y
3
Ta s chng minh nếu x là s nguyên l thì
fx
cũng là số nguyên l.
Tht vy, nếu
2f x k
, vi
k
, thay
2xxk
(vi x l) và
yk
ta được :
2 2 2 2 4, ,f x k f x xx k k x
l
2 2 4, ,f x x k x x
l
2 2 2 4, ,k x k x x
l
Do đó
4 2 4,kx
vi mi s nguyên l x.
Điu này vô lí vì
24x
không phải lúc nào cũng chia hết cho 4.
Như vậy nếu x l thì
fx
l.
T đó nếu x l thì
2xy
l, dẫn đến
2f x y
lẻ, do đó
2x f x y
chn
Do đó
2 2 2f x f x y x f x y
.
Kết hp vi
3
ta được
2 2 2 2 4 2 2 2,x f x y x y f x y x y
,,x y x
l
 2,f xxx
Th li tha mãn.
Câu 34. Tìm tt c hàm s
:f
sao cho
2 3, 1f f n f n n n
Li gii
Gi s tn ti hàm s
fn
tha mãn yêu cu bài toán.
Cho
0n
, t
1
0 0 3 0 3 2f f f f n
Nếu
00f
thì
0 0 0f f f
mâu thun
2
. Vy
00f
Nếu
02f
thì t
2
ta có
0 1 2 0 1 1 2 2.2 3 2 6f f f f f f f f f
6 1 2.1 3 1 1f f f f
Suy ra
6f
, loi.
Như vậy
02f
. Tương tự cũng có
03f
.
Chinh phc olympic toán| 43
Phương trình hàm trên tập ri rc
CHINH PHC OLYMPIC TOÁN
Do đó
01f
. Khi đó từ
2
ta có
1 0 2 2 1 2.1 3 1 3f f f f f f f
Ta s chng minh hàm cn tìm là
1,f n n n
bng quy np toán hc.
Tht vy. Vi
0n
thì
0 1 0 1f
.
Gi s khẳng định đúng tới
,( )n k k
. Tc là:
( ) 1f k k
Vi
1nk
ta có
1 2 3 2 3 1 1 1f k f f k k f k k k k
Vy khẳng định đúng với
1nk
Theo nguyên lý quy np toán hc, ta có
1,f n n n
.
Th lại hàm tìm được tha mãn yêu cu bài toán.
Câu 35. Chng minh rng tn ti duy nht hàm s
: * *f
tha mãn
, , *f m f n n f m b m n b
i
Li gii
Gi s tn ti hàm s
fn
tha mãn yêu cu bài toán.
Ta chng minh f là đơn ánh.
Tht vy, gi s
1 2 1 2
, , *f n f n n n
.
T
i
ta có
1 2 1 2 1 2
f m f n f m f n n f m b n f m b n n
Vy f là đơn ánh.
Vi
 *n
, ta có
1 1 1 1f f f n n f f b n f b b f b f n
11f n b f n f
, vì f là đơn ánh
11f n f n f b a
, vi
1a f b
.
Suy ra
1 ... 2 1 1 , *f n f n f f f b a n
T đó:
,*f n b na f n na b n
.
Lúc này
f m f n n f m b f m na b n m b a b
m na b a b n ma ba b
22
11na n a a
, vì nếu
1a
thì
*fn
khi
nb
.
Suy ra
,*f n n b n b
.
Th li hàm vừa tìm được tha mãn yêu cầu đề.
44 | Tạp chí và tư liệu toán hc
Bồi dưỡng hc sinh gii
TẠP CHÍ VÀ TƯ LIỆU TOÁN HC
Câu 36. Hãy xác định tt c hàm s
**
:f
thỏa mãn đẳng thc:
1 2 . 3f n f n f n f n a
1
Vi a là s t nhiên tha mãn
1a
là s nguyên t
Li gii
Gi s phương trình có nghiệm.
Thay
n
bi
1n
lúc đó
1
tr thành
1 2 3 4 2f n f n f n f n a
Ly
21
vế vi vế ta được:
2 3 4 2f n f n f n f n f n
3
Thay n bi 1 lúc này
3
tr thành:
3 1 4 5 3f f f f f
Thay n bi 2 lúc này
3
tr thành:
4 2 5 6 4f f f f f
Thay n bi 3 lúc này
3
tr thành:
5 3 6 7 5f f f f f
Thay n bi 4 lúc này
3
tr thành:
6 4 7 . 8 6f f f f f
T đây ta đã nhận ra quy luật đặc bit của bài toán này đó chính là nếu thay n bi s l thì
ta luôn biu th đưc:
3 1 4 . 6 ... 2 . 2 1 2 1f f f f f n f n f n
4
Nếu thay n bi các s chn ta s đưc một đặc bit khác
4 2 5 7 ... 2 1 2 2 2f f f f f n f n f n
5
Nếu
13ff
thì lúc này ta s ly tc có ngay
2 1 2 1f n f n
lúc này s
vô s s bé hơn
1f
1f
là 1 s hu hn. Suy ra vô lí. Tc là
31ff
.
Tương tự ta cũng sẽ
42ff
.
Nếu
13ff
24ff
lúc này
2 1 2 1f n f n
2 2 2f n f n
.
Suy ra
31ff
42ff
s có vô s ước s khác nhau (vô lí).
T đó chúng ta sẽ có 3 trường hp
Trường hp 1.
3 1 2 1 2 1
4 2 2 2 2
f f f n f n
f f f n f n





, lúc này
1 2 1
22
f f n
f f n

Thay bởi 1 vào phương trình
1
lúc này
1 2 3 . 4f f f f a
Mà li có
31
42
ff
ff
1 2 2 . 1 1 1 2 1 1f f f f a f f a
Mà li có
1a
là s nguyên t nên ta có
12
22
2
21
2
1
2 2 1
22
f
khi n k
fn
fa
a khi n k
a khi n k
fa
fn
khi n k
f





Trường hp 2.
13
24
ff
ff
lúc này
1 2 1f f n
vi mi
*
n
.
Chinh phc olympic toán| 45
Phương trình hàm trên tập ri rc
CHINH PHC OLYMPIC TOÁN
Thay vào
5
ta được
1
2 2 2
4 2 1 2 2 2
n
f n f n
f f f f n f n

Lúc này
42ff
s các ước s nguyên dương đôi một khác nhau, điều này
không th xy ra nên phi có
11f
.
Thay n bởi 1 vào phương trình
1
thì ta s có được
1 2 3 . 4f f f f a
Mà li có
2 1 1fn
1 2 1
4 2 1 2 2 2 1
12
2
2
nk
f f a f n f n a f n
an
x n k



Vi
*
x
Trường hp 3. Nếu
24
13
ff
ff
.
Lp luận tương t như trên
12
11
21
2
khi n k
fn
an
y khi n k
trong đó
*
y
.
Câu 37. Tìm tt c các hàm s
**
:f
tha mãn
1.
t
f n a f n an t a k
vi
...
t
t
f n f f f n
vi
, at
là s t nhiên tùy ý tha mãn
2 1 1k t a
.
Li gii
Đặt
n
f n n k a
. Ta s chng minh
0
n
a
vi mi s t nhiên dương n bt kì
T gi thiết ta suy ra
11
1
1 1 1
t a k n t a k
an
a f n an t a k f n n
a a a
1
Vi
22n a tk
thì
1
1 2 2 2 2
1
n t a k
n t a k a ak k k f n n k
a
Như vậy vi
2 2 1n a tk f n n k
Khi đó với
2 1 2n a kt k t
thì
1 2 2 2 2 2f n n k a kt k t a tk
lúc đó ta
cũng có
2 1 2 1f f f f f n k f n k
Tiếp tục làm như thế cho đến ln th
1t
lúc đó ta sẽ
1
1 1 2 2
t
f n n t k a tk
Và cui cùng
11
11
t t t
f n f f n f n k n t k

Li có vi
1n t k
thì
1 1 1 1
t
a f n f n a f n n t k a f n
11
t
f n an t a k a f n
nên bất đẳng thc trên cho ta
46 | Tạp chí và tư liệu toán hc
Bồi dưỡng hc sinh gii
TẠP CHÍ VÀ TƯ LIỆU TOÁN HC
1 1 1 97 2 2
nn
a n k a an t a k n t k n a
T
1
ta có
21
1 1 1 1 2 1 1
11
nn
kt
tk n
a a a k n t a k a k t a
aa

Mà t
2
ta có
1 1 1 1 1 0
1
n n n n
t
a a a k t k t a k a a t a a
a
T đó ta sẽ
0
n
a
.
Xét vi
1n t k
. Ta s chng minh phn này ca bài toán bằng phương pháp quy np.
Gi s bài toán đúng đến
mn
. Khi đó
1 2 3
1 2 3 ... 2
t t t
f n t k f n t k f n t k f n n
1
1 1 1 1 1 1
tt
f n f f n t k f n t k a n t k t a k a f n t k
Theo gi thiết quy np
( ( 1) ) ( 2)f n t k n t k
Do đó
1 2 2f n an a t k tk ak k an a t k n t k n k
Vy
f n n k
. Th li thy tha mãn.
Câu 38. Cho hàm s
:f
tha mãn:
2 1 2 1 2 1 2 1 3 1 2
,
2
f n f n f n f n f n
n
f n f n
Tìm n sao cho
2009fn
.
Li gii
Gi s tn ti hàm f thỏa mãn đề bài
3 1 2 fn
là s nguyên dương lẻ
2 1 2 1f n f n
là s nguyên dương lẻ
2 1 2 2f n f n f n
2 1 2 1 2 1f n f n f n
2 1 2 1 1
2 1 2 1 3 1 2
f n f n
f n f n f n
2 1 2 2
2 1 2 2
2 2 1 2 3 1 2 3
f n f n
f n f n
f n f n f n f n



Ta s chng minh
1f n f n
(1)
Vi
1 1 0 2 0n f f f
Gi s
1f n f n
đúng ti
*()0 1 2 <k k N f f f f k 
Nếu k chẵn. Đặt
2*k m m
1 2 1 2 2 2f k f m f m f m f k
Nếu k lẻ. Đặt
21k m m
1 2 2 3 1 3 2 2 2 2 1f k f m f m f m f m f m f k
Như vậy trong mọi trường hp khẳng định (2) đúng
Chinh phc olympic toán| 47
Phương trình hàm trên tập ri rc
CHINH PHC OLYMPIC TOÁN
Do đó
0 3 0 0 0, 1 2; 2 3 1 6f f f f f f
3 2 2 8, 13 12 2 9 3 2 74, 27 3 13 2 224f f f f f f f 
53 9 13 2 668, 107 3 53 2 2006, 108 9 27 2016f f f f f f 
107 2009 108ff
Do
2009fn
0;1;2;...;107n
Câu 39. Tìm tt c các hàm s
:f
tho mãn:
1 1 1
, , ,
3 3 9
f xy f xz f x f yz x y z
.
Li gii
Cho
0x y z
thì
2
2
1 1 1 1 1
0 0 0 0 0 0
3 3 9 3 3
f f f f f



.
Cho
1 x y z
thì
2
2
1 1 1 1 1
1 1 1 1 0 1
3 3 9 3 3
f f f f f



.
Cho
0yz
thì
21
00
39
f f x f
.
Do
1
0
3
f
nên
1
,
3
f x x
.
1
Cho
1yz
, ta có
1 1 1
1
3 3 9
f x f x f x f
.
Do
1
1
3
f
nên
1
,
3
f x x
.
2
T
1
2
ta được
1
,
3
f x x
.
Câu 40. Cho
2nn
và hàm s
:f
sao cho:
1
;,
nn
f x y x f x f f y x y
*
a) Gi s rng
2002 0.f
Tính
2002 .f
b) Tìm hàm s
f
.
Li gii
a) T
*
ta được
Vi
0; ,x f y f f y y
Vi
1; 0 : 0 0 0 0.x y f f f
Vi
1, : 1 1 . 1x y f y f f y
Do đó, chứng minh bng quy nạp ta được
1 , 2f n nf n
T
1
ta có
0 1 1 1 0 1 ; 1 1f f f f f f f t f x f x f
.
Do đó, chứng minh bng quy nạp ta được
1 , 3f n nf n
48 | Tạp chí và tư liệu toán hc
Bồi dưỡng hc sinh gii
TẠP CHÍ VÀ TƯ LIỆU TOÁN HC
T
2 , 3
ta được
1 , 4f n nf n
Đặt
*
p
1 ;p ,q
q
f
và ta được
*
n
n
chia hết
p
nên
1nf
.
Do đó ta được
22
10
1 1 1 1
11
f
f f n f n n f nf f f
f
.
Do đó, từ
4
ta được
2002 1f
hay
2002 0f
(loi).
Vy
2002 2002f
.
b) T
*
ta được
1
0 : , 1
nn
y f x x f x x
Nếu
n
chn:
11
0 : ,
nn
nn
f x f x
x f x f x f x
xx

Nếu
n
l thì t
*
1
ta được
2
nn
f x y f x f y
Suy ra
1
1
0.
n
n
nn
n
n
fx
fx
f x f x f x f x
x
x
Do đó
,f x f x x
T
2
, chng minh bng quy nạp ta được
,,
nn
f px pf x p x
*
:
n n n
p f px f px pf x
Vy
, , 3
nn
f px pf x p x
T
3
ta có
*
,
u
uv
v
ta được
1
1
.. 1
n
n
nn
u u v
f f uv f
v v v




1
11
1
n
n
n n n n
f
u
f f v f f
v v v v



Vy
1
1
. 1 4
n
n
f
uu
f u v f
v v v




Ta có
10f
hay
11f
t
4
suy ra
0,f x x
hay
,f x x x
Th li tha mãn
*
.
Vy
0,f x x
.
Câu 41. Tìm tt c các hàm s
:f
tha mãn
2 3 2 3
,,f x y z f x f y f z x y z
Li gii
Đặt
,,P u v t
là phép thế
x
bi
u
,
y
bi
v
,
z
bi
t
, ta có:
0,0,0 0 0Pf
,1,0Px
2
11f x f x f
Chinh phc olympic toán| 49
Phương trình hàm trên tập ri rc
CHINH PHC OLYMPIC TOÁN
0, ,P y z
10
11
f
f
Vi
11f
, ta chng minh
2
1,f x xf x
Gi s điu phi chng minh đúng đến
,x k k
:
2
1f k kf
Ta chứng minh đúng đến
1 x k k
, thc vy:
2 2 2 2
1 1 1 1 1 1f k f k f kf f k f
Vi
10f
, ta chng minh bng quy np
0f x x
Thế vào gi thiết ban đầu, ta nhận được hai hàm thỏa đề:
0fx
f x x
x
.
Câu 42. Cho hàm s
**
:f
thỏa mãn đồng thời hai điều kin:
a)
,,f ab f a b f a b
vi mi
*
,,a b a b
; trong đó
, , ,a b a b
lần lượt bi
chung nh nhất, ước chung ln nht ca hai s nguyên dương
,ab
;
b)
f p q r f p f q f r
vi mi s nguyên t
,,p q r
.
Tính giá tr ca
2013f
? Kí hiu
*
là tp hp tt c các s nguyên dương.
Li gii
Đặt
2 , 3f a f b
. Khi đó ta có các đẳng thc sau:
7 2 2 3 2 2 3 2f f f f a b
8 2 3 3 2 2 3 2f f f f a b
16 7 7 2 2 7 2 2 2 5 2f f f f a b a a b
16 2 8 2f f f a a b
.
Do đó ta có
2
5 2 2 1a b a ab
.
Mặt khác ta có các đẳng thc sau:
12 2 3 7 2 3 7 3 2f f f f f a b
2
12 2 6 2 2 2 3f f f a f a
Suy ra
2
3 2 3 2a b a
.
T
1 , 2
ta có
2
2
5 2 2 2
... 7 7, 8 8
3
3 2 3
a b a ab a
ff
b
a b a


Ta có 2003 là s nguyên t nên
2013 2003 3 7 2003 3 7 2003 10f f f f f f
3
2025 2003 5 17 2003 5 17 4f f f f f
9 3 3 9 5 2 2 5 2 2 5 5f f f f f f
17 7 7 3 2 7 3 17f f f f
Kết hp vi
4
ta được
2003 2025 22 5ff
50 | Tạp chí và tư liệu toán hc
Bồi dưỡng hc sinh gii
TẠP CHÍ VÀ TƯ LIỆU TOÁN HC
Mt khác
2025 9.9.25 9 9.25 9. 5.5.9 9 5 45f f f f f f f
9 5 3.15 45 3 15 45 3 7 5 3
45.3 7 5 3 2025
f f f f f f
f f f
Do đó
2025 2025f
, kết hp với (5) ta được
2003 2003f
.
Do đó từ đẳng thc
3
ta được
2013 2013f
.
Câu 43. Đặt
: 0,1 0,1Ff
2.n
Tìm giá tr nh nht ca c tha mãn điu kin
11
00
n
f x dx c f x dx

Vi
fF
f
là hàm liên tc.
Li gii
Ta có
1 1 1 1
11
0 0 0 0
0,1
nn
n
f x dx n y f y dy n x f x dx n f x dx x

, vy
cn
.
Vi
0p
, ta chn hàm
p
f x x
, khi đó:
1np
c
np
Do đó:
1
lim
p
np
cn
pn


Vy
cn
, li có
2n
nên giá tr c cn tìm là 2.
Câu 44. Tìm tt c các hàm
: 1,1f 
liên tc, tha mãn:
2
2
1
x
f x f
x



, 1,1x
Li gii
Đặt
2
2
1
x
gx
x
. Bài toán tr thành:
f x f g x
vi mi
11x
.
Ta chng minh bài toán nh: Gi dãy s
1nn
a g a
. Khi đó, với mi giá tr dương
1
01a
, ta có:
lim 1
n
n
a

Chng minh. Vi mi s thc
01a
, ta có:
2
2
1
a
a
a
a g a
.
Li có, vi mi
01a
, ta thy
01ga
.
Vì vy
1
n
n
a
là dãy tăng nghiêm ngặt và có gii hn.
Đặt
lim
n
n
aL

. Ta có:
2
2
1
1
L
L g L L
L
Do
0L
nên
lim 1
n
n
a

Chinh phc olympic toán| 51
Phương trình hàm trên tập ri rc
CHINH PHC OLYMPIC TOÁN
f
hàm liên tc n:
0
lim 1 1
h
f h f

, nói cách khác vi mi
0
luôn tn ti
sao cho vi mi
0 h
:
11ff
Đặt
1x
, t chng minh trên ta có các hàm
2
, , ,x g x g x
đều tiến đến 1.
Vy tn ti s nguyên dương k sao cho
1
k
gx
. Điều này đồng nghĩa:
1
k
f g x f
k
f x f g x
nên
1f x f
, do có th chn giá tr vô cùng bé, ta đưc:
1 0 1f x f f x f
vi mi
01x
.
Chứng minh tương tự, ta được
1f x f
vi mi
10x
f
là hàm liên tc nên
0
lim 0 1 1 0
x
f x f f f f
Vy
f x c
vi c là hng s bt k.
Câu 45. th tn ti hay không mt hàm s
:f
, liên tc trên tha mãn
điu kin: Vi mi s thc
x
, ta có
fx
là s hu t khi và ch khi
1fx
là s vô t.
Li gii
Gi s tn ti hàm s liên tc
:f
thỏa mãn điều kin:
: 1 \ *x f x f x
Xét các hàm s
1 , 1g x f x f x h x f x f x
Khi đó
g
h
nhng hàm s liên tc trên . Ta
g
h
không th đồng thi
hàm hng. Thc vy, gi s
12
,g x C h x C
. Khi đó:
21
2 f x C C f x C
(C là hng s)
Vì thế vi

thì
1f f C
, điều này mâu thun vi
*
.
Gi s
h
không hàm hng, không mt tính tổng quát, khi đó tn ti
1 2 1 2
,,x x x x
sao
cho
12
h x h x
.
Lúc này tn ti s hu t
12
,r h x h x


.
Ta có
12
0h x r h x r
Li
h x r
hàm liên tc vi mi
x
thuc nên phương trình
0h x r
nghim, tc là tn ti
0
x
sao cho
0
h x r
, t đó
00
1f x f x r
.
r
nên
00
1,f x f x
hoặc đồng thi hu t hoặc đồng thi tỉ, điều này mâu
thun vi
*
.
Vy không tn ti hàm s thỏa đề.
52 | Tạp chí và tư liệu toán hc
Bồi dưỡng hc sinh gii
TẠP CHÍ VÀ TƯ LIỆU TOÁN HC
Câu 46. Tìm tt c các hàm s
:f
thỏa mãn điều kin
f x f t f y f z
vi mi s hu t
x y z t
, , ,x y z t
theo th t lp thành cp s cng.
USAJMO 2015
Li gii
Do
, , ,x y z t
theo th t lp thành cp s cng nên
x t y z
, kết hp với phương trình đã
cho ta được
1f x f y z x f y f z
vi mi
x y z
x, y, z theo th t
lp thành cp s cng.
Thay
0x
vào (1) ta được
0f f y z f y f z
vi mi
0 yz
.
Đặt
0g x f x f
ta được
02g y z g y g z y z
Vi s nguyên dương
3n
, ta s tìm cách biu din
gn
theo
1 , 2gg
.
Ta
3 1 2 , 4 3 1 2 2 1g g g g g g g g
, dùng quy np ta chng minh
đưc
2 2 1 3g n g n g
.
Ly
2 yz
là các s nguyên dương ta được:
2 2 1 , 2 2 1 , 2 2 1g y g y g g z g z g g y z g y z g
,
Thay vào
2
ta được
2 2 1 2 2 1 2 2 1 2 2 1 .g y z g g y g g z g g g
*
Kết hp vi
3
ta được
1g n ng
Ta biu din
g nx
theo
,2g x g x
.
Bng quy np ta d dàng chng minh
2 2 4g nx g x n g x
.
Ly
2 nm
là các s nguyên dương, thực hiện các thao tác như
*
ta được
2 2 2 2 2 2 2 2g x m n g x g x n g x g x m g x g x g x
Kết hp vi
4
ta được
5g nx ng x
Vy vi
,mn
nguyên dương:
11
m m m
g m ng g g g x xg
n n n
, vi
x
Do
f x f t f y f z g x g t g y g z x y z t
lp thành CSC
Vi
0x
bt k, ta xét CSC
0x y z
ta được
0 1 0 1 0 1 1 1g x g t g g z g x tg g zg g x g zg tg xg
Vy
1 , 0 1 0g x xg x f x f x f f ax b
vi
,,x a b
.
Th li ta thy tha mãn.
Chinh phc olympic toán| 53
Phương trình hàm trên tập ri rc
CHINH PHC OLYMPIC TOÁN
Câu 47. Gi s
,rs
hai s cho trước. Tìm tt c các hàm s
:f
tha mãn
điu kin
,,f x f y f x r y s x y
Romania 2006
Li gii
Thay
0x
vào phương trình ban đầu ta được
,1f f y f r y s y
Thay y bi
fy
thu được:
,,f x f f y f x r f y s x y
,,f x f r y s f x r f y s x y
,,f x r y f r r s f x r f y s x y
,,f x y f r r s f x f y s x y
Đặt
a f r r s
thay vào phương trình trên ta được:
, , 2f x y a f x f y s x y
Thay
0y
vào phương trình (2) ta được:
0 , 3f x a f x f s x
.
T
2 , 3
ta được:
0 , ,f x y f s f x f y s x y
0 , ,f x y f f x f y x y
0 0 0 , ,f x y f f x f f y f x y
Đặt
0,f x f g x x
và thay vào phương trình trên ta được
, , 4g x y g x g y x y
T
4,
theo kết qu v phương trình hàm Cauchy ta được
,,g x bx x b
T cách xác định hàm s
g
ta được
05f x bx f bx c
T
5
thay lại vào phương trình ban đầu ta được:
,,b x by c c b x r y s c x y
2
,,bx b y bc c bx y br s c x y
2
1
1
b r s
b
b
bc c br s c
c r s




Vy bài toán có hai nghim
,,f x x r s f x x r s x
.
54 | Tạp chí và tư liệu toán hc
Bồi dưỡng hc sinh gii
TẠP CHÍ VÀ TƯ LIỆU TOÁN HC
Câu 48. Tìm tt c các hàm s
:f
sao cho vi tt c các s nguyên
,,a b c
tha
mãn
0a b c
, đẳng thức sau là đúng:
2 2 2
2 2 2f a f b f c f a f b f b f c f c f a
IMO 2012
Li gii
Li gii 1. Tôn Ngc Minh Quân
Gi s hàm
:f
thỏa mãn điều kiện đề bài.
Cho
0a b c
, ta được
00f
.
Cho
, , 0a n b n c n
ta được
f n f n
. Đặt
1f t t
.
Cho
2, 1, 1a b c
ta có
20f
hoc
24ft
.
Trường hp 1.
20f
3ft
Ta có
22
2
4 2 (2) 2 2 4 2 2 4 2 2 2 4 0f f f f f f f f f f
Gi s
2 0, 2 1 1f i f i t i k
2 2 2
2 2 2 2 2 20 0f k f k f f k
Ta có
2 2 2
2 3 2 3 2 3 2 3 2 3 3f k f k f f f k f k f t
Vy
2 0, 2 1 ,f i f i t i N
2 0, 2 1 ,f i f i t i
Trường hp 2.
2 4 , 0f t t t
Ta có
2 2 2
3 2 1 2 1 2 2 1 3 2 2 3f f f f f f f f f
Suy ra
3ft
hoc
39ft
a)
39ft
,
24ft
,
1ft
. Ta chng minh
2*
,f n n t n
Tht vy mệnh đề đúng với
1, 2,3n
. Gi s mệnh đề đúng đến
3n
Ta có
2 2 2
1 1 2 1 2 1 1 2 1f n f n f f f n f f n f n f n
2
2
2 2 2
1 2 1 1 1 0f n t n f n t n
2
11f n t n
hoc
2
11f n t n
Gi s
2
1 1 1f n t n f n
Ta có
2 2 2 2
1 2 1 2 2 1 2 2 1 2 1f n f f n f f n f f n f n
2
2 2 2 1 1f f f n f n
22
2 2 2
16 8 .2 1 16 16 1t t n t t t n
.
Đó là điu vô lý (vì
3n
).
Vy
2 * 2
,,f n n t n f n n t n
b)
3 , 0 0, 1 , 2 4f t f f t f t
Chinh phc olympic toán| 55
Phương trình hàm trên tập ri rc
CHINH PHC OLYMPIC TOÁN
222
4 2 2 2 2 2 2 2 ( 4 ) 2 2 4f f f f f f f f f
40f
hoc
4 16ft
Gi s
4 16ft
. Ta có
2 2 2
4 3 1 2 1 4 2 3 4 2 1 3f f f f f f f f f
2 2 2 2 2
256 2 32 32 2t t t t t
2
192 0t
(vô lý).
Vy
40f
.
Ta có
2 2 2
5 4 1 2 1 5f f f f f
(5) (1)f f t
,
222
6 4 2 2 2 6f f f f f
6 2 4f f t
,
2 2 2
7 4 3 2 3 7f f f f f
(7) (3)f f t
,
222
8 4 4 0f f f
80f
Bằng phương pháp quy nạp toán hc ta chứng minh được
41f i t
iN
;
43f i t
i
40fi
iN
;
4 2 4f i t
i
Tht vy gi s
4 0, 4 1 , 4 2 4 , 4 3f k f k t f k t f k t k N
Ta có
2 2 2
4 1 4 1 2 1 4 2 4 4 1 2 1 4 1f k f k f f f k f k f k f f k
4 1 1f k f
2 2 2
4 2 4 2 2 2 4 2 4 4 2 2 2 4 2f k f k f f f k f k f k f f k
4 2 2 4f k f t
2 2 2
4 3 4 3 2 3 4 2 4 4 3 2 3 4 3f k f k f f f k f k f k f f k
4 3 3f k f t
2 2 2
4 4 4 4 2 4 4 2 4 4 4 2 4 4 4f k f k f f f k f k f k f f k
4 4 4 0f k f
.
Suy ra
4 0, 4 1 , 4 2 4 , 4 3 , 0f i f i t f i t f i t t t i
Ngược li, gi s hàm
:f
tha mãn
2 0, 2 1f i f i t t
vi mi
i
Gi s
, , , 0a b c a b c
. Suy ra trong 3 s a,b,c có ít nht mt s chn
+) Nếu a,b,c cùng chn thì
0f a f b f c
2 2 2
2 2 2f a f b f c f a f b f b f c f c f a
+) Nếu
a
chn và b,c l thì
0fa
,
f b f c t
2 2 2
2
2f a f b f c t
2
22f a f b f a f c f b f c t
22
2
( ) 2 2 2f a f b f c f a f b f b f c f c f a
56 | Tạp chí và tư liệu toán hc
Bồi dưỡng hc sinh gii
TẠP CHÍ VÀ TƯ LIỆU TOÁN HC
Tương tự nếu
b
chn a,c l hoc
c
chn a,b l thì ta cũng có:
2 2 2
2 2 2f a f b f c f a f b f b f c f c f a
Vy hàm
:f
sao cho
20fi
,
21f i t t
vi mi
i
thỏa mãn điều
kiện đề bài.
Xét hàm s
:f
tha mãn
2
,0f n n t t t n
Gi s
,,a b c
tha mãn
0a b c
Ta có
2 2 2
,,f a a t f b b t f c c t
Suy ra
2 2 2
4 4 4 2
f a f b f c a b c t
2 2 2
0 2 2 2a b c a b c ab bc ca
4 4 4 2 2 2 2 2 2 2 2 2 2 2 2
2 2 2 4 4 4 8a b c a b b c a c a b b c a c abc a b c
4 4 4 2 2 2 2 2 2
2 2 2a b c a b b c a c
2 2 2
2 2 2 2 2 2 2 2 2
2 2 2f a f b f c a b t b c t a c t
2 2 2f a f b f b f c f c f a
Vy hàm
:f
sao cho
2
,0f n n t t t n
thỏa mãn đề bài.
Xét hàm
:f
tha mãn
4 1 , 4 2 4 , 4 3 , 4 0 , 0f i t f i t f i t f i t t i
Gi s
,,a b c
sao cho
0a b c
+ Nếu
4 0 mod 4a i i b c
+ Nếu b,c đều chia hết cho 4 thì
0f a f b f c
2 2 2
2 2 2 0f a f b f c f a f b f b f c f c f a
+ Nếu
2 mod 4b
2 mod 4c
thì
0, 4 , 4f a f b t f c t
2 2 2
2
32f a f b f c t
2
2 2 2 32f a f b f b f c f c f a t
2 2 2
2 2 2f a f b f c f a f b f b f c f c f a
+ Nếu
1 mod 4b
3 mod 4c
thì
,f b t f c t
2 2 2
2
2f a f b f c t
2
2 2 2 2f a f b f b f c f c f a t
2 2 2
2 2 2f a f b f c f a f b f b f c f c f a
+ Nếu
1 mod 4a
,
0 mod 4b
3 mod 4c
, tương tự như trên ta cũng có:
2 2 2
2 2 2f a f b f c f a f b f b f c f c f a
+ Nếu
1( mod 4)a
,
3 mod 4b
0 mod 4c
, tương tự như trên ta cũng có:
Chinh phc olympic toán| 57
Phương trình hàm trên tập ri rc
CHINH PHC OLYMPIC TOÁN
2 2 2
2 2 2f a f b f c f a f b f b f c f c f a
+ Nếu
1 mod 4a
,
2 mod 4b
1 mod 4c
, 4 ,f a t f b t f c t
2 2 2
2
( ) ( ) ( ) 18f a f b f c t
2 2 2 2
2 2 2 8 8 2 18f a f b f b f c f c f a t t t t
2 2 2
2 2 2f a f b f c f a f b f b f c f c f a
+ Nếu
1 mod 4a
,
1 mod 4b
2 mod 4c
, tương tự như trên ta cũng có:
2 2 2
2 2 2f a f b f c f a f b f b f c f c f a
+ Nếu
2 mod 4a
,
0 mod 4b
2 mod 4c
hoc
2 mod 4a
,
1 mod 4b
1 mod 4c
; hoc
3 mod 4a
,
0 mod 4b
1 mod 4c
hoc
3 mod 4a
,
1 mod 4b
0 mod 4c
, tương tự như trên ta cũng có:
2 2 2
2 2 2f a f b f c f a f b f b f c f c f a
+ Nếu
3 mod 4a
,
3 mod 4b
,
2 mod 4c
thì
,4f a f b t f c t
2 2 2
2
18f a f b f c t
;
2
2 2 2 18f a f b f b f c f c f a t
2 2 2
2 2 ( ) 2f a f b f c f a f b f b f c f c f a
Vy tt c các hàm
:f
thỏa mãn đề bài là:
:f
:
2 0, 2 1f i f i t t i
:f
:
2
,0f n n t t t n
:f
:
4 0, 4 1 , 4 2 4 , 4 3 , 0f i f i t f i t f i t t t i
.
Cách 2.
Thay
0a b c
vào phương trình ban đầu ta được:
2 2 2 2 2
0 0 0 2 0 0 2 0 0 2 0 0 3 0 6 0 0 0f f f f f f f f f f f f
Thay
,0b a c
vào phương trình ban đầu ta được:
2 2 2
0 2 2 0 2 0f a f a f f a f a f a f f f a
2 2 2 2
2 2 0f a f a f a f a f a f a f a f a f a f a
Ta có th viết lại phương trình ban đầu dưới dng:
2
2
20f c f c f a f b f a f b
22
2 4 4
2
f a f b f a f b f a f b
f c f c f a b
2f a b f a f b f a f b
Nếu
0fb
:
modf a b f a f a b
Trường hp 1.
1 0 0f f x x
.
58 | Tạp chí và tư liệu toán hc
Bồi dưỡng hc sinh gii
TẠP CHÍ VÀ TƯ LIỆU TOÁN HC
Trường hp 2.
1 0,f
ta có
2 1 1 2 1 1 2 0f f f f f f
or
2 4 1ff
Ta xét hai trường hp nh:
Trường hp 2.1:
1 0, 2 0 mod 2 (1)f f f x f x f x f
nếu
x
l
0fx
nếu
x
chn.
Trường hp 2.2:
1 0, 2 4 1 3 2 1 2 2 1f f f f f f f f
3 5 1 4 1 3 1 9 1f f f f f f
Nếu
1 0, 2 4 1 , 3 1f f f f f
:
4 1 3 2 1 3f f f f f
4 2 2 2 2 2f f f f f
41ff
hoc 0 và
4 16 1ff
hoc 0
4 0 mod 4 .f f x f x
Nếu
1 0, 2 4 1 , 3 9 1f f f f f
:
4 1 3 1 3 2 1 3 16 (1)f f f f f f f
hoc 4
1f
4 2 2 2 2 16 1f f f f f
hoc
0. 4 16 1ff
Nếu
4x
, khi đó
2
1f x f x
Dùng quy np, ta chng minh:
2
1f x f x x
Nếu .
xm
.
, khi đó
2
1f x f x x
, đúng với mt s giá tr
m
.
Gi s điu phi chng minh đúng với
mk
:
2
1 1 2 1 1 1f k f k f f k f f k
hoc
2
11fk
2
1 1 2 2 1 2 1 1f k f k f f k f f k
hoc
2
13fk
2
1 1 1f k f k
.
Vậy điều phi chng minh đúng với
1mk
.
Vì nó vẫn đúng với
4m
, theo quy np toán hc ta có th kết lun
2
1f x f x x
.
Câu 49. Tìm tất cả các hàm
,: fg
có đạo hàm trên
thỏa mãn
'
gx
fx
x

;
'
fx
gx
x

x

Li gii
Ta có
. ' ' 'x f x g x x f x g x f x g x


0, 0
g x f x
x f x g x x
xx



0x f x g x a x


f x g x
a
x
0x
1
Chinh phc olympic toán| 59
Phương trình hàm trên tập ri rc
CHINH PHC OLYMPIC TOÁN
Tương tự ta
' 0, 0
f x g x
x
x



,0f x g x bx x
0x
2
Từ
1 , 2
1
2
a
f x bx
x




;
1
, 0, ,
2
a
g x bx x a b
x



Câu 50. Tìm tt c các hàm
*
:f
có đạo hàm trên
*
tha mãn
*
,f xy f x f y x y
1
Li gii
Lấy đạo hàm hai vế
1
lần lượt theo biến
,xy
ta có
*
' ' ,yf xy f x x y
*
' ' ,xf xy f y x y
. ' . 'x f x y f y
*
,xy

. '( )x f x a
*
x

.lnf x a x b
*
xR

Th li
0b
.
Vy
.lnf x a x
*
x

.
Câu 51. Tìm tt c các hàm
:f
tha mãn
1f f n n b n
trong đó
b
là s nguyên dương chẵn.
Li gii
Gi s
f
thỏa mãn đề bài. D thy
f
đơn ánh
Trong
1
thay
n
bi
fn
ta được
f f f n f n b



Suy ra
f n b f n b n
Vy nếu
,0 1m qb r r b
thì
1f m f qb r f q b r b f r qb
Bây gi ta ch cần xác định hàm
f
trên tp
0,1,2,..., 1Ab
.
Xét
xA
, đặt
f x y
thì
f y x b
Gi s
y q b r
thì
f y f r qb
suy ra
2x b f r qb qb x b b
nên
0q
hoc
1q
.
Do đó nếu
xA
thì
i)
,,f x r b r A f r x
ii)
,,f x r r A f r x b
Vy hàm
f
được xác định như sau
ii
ii
f qb r qb f r
f a b
f b a b

vi
ii
a b A
.
60 | Tạp chí và tư liệu toán hc
Bồi dưỡng hc sinh gii
TẠP CHÍ VÀ TƯ LIỆU TOÁN HC
Câu 52. Tìm tt c các hàm
:f

tha mãn:
i)
,f xf y yf x x y
1
ii)
lim 0
x
fx

IMO 1983
Li gii
Thế
1x
ta được
1f f y yf
Nếu
,f a f b
thì
11af f f a f f b bf a b
1f
tiến ti s nguyên
dương. Vì vậy hàm
f
đơn ánh.
Thế
xy
ta được
()f xf x xf x
Vy
xf x
là điểm bất động ca hàm
f
.
Khi đó, thay
1y
thu được
1 1 1f xf f x f
x
khác 0 nên 1 điểm bt
động ca hàm f. Ta chứng minh 1 là điểm bất động duy nht ca hàm
f
.
Gi s tn tại a,b là điểm bất động ca
f
, ta có
f a a
f b b
.
Khi đó, thế
,x a y b
vào
i
ta được
f ab f af b bf a ab
Suy ra
ab
cũng là điểm bất động ca
f
.
Thế
1
,x y a
a

thu được
1 1 1 1 1
11f f a f f a af f
a a a a a
Vy
1
a
là điểm bất động ca
f
.
Nếu
f a a
vi
1a
, khi đó
n
fa
là điểm bất động ca
f
, mâu thun
ii
Nếu
f a a
vi
01a
, khi đó
11
nn
f
aa



là điểm bất động ca
f
, mâu thun
ii
T các kết qu trên ta thu được điểm bất động duy nht là 1, suy ra
1
1xf x f a
x
.
Câu 53. Chng minh rng tn ti song ánh
:f

sao cho
3 4 ,f mn m n f m f n f m f n m n
IMO Shortlist 1996
Li gii
Xét hàm
: 3 1 4 1g

tha mãn:
1
41
3
x
g x f




thì ta có
g
là song ánh và
3 1 3 1 3 1 3 1 ,g m n g m g n m n
.
Tht vy
3 1 3 1 3 3 1 4 3 1g m n g mn m n f mn m n
4 4 1 4 1 4 1 3 1 3 1f m f n f m f n f m f n g m g n
Vy vi mi
, 3 1xy
, ta có
.g xy g x g y
Chinh phc olympic toán| 61
Phương trình hàm trên tập ri rc
CHINH PHC OLYMPIC TOÁN
Như thế, ta ch cn ch ra mt song ánh
g
đủ, khi đó
3 1 1
4
gx
fx

tha mãn
đề bài.
Xét
12
,PP
là tp các s nguyên t dng
3 1,3 2kk
ơng ứng và
12
,QQ
là tp các s
nguyên t dng
4 1, 4 3kk
tương ứng, ta xét song ánh
1 2 1 2
:h P P Q Q
Sao cho
1 1 2 2
,h P Q h P Q
và xác định
g
như sau:
11g
nguyên t thì
i
g n h p
Rõ ràng song ánh
g
như vậy thỏa mãn đề bài.
Câu 54. Tìm tt c các hàm
:f
tha:
3 2 6 ,f f f n f f n f n n n
Li gii
Đặt
...
k
k
a f n f f f n
(lp k ln).
Ta lặp được dãy
1 2 3
3 2 6
k k k k
a a a a
vi mi n
Xét tp
0 1 2
, , ,...S a a a
đây tp các s nguyên dương nên sẽ tn ti mt phn t
giá tr nh nhất. Đặt
j
là s sao cho
j
a
là giá tr nh nht trong tp
S
.
Ta có bất đẳng thc
1 2 3
3 2 6 6
k k k k j
a a a a a
Đẳng thc xy ra khi
12k k k j
a a a a

Lại có khi đặt
3kj
thì đẳng thc xy ra, vy ta có
1 2 3j j j j
a a a a
.
Tnh tiến giá tr ta thu được
kj
aa
vi mi
kj
.
Thc hiện tương tự cho tnh tiến lùi, lưu ý rằng
3 2 1
32
6
k k k
k
a a a
a

, ta thu được
kj
aa
vi mi
kj
. Vy
kj
aa
vi mi
0k
.
T chng minh trên dẫn đến
10
aa
hay
f n n
, n
.
Câu 55. Tìm tt c các hàm s
: 0; 0;f  
thỏa mãn điều kin:
, 0; 1f f x yf yf x x y 
Li gii
Vi mi
0;t 
, ta chn tùy ý mt
0
x
c định và
0
u f x
,
0
v tf x
thì
fu
t
fv
Ta thay trong
1
y
bi
1
fy
ta có
1
fx
f f x f
f y f y




62 | Tạp chí và tư liệu toán hc
Bồi dưỡng hc sinh gii
TẠP CHÍ VÀ TƯ LIỆU TOÁN HC
Hay
. 2
fx
f f y f f x
fy




Trong
1
, thay
y
bi
1
fx
ta được
1
1 0; 3f f x f x
fx

T
2 , 3
suy ra
1 .
fy
fx
ff
f y f x




Do đó
1.
f u f v
ff
f v f u




Do vy,
1
1 . 0;f t f t
t

.
Th li, hàm cn tìm là
0;
a
f x x
x
đó
0a
là hng s.
u 56. Chng minh rng tn ti duy nht mt hàm s
f
xác định trên tp các s thc
dương, nhận giá tr thực dương và thỏa mãn
6.f f x x f x
Putnam 1988
Li gii
Vi mi s thực dương
0
x
c định, ta xây dng dãy
1
n
n
f
như sau:
1 0 2 0 1 0
, , .
nn
f x f f x f f f x
Khi đó, từ đẳng thc gi thiết ta suy ra dãy
1
n
n
f
thỏa mãn phương trình truy hồi
21
6,
n n n
f f f


Hay
21
6 0.
n n n
f f f

Đến đây, giải phương trình đặc trưng của dãy
1
n
n
f
, ta được hai nghim là 2 và
3
.
Do đó,
2 3 ,
n
n
n
f a b
trong đó, các hằng s a, b tìm được ph thuc vào
12
,ff
.
Tuy nhiên, nếu
0b
, thì tn ti
n
đủ ln sao cho
0
n
f
(ta có th thấy được d dàng bng
cách chn
n
chẵn đủ ln nếu
0b
, và chn
n
l đủ ln nếu
0b
).
Do vy,
0b
. Thành ra
2
n
n
fa
Suy ra
32
00
.2 , .2f f x a f x a
, thay hai giá tr này vào đẳng thc
0 0 0
6f f x x f x
Ta được
0
2ax
. Dẫn đến
00
2f x x
Và vì điều này đúng với mi
0
x
dương nên
2 , 0f x x x
.
Chinh phc olympic toán| 63
Phương trình hàm trên tập ri rc
CHINH PHC OLYMPIC TOÁN
Câu 57. Hàm s
:f
thỏa mãn đồng thời các điều kin sau:
: , 1
: 2 2 , 2
: 0 1 3
i f f n n n
ii f f n n n
iii f
Tìm giá tr
1995 , 2007ff
Olympic Ukraine 1995
Li gii
Cũng nhận xét và lý luận như các ví dụ trước, ta đưa đến
fn
phi có dng:
f n an b
Khi đó điều kin
i
tr thành:
2
,a n ab b n n
Đồng nht các h số, ta được
2
11
1
00
0
aa
a
bb
ab b





Vi
1
0
a
b
ta được
f n n
. Trường hp này loi vì không tha mãn
ii
Vi
1
0
a
b

ta được
f n n b
T điu kin
iii
cho
0n
ta được
1b
Vy
14f n n
Hin nhiên hàm s này tha mãn điu kin bài toán.
Ta phi chng minh
1f n n
là hàm duy nht thỏa mãn điều kin bài toán
Tht vy gi s tn ti hàm
gn
khác
fn
cũng thỏa mãn điều kin bài toán.
T
iii
suy ra
0 0 1fg
T
iii
suy ra
1 1 0fg
S dụng điều kin
,i ii
ta nhận được
22g g n g g n
n
Dno đó
22g g g n g g g n
n
Hay
22g n g n
n
Gi s
0
n
là s t nhiên bé nht làm cho
00
5f n g n
Do
fn
cũng thỏa mãn
4
nên ta có
0 0 0 0
00
2 2 2 2
22
g n g n f n f n
g n f n
Mâu thun với điều kin n0 là s t nhiên bé nht tha mãn
5
Vy
f n g n
,
n
Chứng minh tương tự ta cũng được
f n g n
vi mi n nguyên âm.
64 | Tạp chí và tư liệu toán hc
Bồi dưỡng hc sinh gii
TẠP CHÍ VÀ TƯ LIỆU TOÁN HC
Vy
1f n n
là nghim duy nht.
T đó tính được
1995 , 2007ff
.
Câu 58. Tìm
: 0,1f
tha mãn:
, , 0,1f xyz xf x yf y zf z x y z
Li gii
Chn
x y z
ta được
3
3f x xf x
Thay
,,x y z
bi
2
x
ta được
6 2 2
3f x x f x
Mt khác
6 2 3 2 2 3 3
..f x f x x x xf x x f x x f x
Hay
2 2 2 2 4
33x f x xf x x f x x f x
2 2 4
23x f x xf x x f x
2
3
31
,
2
f x f
x
x x
Thay
x
bi
3
x
ta được
9
63
31
,
2
x
f x f x x
9
22
31
3 3 ,
2
x
x f x xf x x
39
2
3 1 3 1
3 3 , 0, 0
22
xx
x f x xf x x f x x

Vy
0fx
vi mi
0;1x
.
Câu 59. Tìm tt c các hàm
f
xác định trên và thỏa mãn đồng thời các điều kin sau:
2 2 3 ,
11
f n f k n f k n f n f k k n
f
Li gii
Cho
0kn
22
2 0 2 0 3 0 0 0 0 2f f f f f
Nếu
00f
chn
0n
ta được:
20fk
do đó
0fk
vi mi k
Chn
1k
ta được
10f
mâu thun vi gi thiết.
Vy
02f 
Chn
1n
ta được phương trình
2 1 1 2 1 3 1 ,f f k f k f f k k
2 1 2 1 3 ,f k f k f k k
Đặt
k
x f k
ta có phương trình sai phân
11
2 3 2 0
k k k
x x x

Phương trình đặc trưng là
2
1
2 3 2 0 2
2
Vy
12
1
2
2
n
n
f n c c



.
Chinh phc olympic toán| 65
Phương trình hàm trên tập ri rc
CHINH PHC OLYMPIC TOÁN
Ta tìm
12
,cc
t điu kin
0 2, 1 1ff
.
D tìm được
12
0, 2cc
Vy
1
2
2
n
fn



.
Câu 60. Tìm tt c các hàm s
**
:f
thỏa mãn đồng thời hai điều kin sau:
**
*
2 , ,
1 ,
f f n n k n k
f n f n n
Li gii
Gi s có hàm
f
tho mãn các điều kin
1 ,2
.
Ta chúng minh hàm
f
là một đơn ánh.
Tht vy, vi mi
*
,mn
mn
, nếu có
f m f n
thì do
1
suy ra
22f f m f f n m k n k m n
(vô lý)
Suy ra
f
là đơn ánh. Do
2
1f n f n
vi mi
*
n
suy ra
11f n f n
vi mi
*
n
1 1 1f f n f f n f f n
vi mi
*
n
11f f n f f n
vi mi
*
n
1 2 1 2n k n k
vi mi
*
n
11f f n f f n
vi mi
*
n
11f n f n
vi mi
*
n
( do
f
là đơn ánh)
11f n f n
vi mi
*
2 ; nn
Truy hồi ta được
11f n n f
vi mi
*
n
1 1 2 2 1f f n f n f n f
vi mi
*
n
2 2 2 1n k n f
vi mi
*
n
11fk
vi mi
*
n
Suy ra
11f n n k
vi mi
*
n
f n n k
vi mi
*
n
Th li
f n n k
vi mi
*
n
thỏa mãn các điều kin
1,2
Vy các hàm
f
cn tìm thỏa mãn đề bài là
f n n k
vi
*
n
.
66 | Tạp chí và tư liệu toán hc
Bồi dưỡng hc sinh gii
TẠP CHÍ VÀ TƯ LIỆU TOÁN HC
Câu 61. Tìm tt c các hàm s
:f
thỏa mãn đồng thời hai điều kin sau:
2013 2016
4,
f
f f n n n
Li gii
Gi s có hàm
f
tho mãn các điều kin
1 ,2
.
T điu kin
1
ta d dàng chứng minh được
f
là một đơn ánh.
T
1
ta suy ra
4f f f n f n
vi mi
n
44f n f n
vi mi
n
(3)
Vi
4n k r
vi
k
;
0, 1, 2, 3r
T
3
ta suy ra
4 4 4f k r k f r
vi mi
k
Tính
1f
. Do
2013 4.503 1
nên
2013 2012 1 2016 1 4f f f
Tính
0f
. Ta có
0 4 1 0 1f f f f
(do
f
là đơn ánh)
Tính
2f
3f
. Gi s
24f m r
vi
m
0,1,2, 3r
Do
1
mà ta có
2 6 2 4ff
2 4 4 6f f f m r m f r
;
0fr
0m
hoc
1m
+ Vi
0m
, thì
6fr
22f f f r f r
+ Vi
1m
, thì
2fr
24fr
Trường hp 1. Xét
0m
6
2 0, 1, 2,3
fr
f r r

+ Vi
0r
thì
06
2 0
f
f
vô lý do
01f
+ Vi
1r
thì
16
2 1
f
f
vô lý do
14f
+ Vi
2r
thì
26
2 2
f
f
vô lý
+ Vi
2r
thì
26
3 2
f
f
vô lý
Vy khi
0m
ta có
0 1; 1 2; 2 3; 3 6f f f f
Trường hp 2. Xét
1m
2
2 4 0,1,2, 3
fr
f r r
+ Vi
0r
thì
02f
vô lý do
01f
+ Vi
1r
thì
12f
vô lý do
14f
Chinh phc olympic toán| 67
Phương trình hàm trên tập ri rc
CHINH PHC OLYMPIC TOÁN
+ Vi
2r
thì
22
2 6
f
f
vô lý
+ Vi
2r
thì
27
3 2
f
f
vô lý
Vy khi m =1 ta có
0 1; 1 4; 2 7; 3 2f f f f
Suy ra
1 , n 0 mod 4
3 , n 1 mod 4
5 , n 2 mod 4
1 , n 3 mod 4
n
n
fn
n
n




Th li:
fn
thỏa mãn đồng thời các điều kin (1) và (2) nên
fn
là các hàm cn tìm.
Câu 62. Tìm tt c các hàm s
*
:f
thỏa mãn điều kin sau:
1 1 . 3 , 1f n f n f n f n n
Li gii
Gi s có hàm
f
tho mãn các điều kin
1
.
Ta lp dãy
n
a
vi mi
n
ta đặt
n
a f n
khi đó
1
tr thành
2 1 3
,
n n n n
a a a a n
2
Thay
2nn
trong phương trình
2
ta được
2 4 3 5
,
n n n n
a a a a n
3
Tr tng vế ca
2 ,3
ta được
4 3 5 1
,
n n n n n
a a a a a n
.
Thay
n
lần lượt bi
0,1, 2, 3,......
ta có
4 0 3 5 1
5 1 4 6 2
6 2 5 7 3
. . . . . . . . . . . . . . . .
a a a a a
a a a a a
a a a a a
Suy ra
4 0 3 4 5 2 4
.....
n n n
a a a a a a a a

,.n
4
Ta chng minh rng
4nn
aa
, n
4f n f n
,.n
5
Hay
f
là hàm tun hoàn vi chu k 4.
Tht vy. Gi s tn ti s
*
0
n
00
40
;
nn
a a n n

Do
*
n
a
vi
n
nên
00
4
1
nn
aa

suy ra
0 0 0 0
4 0 3 4 5 2 4 3 4 5 2
..... .....
n n n n
a a a a a a a a a a a a
Do
2 1 3n n n n
a a a a

vi
n
, nên vi 4 s liên tiếp
0 0 0 0
1 1 2
, , ,
n n n n
a a a a
phi ít nht
mt s lớn hơn 1.
Do đó khi
n
thì
0
3 4 5 2
.....
n
a a a a

suy ra
00
4nn
aa

(vô lý)
68 | Tạp chí và tư liệu toán hc
Bồi dưỡng hc sinh gii
TẠP CHÍ VÀ TƯ LIỆU TOÁN HC
Suy ra
4 0 4
00
nn
a a a a
vi
n
4f n f n
vi
0, 1, 2, 3r
trong đó
f
là hàm tun hoàn chu k 4.
Hàm
f
được xác định khi ta tính đưc
0 1 2 3
0 ; 1 ; 2 ; 3f a f a f a f a
bi t
2
ta có
0 2 3 4 0 2 1 3
1 3 4 2 0 2 0 2 1 3
a a a a a a a a
a a a a a a a a a a



Mt khác
0 2 0 2 0 2
1 3 1 3 1 3
1 1 1
1 1 1
a a a a a a
a a a a a a
Suy ra
0 2 1 3
1 1 1 1 2a a a a
6
Do đó có các khả năng xảy ra.
Kh năng 1.
02
1 1 0 0
3 3 2 2
13
1 1 0
2 v a 3 1 v a 5
3 v a 2 1 v a 5
1 1 2
aa
aa
aa
aa



Suy ra
0 1 2 3
0 ; 1 ; 2 ; 3 ; ; ; 1;2;5;3 , 1;3; 5; 2 , 5; 2;1; 3 , 5; 3;1; 2f f f f a a a a
Ta tìm được 4 hàm cn tìm vi
fn
vi
fn
xác định bi
0 , 0 mod 4
1 , 1 mod 4
2 , 2 mod 4
3 , 3 mod 4
fn
fn
fn
fn
fn
7
đều thỏa mà điều kin 1.
Kh năng 2.
02
13
1 1 1
1 1 0
aa
aa
0 1 2 3 0 1 2 3
2 ; ; ; 2; 2; 2; 2a a a a a a a a
Suy ra
( ) 2fn
vi mi
n
Kh năng 3.
02
13
1 1 2
1 1 0
aa
aa
Lp luận tương tự như khả năng 1 ta được
0 1 2 3
0 ; 1 ; 2 ; 3 ; ; ; 2;1;3;5 , 3;1; 2; 5 , 2; 5; 3;1 , 3; 5;2; 1f f f f a a a a
Ta được 4 hàm
fn
cn tìm và c 4 hàm đều thỏa mãn điều kin
1.
Vi
fn
xác định bi
7.
Vy có 9 hàm
f
tha mãn là nghim của phương trình
1.
Câu 63. Tìm tt c các hàm
:f

tha mãn:
f x f y f x y f y
Li gii
Ta để ý rng:
f a f b f c fa f b c f c f a b c f b f c
Chinh phc olympic toán| 69
Phương trình hàm trên tập ri rc
CHINH PHC OLYMPIC TOÁN
Thế y bi
y f z
ta được:
2f x f y f z f x f y z f z f x y z f y z f z
f x y f z f y f z f x y z f z f y z f z
Thu được:
2f x y z f x y z f z
vi mi
, , 0x y z
Vi giá tr ca
y
, ta chn
x
sao cho
,x y f y
, t đó ta có kết qu
2f f y f y
Thế
yx
vào phương trình ban đầu
2f x f x f x f x
D nhn thy
f x x
, vì vy:
2 , 2f x f f x f x f x f f x f x
Theo phương trình hàm Cauchy, ta được
2f x x
vi mi
x
.
Câu 64. Tìm s nguyên dương
m
nh nht sao cho tn ti hàm s
*
: \ 1;0;1f 
thỏa mãn đồng thời các điều kin sau
i)
2015 , 1 2016 ;f m f f m f
ii)
1
, 1, 2,....
1
fn
f n m n
fn
Li gii
Ta có
*
1
2 4 ,f n m f n m f n n
fn
Vi
1m
, ta có
*
4 4 , ,f n f n f n k f n k n
Ta có
*
1
1
2 ; 1 ,
1
fn
f n f n n
f n f n
1
1 2015 4.503 3 3
1
f f f f
f
: vô lý.
Vi
2m
, ta có
*
8 8 , ,f n f n f n k f n n k
*
1
1
4 ; 2 ,
1
fn
f n f n n
f n f n
Ta có
1
2 2015 251.8 7 7 ;
3
f f f f
f
2
1
3 2016 251.8 8 8
4
21
2 4 2 1
21
f f f f
f
f
f f f
f
Điu mâu thun trên dẫn đến
3.m
Vi
3,m
ta xây dựng được vô s hàm
f
tha yêu cầu bài toán như sau
70 | Tạp chí và tư liệu toán hc
Bồi dưỡng hc sinh gii
TẠP CHÍ VÀ TƯ LIỆU TOÁN HC
Cho
\ 1;0;1a
, đặt
11
1 ; 2 ; 3 ;
1
a
f a f f
aa
1
3 , 1
1
fn
f n n
fn
Khi đó, chứng minh quy np thì hàm s xác định trên
*
*
\ 1;0;1 ,f n n
Hơn nữa theo chng minh trên
1
6fn
fn
,
*
12 , ,f n k f n n k
Khi đó
12
11
2015 167.12 11 11 3
5 1 2
f
f f f f
f f a
13
11
2016 167.12 12 12 4
6 1 3 1
f
a
f f f f
f f a

Vy hàm s tha mãn yêu cu bài toán.
Câu 65. Xác định hàm s
fx
liên tc

thỏa mãn đồng thời các điều kin:
22f x f x
vi mi
x
,
1
32
11
fx
x
f f x e x e f x
vi mi
x
,
2
1 1 1f e e f
,
3
fk
là s nguyên dương với mi s nguyên dương
k
,
4
Li gii
Vi
,ab
f a f b
, suy ra
33
11
f a f b
f a e f b e
. Do đó
33
11
f a f b
f f a e f f b e
Hay
22
11
ab
a e f a b e f b
. Vì
0f a f b
nên ta suy ra
22
11
ab
a e b e
.
Xét hàm s
2
1
x
h x x e
trên
, ta có
2
1' 20
xx
h x e x ex
vi mi
x
.
Do đó hàm số
2
1
x
h x x e
đồng biến trên
.
Do đó từ
22
11
ab
a e b e
, ta suy ra
h a h b
hay
ab
.
Vy
fx
là đơn ánh. Kết hp vi
fx
liên tc ta suy ra
fx
là hàm đơn điệu thc s.
Mt khác, theo gi thiết
2 2 1 1f f f
nên ta suy ra
fx
là hàm tăng thực s trên
tp
.
T
2
ta cho
1x
thì
1
3
1 1 1 1
f
f f e e f
.
Kết hp vi
3
ta suy ra
1
3
1 1 1
f
f f e f e
.
fx
là hàm tăng thực s trên
nên ta suy ra
1
3
1 1 1
f
f e e
.
Xét hàm s
3
1
x
g x x e
trên
, ta có
23
30' 1
xx
g x e x ex
vi mi
x
.
Do đó hàm số
3
1
x
g x x e
đồng biến trên
.
Chinh phc olympic toán| 71
Phương trình hàm trên tập ri rc
CHINH PHC OLYMPIC TOÁN
Do đó từ
1
3
1 1 1
f
f e e
, ta suy ra
11g f g
hay
11f
.
22f x f x
vi mi
x
11f
nên theo quy np ta có
22
nn
f
vi mi
s t nhiên
n
.
Vi mi s t nhiên
n
, ta có
11
2 2 2 1 2 2 2 2 1 2 2
n n n n n n n n
f f f f f

Vì điều kin
4
nên
2 1 , 2 2 , , 2 2 1
n n n n
f f f
đều là các s nguyên dương.
Do đó ta suy ra
2 1 2 1, 2 2 2 2, , 2 2 1 2 2 1
n n n n n n n n
f f f
.
Vy
f n n
vi mi s nguyên dương
n
.
T
22f x f x
vi mi
x
. Ta suy ra
22
nn
f x f x
vi mi
x
.
Cho vi mi
2
n
m
x
vi mi
,mn
là s nguyên dương ta suy ra
2
2
n
n
m
f m f



.
Do đó
2 hay
2 2 2
n
n n n
m m m
m f f

mi s nguyên dương
,mn
.
Vi mi
x
tùy ý cho trước đều tn ti dãy s
k
q
,
k
q
có dng
2
n
m
hi t đến
x
.
fx
là hàm liên tc nên
lim lim lim
k k k
k k k
x q f q f q f x
 
Th li ta thy hàm s
f x x
tha mãn mọi điều kin ca bài ra.
Câu 66. Tìm tt c các hàm
:f
thỏa mãn đồng thời hai điều kin sau:
Vi mi cp a, b nguyên dương không nguyên tố cùng nhau, có
.f a f b f ab
Vi mi b a, b nguyên dương tồn ti mt tam giác không suy biến độ dài ba
cnh là
,f a f b
1f a b
.
Li gii
T điu kin
2
, vi mi b a, b nguyên dương, ta có
1;
1;
1;
f a f b f a b
f a f a b f b
f a b f b f a
Nếu
2
2: 4 2 ;2 2 3 .a b f f f f
Nếu
a 3;b 2 : 2 3 4f f f
Ta có
2
2 4 2 3 2 2 2 3 2f f f f f f f
2 1 or 2 2.ff
Nếu
21f
. Do
2 2 1 1 1.f f f
72 | Tạp chí và tư liệu toán hc
Bồi dưỡng hc sinh gii
TẠP CHÍ VÀ TƯ LIỆU TOÁN HC
Quy np chng minh
1fn
vi mi
n
nguyên dương.
Cho
, 2 : 1 2 2 1 1a n b f n f n f f n
.
Nếu
22f
, bng quy np chứng minh được
1
2 2 . 2 .... 2 2
k
k k k
f f f f
.
Do
4 2 3 2 2 3 3f f f f f
Quy np chng minh
,2f n n n
Cho
1, 2 : 1 2 1 .a n b f n f n f n f n n
Ly
r
là s nguyên ln nht sao cho
2
r
không vượt quá
n
.
Nếu
2
r
n
thì theo chng minh trên có
f n n
.
Nếu
2
r
ns
vi
12
r
s
.
Với
2 ; 2 1.
rr
a n s b s
Ta có
2 1 2 1 2 1 2 2 1 1
r r r r r
f s s f n f s f s s
11
2 2 1 2 2 1 2 1 1
r r r r r
f n f f s s s n f n n

Vậy
,2f n n n
.
Do
1 2 2 4ff
nên
1f
bng
1, 2
hoc
3
.
Vy
1fn
vi mi n nguyên dương hoặc
,2f n n n
;
1 1; 2; 3f
.
Câu 67. Tìm các hàm s
: 1;f 
tho mãn điều kin:
f x f y y x f xy
vi mi
,1xy
1
Li gii
Vi mi t > 1, thay
; ; 2 , ; 4x y t t
2 ; 2t
vào
1
ta được:
2 2 2
4 4 4
2 2 2 2 4
f t f t f t
f t f t f t
f t f t f t
4 3 2 2 5 4 , 1f t f t t f t t
2
Ly
51
42
22
t f f
Thay vào
2
ta đưc
5
2 2 5 4
2
t f t t f t



Do đó với mi
2
5
1, 4
22
f
t t f t
t
T
1
ta có
22
4 4 4
f
f t f t f t
t
vi
5
1, .
2
tt
Vi
5
2
t
, t
1
thay
5
,2
2
xy
ta có:
Chinh phc olympic toán| 73
Phương trình hàm trên tập ri rc
CHINH PHC OLYMPIC TOÁN
4 2 2 2
51
25
5
2 2 5
2
ff
f f f



22
,1
f
f t t
t
Đặt
22
c
c f f x
x
vi
1x
.
Th li thỏa mãn điều kin
1
.
Vy hàm s cn tìm là
c
fx
x
.
Câu 68. Tìm tt c các hàm
**
:f
thỏa mãn đẳng thc:
2 2 2 2
22f f m f n m n
, vi mi
*
,.mn
Li gii
Nếu
*
12
,mm
sao cho
12
f m f m
2 2 2 2 2 2 2 2
1 2 1 2
2 2 2 2f f m f n f f m f n m n m n
,
Suy ra
12
mm
hay
f
là đơn ánh.
T đó
22
22
2 2 2 2
2 2 2 2f m f n f p f q m n p q
1
Dế thy vi mi
*
,3nn
ta có
2 2 2 2
2 2 1 2 2 1n n n n
2
Chú ý. Điu này vẫn đúng nếu ta nhân c 2 vế vi cùng mt tha s
Đặt
2
1 3 3f a f a
. Theo
1
suy ra:
2 2 2 2 2
2 2 2 2 2
5 2 3 2 3 3 3 27f a f a f a f a f a
phương trình
22
2 27xy
ch nghiệm nguyên dương
; 3, 3xy
hoc
5,1
nên ta có
22
1, 5 5f a f a
.
Cũng từ
1
ta có
2 2 2 2
2 2 2 2
2 4 2 2 5 24f a f a f a f a
.
Vì phương trình
22
12xy
ch có nghiệm nguyên dương là
,xy
4,2
nên
22
4 4, 2 2f a f a
T
1
ta
2 2 2 2
2 2 2 2
4 2 3 2 1f k a f k a f k a f ka
, suy ra t khai trin
2
Vì vy theo các kết qu trên và phép quy np ta suy ra
2
f ka k
, vi mi k là s nguyên
dương. Do đó
3
1f a a f
f
đơn ánh nên
3
11aa
.
Vy
f n n
vi mọi n nguyên dương. Thử li tha mãn bài toán.
74 | Tạp chí và tư liệu toán hc
Bồi dưỡng hc sinh gii
TẠP CHÍ VÀ TƯ LIỆU TOÁN HC
Câu 69. Tìm tt c các s nguyên không âm
n
sao cho tn ti mt hàm
: 0;f
khác hng thỏa mãn đồng thời 2 điều kin sau
i)
,,f xy f x f y x y
ii)
22
2 0;1; 2;...; .,y f x f y x yf x n 
Li gii
Vi
a
bt kì, bng cách thay
*
;
k
kx y a
vào i) đưc
12 2 0;1; 2;...;
kk
f a f f a n


1
Nếu
20f
thì
2 0;1; 2;...; 0
k
f a n f a
Nếu
20f
thì ta thy
0fa
hoc
1fa
.
Tht vy, nếu
1fa
thì bng cách cho
k 
, ta thy
221
kk
f a f f a



.
Nên
1
không th xy ra, còn nếu
01fa
thì vi
k
đủ ln ta có
2012
kk
f a f f a


Nên
1
cũng không thể xy ra.
Thành thử, ta đã chứng minh được vi mi
a
thì
0fa
hoc
1fa
.
T đó suy ra
22
2 0;1; 2 ; ,f x x yy f x f y



2
Do đó,
2.n
Nếu
0,n
thì
22
;2,y f x fx x yyf
.
f
khác hng nên tn ti
0
x
sao cho
0
0fx
.
Khi đó
00
1 1 1f x f x f f
.
Do
f
khác hng nên tn ti
1
x
sao cho
1
1fx
.
T i), ta có
1
0 0 0 0f f x f f
Bây gi, s dng
2
ta được
22
2 2 1 0 1 0 1f f f
.
Điu vô lí này chng t
0n
không tha mãn.
Nếu
1n
thì hàm s
0
10
0 if
fx
i
x
xf
.
Thỏa mãn đề bài. Do đó
1n
thỏa mãn đề bài.
Nếu
2n
thì ta thy không th tn ti 2 s
,;,1ppq q
sao cho
22
0fp q
.
Tht vy, nếu trái li, thì
,xy
ta có
22
2 2 2 2 2 2 2 2
0 f p q f x y f p q x y f xp yq xq yp
Chinh phc olympic toán| 75
Phương trình hàm trên tập ri rc
CHINH PHC OLYMPIC TOÁN
Kết hp vi
2
suy ra
0.xp yq xp yf qf 
Thế nhưng, do
,1pq
nên tn ti
,xy
để
1.xp yq
Do đó
1 0.f xp yq
Điu vô lí này chng t
22
1; , ; , 1.y x yf yx x
Bây gi, ta xét hàm s
0
1
p
if
fx
px
if
qx
x
qx
, trong đó
,pq
2 s nguyên t phân bit
dng
4 3.k
Ta s chng minh hàm
fx
xây dựng như trên thỏa mãn 2 điều kin:
i)
,,f xy f x f y x y
ii)
22
2 0;1; 2;...; .,y f x f y x yf x n 
Kiểm tra điều kin i)
Nếu
p xy
q xy
thì hin nhiên
0f xy f x f y
.
Nếu
x
pq y
pq
thì
1f xy f x f y
Kiểm tra điều kin ii)
0;1fx
nên
22
2 0; 2, 1;y f x f y x yfx
D thy
2
22
11
22
2
1
00
2
00
f f p
f p q f p
fp
fq
fff

nên
22
2 0; 2, 1;y f x f y x yfx
Vy
, 21n n
là tt c các giá tr thỏa mãn đề bài.
Câu 70. Tìm tt c các hàm s
: * *f
tho mãn điều kin:
3
2 2 2 2 *
2 . . , ,f m n f m f n f n f m m n
Li gii
Gi s tn ti hàm s
f
tha mãn các yêu cu của đề bài.
Nếu
f n c
, vi
c
là hng s thì hin nhiên thỏa mãn điều kin ca bài toán.
Nếu tn ti
*
,mn
sao cho
f m f n
thì ta gi
,ab
là 2 s tha mãn
*
min , ,f a f b f m f n m n
1
Gi s
f a f b
. Ta có
3 2 2 3
2 . . 2f b f a f b f b f a f a
.
Vy
2 2 2 2
f b f a b f a f a b f b f a f b
.
76 | Tạp chí và tư liệu toán hc
Bồi dưỡng hc sinh gii
TẠP CHÍ VÀ TƯ LIỆU TOÁN HC
T đó
2 2 2 2
f a f b f a f b f a b f b f a b f b
2
Rõ ràng
2
mâu thun vi
1
.
Do đó
f n c
, vi c là hng s là tt c các hàm cn tìm.
Câu 71. Tìm tt c các hàm s
:f
tho mãn điều kin:
0fc
31
1 , *
3
fn
f n n
fn
1
Li gii
T
1
ta có
1
tan
3
6
1
1
1 1 tan
6
3
fn
fn
fn
f n f n

0 tan
6
1
1 0 tan
6
f
f
f
Do đó ta đặt
0 tanfc
thì
1 tan
6
f



tan tan
1 tan
2
66
6
2 tan
6
1 1 tan
1 tan tan
6
66
f
f
f











Ta chng minh quy np công thc
tan ,
6
f n n



2
Tht vy, vi
0, 1,2n
công thc
2
đúng.
Gi s
tan
6
n
fn



Ta có
tan tan
tan
66
6
1 tan 1
6
1 tan
1 tan tan
6
66
n
fn
f n n
fn











Hay
2
đúng với
1.n
Nghim ca bài toán là
tan ,
6
f n n



.
Câu 72. Tìm tt c các hàm s
:f
tha:
2 2 ,f a f b f f a b a b
International Mathematical Olympiad 2019
Li gii Trần Bá Đạt
Lần lượt thay
0, 0ab
ta được:
Chinh phc olympic toán| 77
Phương trình hàm trên tập ri rc
CHINH PHC OLYMPIC TOÁN
02f f b f f b
,
2 2 0 ,f a f f f a
vi
,ab
Do đó
2 2 0 ,f a f a f
vi mi
a
Thay vào phương trình đề bài ta được
2 2 0f a f b f f f a b
Cho
0,a b a b
trong phương trình trên ta được:
02f f a b f f a b
Do đó:
0f a f b f a b f
Đặt
0g x f x f
suy ra
g a g b g a b g x cx
hay
f x cx d x
.
Thay vào phương trình ban đầu và cân bng h s cho
ab
ta thu được:
0cd
hoc
2c
.
Vy
0f
hoc
2f x x d
.
Câu 73. Có tn ti hay không hàm s
:f
sao cho
, , 1f m f n f m n m n
Li gii
Gi s tn ti hàm
f
tho mãn đề bài.
T
1
cho
0m
ta có
0f f n f n
2
.
Vi
12
,nn
12
f n f n
thì
12
f f n f f n
T
2
suy ra
12
0 0 ,f n f n
do đó
12
nn
nên
f
là đơn ánh.
Cho
0n
t
1
ta có
00f m f f m m f m
T đó ta được
00f
thay vào
2
,f f n n n
3
.
T
1
thay
m
bng
fm
và áp dng
3
ta được
.f f m f n m n
Xét
,,,m n p q
là các s nguyên sao cho
,m n p q
khi đó
f f m f n m n p q f f p f q
Theo chng minh trên
f
là đơn ánh, nên suy ra
f m f n f q f p
Do đó với mi
n
ta có
1 1 1 1f n f n f n f n f n f n f n f n
1 1 ... 2 1 1 0f n f n f n f n f f f
Nên
fn
mt cp s cng vi s hạng đầu
1
00Uf
công sai
1df
suy
ra
11
, 0.
n
f n U U nd nd n
Ta xét vi hai s
0, 0nm
sao cho
0m nd
thay vào
1
đưc
f m f n f m nd f m n f m nd md n m nd d md n
T đó có
2
1d 
, điều này vô lý do vy không tn ti hàm
f
tho mãn yêu cu của đề bài.
78 | Tạp chí và tư liệu toán hc
Bồi dưỡng hc sinh gii
TẠP CHÍ VÀ TƯ LI
U TOÁN H
C
Câu 74. Cho hàm s
:f
là hàm s thỏa mãn các điu kin sau:
i)
,,f mn f m f n m n
ii)
mn
là ước ca
f m f n
vi mi
,mn
Chng minh rng tn ti mt s t nhiên l
k
sao cho
,.
k
f n n n
India National Olympiad 2018
Li gii
Gi
,P x y
là phép thế
,m x n y
vào điều kin i)
,Q x y
là phép thế
,m x n y
vào điều kin ii)
Thế
1, 1P
thì ta được
1.1 1 1 1 1f f f f
f
Thế
2, 2Q
thì ta được:
2 2 2 2 2 2 2 2 , , 2, 1
k
f f f f q k q
Gi s ta xét vi
1q
thì tn ti mt s nguyên t
p
sao cho
pq
suy ra
p
mt s
nguyên t l.
T
2 2 , , 2, 1
k
f q k q
nên ta suy ra:
2.pf
Thế
1
2,
2
p
P



thì ta được
11
2. 1 2
22
pp
f f p f f


T
2pf
nên ta suy ra:
1.p f p
Thế
1, 1Qp
thì ta được:
1 1 1 1 1 1 1 1 1p f f p p f f p p f p p
Điều này là hoàn toàn vô lý, do đó ta phải có
1 2 2 .
k
qf
Thế
2,1Q
thì ta được:
2 1 2 1 3 2 1 2 1 0 mod3 1 1 0 mod3
k
kk
f f k
phi là s l.
T điu kin i) thì ta được:
2.2...2 2 2 ... 2 2 .2 ...2 ,
k k k
f f f f
trong đó
m
ln s
2
2 2 2 .
m
m k km
f
T đây, ta thế
,2
m
Qn
thì ta được:
2 2 2 2 1
m m m km
n f n f n f n
Mà ta biết rng:
kk
x y x y
khi
k
là s l.
T đó ta suy ra:
2 2 2
k
m k m
nn
T
1
2
thì ta được:
Chinh phc olympic toán| 79
Phương trình hàm trên tập ri rc
CHINH PH
C OLYMPIC TOÁN
2 2 2 , 2 , 3
m km k km m k
n f n n m n f n n m
Khi đó, với
m
là mt s t nhiên đủ ln thì
3
xy ra khi:
,,
k
f n n n k
là s t nhiên l.
Vy t đây ta suy ra được điều phi chng minh.
Câu 75. Tìm tt c các hàm s
**
:f
thỏa mãn đồng thời các điều kin sau:
i)
0 0, 1 1ff
ii)
0 1 2 ...f f f
iii)
2 2 2 2 *
,,f x y f x f y x y
Baltic MO
Li gii
Ta có:
2 2 2 2 2 2 2 2
2 1 1 1 1 2, 5 1 2 1 2 5,...,
nn
f f f f f f f f f x x
đây
2
01
1, 1,
nn
x x x n
Hin nhiên thì ta có:
lim
n
x
x


T đây suy ra nếu
1f m f m
thì:
2
2 2 2 2 2 2
22
1 1 1 1 1 1 1 1
1 , 1,2 2
f m f m f f m f m f m
f m k f m k m
Quy np lên thì ta có tn ti vô hn s
m
sao cho
22
1 , 1, 2 2f m k f m k m
Ta chn
m
đủ ln sao cho tn ti
n
để
22
1
, 1, 2 2
nn
a a m m m


Khi đó thì
1nn
aa
điu này hoàn toàn vô lý nên suy ra
f
là hàm s tăng thực s.
T đó thì hiển nhiên ta có:
*
,,f n n n
th li thì thy tha mãn.
Vy tt c các hàm s tha mãn yêu cu bài toán là
*
,.f n n n
Câu 76. Tìm tt c các hàm s
:f
thỏa mãm các điều kin sau:
i) Nếu
ab
thì
f a f b
ii)
22
,,f ab f a b f a f b a b
Mathlinks Contest
Li gii
Trước hết, ta có mt nhn xét nh sau đây.
Nếu
fx
mt nghim hàm thì
f x c
cũng một nghim hàm tha mãn yêu cu bài
toán.
80 | Tạp chí và tư liệu toán hc
Bồi dưỡng hc sinh gii
TẠP CHÍ VÀ TƯ LI
U TOÁN H
C
Do đó ta có thể gi s rng
1 0.f
Chú ý rng t
1 n
thì
0, .f n n
Ta s giải bài toán này thông qua các bước sau đây.
c 1. T
1.1 1 1 1 1f f f f
suy ra
21ff
hoc
2 0.f
c 2. Gi
n
là s nguyên sao cho
1
là s chính phương modulo
.n
Do đó tồn ti
s
a
tha mãn:
2
1.a kn
Suy ra:
22
1 1 1 1 0f a f a f a f f a f kn f
Nhưng
2
1f n f kn f a
1f n f
nên
1 0.f n f
Do đó nếu tn ti
u
sao cho
2
1 modun
thì
0.fn
c 3. T c 2 d thy
0fp
vi mi
p
là s nguyên t
1 mod 4 .p
c 4. Gi s
0f a f b
0f ab f a f b
thì
22
0,f a b
vô lý.
Do đó nếu
0f a f b
thì
0.f ab
c 5. Gi
,ab
hai s nguyên tha mãn
, 1,ab
khi đó gọi
p
một ước ca
22
ab
thì ta có:
22
0 mod .a b p
Mà ta mt b đề quen thuc sau. Nếu
p
mt s nguyên t có dng
43k
thì vi mi
b s
,ab
tha mãn
22
p a b
thì ta s
pa
.pb
,1ab
nên nếu
22
p a b
thì
p
ch có dng
4 1.k
T c 4 thì ta có:
22
ab
là tích ca các s nguyên t
i
p
tha mãn
0
i
fp
nên
22
0.f a b
Mà t
22
,,f ab f a b f a f b a b
ta có
, 1 .a b f ab f a f b
c 6. Cho
a bc
vào phương trình đã cho thì ta được:
2 2 2
1f b c f b c f bc f b
Nhưng do
22
1f b f b c
2
f bc f b c
Do đó thì ta có:
2
*f bc f b c
Thế
1c
vào
*
thì được:
2
.f b f b
Thế
cb
vào
*
thì đưc:
23
.f b f b
T đấy, bng phép quy nạp thì ta được
,1
k
f b f b k
c 7. S dng c 5c 6 thì ta có:
i
n
ii
f p f p
đây
i
p
là các s nguyên t
Xét hàm s
fx
xác định bi:
*
1 0, 2 0ff
Chinh phc olympic toán| 81
Phương trình hàm trên tập ri rc
CHINH PH
C OLYMPIC TOÁN
*
0fp
vi các s nguyên t
p
sao cho
1 mod 4p
2.p
*
0
p
f p a
vi mi s nguyên t
p
còn li, đây
p
a
là các s nguyên không dương.
*
i
n
ii
f p f p
đây
i
p
là các s nguyên t.
Ta có th chng minh
fx
thỏa mãn các điều kin:
Hin nhiên nếu
ab
thì
.f a f b
1.1 1 1 1 1 0f f f f
2
.1 1 1f a f a f a f
Ta có mọi ước nguyên t
p
ca
2
1a
đều tha mãn
1 mod 4 .p
Vi hai s nguyên
,1ab
bt kì, ta gi:
i
p
là các ước nguyên t ca
a
không chia hết cho
.b
i
q
là các ước nguyên t ca
b
không chia hết cho
.a
i
r
là các ước nguyên t ca
a
.b
ii
f a f p f r

ii
f b f q f r

i i i
f ab f p f q f r
22
,
ii
f a b f r f s

đây
i
s
là các ước nguyên t ca
22
.
,,
ab
A
a b a b

Nhưng tương tự c 5 ta có:
Các ước nguyên t ca
A
là các s nguyên t tha mãn
1 mod 4
i
s
và do đó
0.fA
Suy ra
22
i
f a b f r
Hay ta có
22
,,f ab f a b f a f b a b
Và ta có nghim của phương trình hàm là:
Cho
M
là mt s nguyên, hàm
f
được xác định như sau:
*
1fM
*
2fM
*
f p M
vi mi s nguyên t
p
tha mãn
1 mod 4 .p
*
p
f p M a
vi mi s nguyên t
p
còn li, đây
p
a
là các s nguyên không dương.
*
i
n
ii
f p M f p M
đây
i
p
là các s nguyên t.
82 | Tạp chí và tư liệu toán hc
Bồi dưỡng hc sinh gii
TẠP CHÍ VÀ TƯ LI
U TOÁN H
C
Câu 77. Tn ti hay không hàm s
: 1,2,...,fn
thỏa mãn điều kin:
i)
f
là hàm đơn ánh
ii)
f ab f a f b
vi mi
, 1, 2,...,a b n
ab n
Li gii
Ta có th ch ra hàm s
f
như sau:
Kí hiu các s nguyên t bé hơn hoặc bng
n
theo th t tăng dần là:
12
, ,..., .
k
p p p
Khi đó nếu
1
, , 1
i
k
i i i
i
a p n
t
1
1 , 1
k
i
ii
i
f p n f a n
Ta s chng minh hàm s này tha mãn yêu cu bài toán.
Tht vy, vi
1
i
k
i
i
ap
1
i
k
i
i
bp
có ít nht mt giá tr
i
j
khác nhau.
Thì
1
1
k
i
i
f a n
1
1
k
i
i
f b n
Hin nhiên ta ch th biu th
,f a f b
mt cách duy nht sang h số
1n
vì
thế
.f a f b
Mt khác ta có
1 1 1
1
ii
k
k k k
i i i i i i i i
i i i
i
f ab f p f p f p f p f a f b




Hay ta có:
,f ab f a f b
đúng theo giả thiết đề bài.
Vy t đó hàm số xây dựng như trên thỏa mãn yêu cu bài toán.
Câu 78. Gi s Josephus
1n
người bn,
n
người này đúng thành một vòng tròn
đánh số t
1
đến
n
theo chiều kim đồng h, t sát theo nguyên tắc, người th nht cm
dao đếm
1
ri t sát, người th hai đếm
2
ri t sát,…Quá trình dừng li khi còn mt
người. Gi
fn
hàm s biu th v trí cùa người sống sót đó. Câu hỏi đặt ra là, hãy
tính
fn
?
Bài toán c Josephus
Li gii
Ta s xét hai kh năng của
n
là chn và l.
Kh năng 1. Khi s
n
s chn, tc
2.nk
Sau vòng
1
thì còn ngưi v trí l. S người
này đánh lại thành
1, 2,..., .k
Nếu lượt trước người đó số
21i
thì sau đó mang s
.i
Người sng sót s
2fk
sau mang s mi là
.fk
Vy t đây ta có
2 2 1f k f k
Kh năng 2. Khi s
n
s l, tc
2 1.nk
Sau vòng
1
ta ngm hiu rng
22k
người bằng cách tính trùng người th
1
thành
2 2,k
còn li những người s
3, 5,..., 2 1k
đánh số li là
1, 2,..., .k
Chinh phc olympic toán| 83
Phương trình hàm trên tập ri rc
CHINH PH
C OLYMPIC TOÁN
Nếu lượt trước người đó số
21i
thì sau đó mang s
.i
Người sng sót s
21fk
sau mang s mi là
.fk
Vy t đây ta có
2 1 2 1f k f k
Như vậy thì
1 1, 2 2 1, 2 1 2 1f f k f k f k f k
Ta chng minh bng quy np rng.
Nếu biu diễn trong cơ số
2
ca
n
là:
11
2
... , 1,
k k k
n a a a a

vi
ik
0,1
i
a
Thì
11
2
... .
kk
f n a a a
Tht vy:
Vi
1n
thì ta thy hiển nhiên đúng.
Gi s vi mi
kn
thì mệnh đề đúng. Ta sẽ xét hai trường hợp sau đây:
Trường hp 1. Nếu
n
là s chẵn, đặt
2.nm
Khi đó nếu như:
11
2
...
kk
m b b b
thì
11
2
2 ... 0
kk
m b b b
1 1 1 1
2
2 2 1 2 2 ... .2 1 1 ... 01
k
kk
f m f m b b b b

Vy trong trưng hp 1 thì mệnh đề đúng.
Trường hp 2. Nếu
n
là s lẻ, đặt
2 1.nm
Khi đó nếu như:
11
2
...
kk
m b b b
thì
11
2
2 1 ... 1
kk
m b b b

1 1 1 1
2
2 1 2 1 2 2 ... .2 1 1 ... 11
k
kk
f m f m b b b b

Vy trong trưng hp 2 thì mệnh đề đúng.
T đó, theo nguyên quy np thì mệnh đề ban đầu đúng từ đó ta suy ra điều phi
chng minh.
Câu 79. Cho hai hàm s
**
,:fg
là hai hàm s thỏa mãn đồng thời các điu kin:
i)
g
là hàm s toàn ánh
ii)
2 2 2
2,f n n g n n
Nếu
2019 ,f n n n n
thì
f
có vô s đim bất động.
Li gii
Đầu tiên ta định Dirichlet v s nguyên t thì dãy s
i
p
vi
i
p
các s nguyên
có dng
83k
là mt dãy vô hn.
T đó với mi
,n
theo công thc ca kí hiu Legendre ta có:
2
1
8
2
11
n
p
n
p



S dụng điều kin i) thì ta tìm được dãy
1
n
n
x
sao cho
,.
nn
g x p n
Ta có
2 2 2 2 2
2 2 mod
n n n n n n
f x x p f x x p
2
1
n
p




nên suy ra:
n
nn
p f n
px
84 | Tạp chí và tư liệu toán hc
Bồi dưỡng hc sinh gii
TẠP CHÍ VÀ TƯ LI
U TOÁN H
C
Suy ra tn ti hai dãy s nguyên dương
n
a
n
b
sao cho
n n n
n n n
x a p
f x b p
T điu kin ii) thì ta được
22
21
nn
ba
Cui cùng, s dng gi thiết:
2019 ,f n n n n
thì ta có:
2019
11
n
n
nn
n
fx
b
xa
x
2
1
lim 2 lim 1
n
n
xx
n
a
a
a
 
Suy ra tn ti
0
N
sao cho
0
1,
nn
a b n N
Vy t đó
0
,.
nn
f p p n N
Và t đây ta suy ra điều phi chng minh.
Câu 80. Tìm tt c các hàm s
**
:g
thỏa mãn điều kin sau:
g g 1 3 ,g n n n n g n n
Doãn Quang Tiến
Li gii
Đặt
*
,g n f n n n
thì thay vào phương trình hàm ban đầu ta được:
*
*
1 1 3 ,
1 2, 1
f f n f n f n n n f n n n
f f n f n n n
T đây, ta đã chuyển bài toán ban đầu thành mt bài toán khác v gn đẹp hơn rt
nhiu.
Thay
1n
vào
1
thì ta được
1 2 3f f f
T đó ta suy ra:
22f
1 2.ff
T đó ta xét hai trường hp sau:
Trường hp 1.
21f
1 2.ff
Bây gi ta đặt
12f k f k
Thay
2n
vào
1
thì ta đưc
2 3 4f f f
T đây suy ra
3 4 1 4f f k
T
31f
nên suy ra:
3.k
Nếu
1k
thì ta có:
2 1 1 1,f f f k f
điều này cũng mâu thuẫn.
Nếu
2k
thì ta cũng có:
2 1 2 1,f f f k f
điều này cũng mâu thuẫn.
Nếu
3k
thì ta có:
2 1 3 4 4 3 1,f f f k f k
điều này cũng mâu thuẫn.
Vy tóm li không có giá tr nào ca
k
tha mãn nên trưng hp 1 không xy ra.
Trường hp 2.
22f
1 1.ff
Thay
2n
vào
1
thì ta được
2 3 4f f f
T đó thì ta dễ thy
32f
và ta tính toán được các giá tr sau:
Chinh phc olympic toán| 85
Phương trình hàm trên tập ri rc
CHINH PH
C OLYMPIC TOÁN
4 5 3 5 2 3
5 6 4 6 3 4
6 7 5 7 4 4
f f f f
f f f f
f f f f
T đấy ta d đoán được rng, hàm s
fn
được xác định như sau:
*
1,f n n n n


trong đó
15
2

Bây gi ta s chng minh rng, hàm s này hàm s tha mãn
1
để ri t đó suy ra
công thc ca hàm
gn
và t đó ta hoàn tất bài toán.
Mà trước tiên, để chng minh nhận định đó, ta cần phi có hai b đề sau:
B đề 1. Vi mi s
*
n
thì ta có
1
1
n
nn
n




Chng minh
Trước hết ta có
1 1 1 2n n n n n n n




1 1 1 1 1 1 1n n n n n n




Do lưu ý rằng:
1 5 1 5
1
22
nên suy ra:
1 1.
Vy t đó ta thấy b đề 1 đưc chng minh.
B đề 2. Vi mi s
*
n
thì ta có
2 1
1
1
n if n n n
n
n otherwise







Chng minh
Hin nhiên thì ta có
1n


bng
1n


hoc
2n


Gi s
11nn




thì t đó ta có:
1 1 1 1 1 1 1 1n n n n n n n n








Và như trên thì ta có
1 1 1 2n n n n n n n




T đó thì ta suy ra được:
1 1.n n n




Gi s
12nn




thì t đó ta có:
1 1 1 1 1 1n n n n n n n








T đó thì theo b đề 1 thì ta thu được:
1.n n n




Vy t đó thì b đề 2 đưc chng minh.
Quay tr li vi vic gii bài toán
Ta s s dụng phương pháp quy nạp để chng minh kết qu ban đầu.
Vi
1n
thì
1 .1 1 1 1.f


86 | Tạp chí và tư liệu toán hc
Bồi dưỡng hc sinh gii
TẠP CHÍ VÀ TƯ LI
U TOÁN H
C
Vi
2n
thì
2 .2 2 1 2.f


Gi s kết qu đúng với
1.jn
S dng
1
thì ta có:
1 2 2 1 2 1 1 1f n n f f n n f n n n n n n n


Mà t
1 2 1 1n n n n n


ta có
1 2 1f n n n n


Gi s
n
tha mãn:
1n n n




thì t đó ta có:
12nn




Và do đó ta suy ra được
11f n n n


Nếu
n
không tha mãn
1n n n




thì tc là ch có th xy ra:
1 1.n n n




Và theo b đề 2 thì ta được:
11nn




Và t đó ta suy ra được:
1 2 1 2 1 1 1f n n n n n n n n n n




Vy t đó theo nguyên lý quy nạp thì mệnh đề
1
đưc chng minh hoàn toàn.
T đây suy ra tất c các hàm s tha mãn
1
*
1,f n n n n


trong đó
15
.
2

Hay t đây ta suy ra được hàm
gn
mà chúng ta cn tìm là:
*
1 1,g n f n n n n n n n
trong đó
15
.
2

Câu 81. Cho ba s thc
,,a b c
không âm, phân bit sao cho tn ti hàm
,:fg
tha mãn
x
af xy bf cf x g y
y



vi mi s thực dương
xy
.
Chng minh rng tn ti hàm
:h
sao cho:
2 , 0
x
f xy f f x h y x y
y



Iran TST 2019
Li gii
Đặt
,P x y
là phép thế cho phương trình:
x
af xy bf cf x g y
y



Trường hp 1: Nếu
0a b c
:
Ta có
1
,1
g
P x f x
a b c


là hng s, vy:
2
x
f xy f f x
y




Suy ra
0h y y
, tha mãn.
Chinh phc olympic toán| 87
Phương trình hàm trên tập ri rc
CHINH PH
C OLYMPIC TOÁN
Trường hp 2: Nếu
0a b c
:
Ta có
x
a f xy f x b f f x g y
y







và đặt phép thế
,Q x y
Ly
1, ,Q y Q x y
ta được
1
1 1 0 1
x
a f xy f x f y f b f f x f f
yy




Thế
1
y
y
o
1
ta được:
1
1 1 0
x
a f f x f f b f xy f x f y f
yy




Do
ab
,
0a b c
22
0ab
, vì vy:
1f xy f x f y f
, , 0xy
1
1
x
f f x f f
yy
,
,0xy
Cộng hai phương trình với nhau
1
2 2 1
x
f xy f f x f y f f
yy
Vy
1
21h y f y f f y
y



, tha mãn.
Câu 82. Tìm tt c hàm s
:f
tha mãn:
! ! ! ! , ,n f m f n f m m n
BMO Shortlist 2018
Li gii
Đặt phép thế
,P m n
cho phương trình ban đầu
1,1P
1 1 ! 1 1 !f f f
1 ! 1 1 1ff
, t đây hiển nhiên
11f
1, ! 1 ! 1 ! !P n n f n f n n f n n
Gi
p
là s nguyên t tùy ý, ta có
1, 1 1 ! 1 1 ! 1P p p f p
.
Theo định lý Wilson trong s học ta được
1 ! 1pp
, suy ra
1 ! 1p f p 
Lưu ý rằng nếu
11f p p
thì
1!fp
là tích ca ít nht
p
tha s nguyên dương và
là mt s chia hết cho
p
, do đó
1 ! 1 1 modf p p
- mâu thun chng minh trên.
Vy
11f p p
, li có
f n n
nên
11f p p
vi mi s nguyên t
p
.
Li có
,1P m p 
1 ! ! ! 1 ! 1 ! ! ! !p f m f m p p f m f m f m
88 | Tạp chí và tư liệu toán hc
Bồi dưỡng hc sinh gii
TẠP CHÍ VÀ TƯ LI
U TOÁN H
C
Vi giá tr
m
bt k, ta chn
p
đủ lớn để thu được:
!!f m f m
, s dng kết qu này ta
đưc:
! ! ! !n f m f n f m
tương đương
! ! ! !n f m f n n
Thay
1mp
vi
p
đủ lớn vào phương trình trên ra được
!!f n n
vi mi
n
.
Vy
f n n
là hàm s cn tìm.
Câu 83. Tn ti hay không hàm s
**
:f
thỏa mãn điều kin sau:
*
3 2 ,f f n n f n n
Li gii
Gi s tn ti hàm s
f
tha mãn yêu cu bài toán.
Vi mi
*
i
ta xây dng dãy s như sau
1
n
n
a
sao cho:
11
,
nn
a i a f a

Khi đó thì ta có
1 1 1 1 1
2 3 2 3
n n n n n n n
a f a f f a f a a a a
Hay ta có
41
4 3 0, 1
n n n
a a a n

Do
0, 1
n
an
nên đẳng thc không th xy ra.
Nên t đó ta kết lun rng không tn ti hàm s
f
tha mãn yêu cu bài toán.
Câu 84. Tìm tt c các hàm s tăng thực s
**
:f
thỏa mãn điều kin sau:
*
2,f n f n f n n
Li gii
Do
f
là hàm s tăng thực s nên ta có:
11f n f n
hay
11f n n f n n
Suy ra:
f n n
là hàm s tăng.
Mặt khác ta đặt
01
1,
n n n
a a a f a
T đó ta suy ra
01
...aa
1
2
nn
f a f a
Do đó
*
11
,
n n n n
f a a f a a n

Suy ra có vô hn b s
,mn
sao cho:
f m m f n n
Suy ra
,f n n c
vi
c
là hng s.
Vy tt c các hàm s tha mãn yêu cu bài toán là:
*
,,f n n c n c
là hng s.
Câu 85. Tìm tt c các toàn ánh
:f
sao cho vi mi
,mn
tha mãn:
f m f n m n
Li gii
Kí hiu
P
là tp tt c các s nguyên t.
Xét đơn ánh
:g P P
thì nếu
1
i
k
i
i
np
thì
1
.
i
k
i
i
f n g p
Chinh phc olympic toán| 89
Phương trình hàm trên tập ri rc
CHINH PH
C OLYMPIC TOÁN
Kí hiu
n
là s ước nguyên dương của
.n
Mà ta có nhn xét sau:
n f n
do
f
là toàn ánh.
Vi mi s nguyên t
p
thì
fp
ch có đúng hai ước nguyên t nên nó cũng là số nguyên
t.
Xác định hàm
g
như trên thì từ đó ta có:
f p g p
nên ta s chng minh
g
là song ánh.
Tht vy, do
f
là toàn ánh nên
g
là toàn ánh nên
g
là song ánh.
Tiếp theo, ta s chng minh
k
k
f p g p
vi
k
là s nguyên dương bằng quy np
Ta thy rng, vi
1k
thì hiển nhiên đúng.
Gi s mệnh đề đúng với
1.k
Ta
k
fp
chia hết cho
21
1, , ,...,
k
g p g p g p
ngoài ra không chia hết cho s
nguyên dương nào khác.
Do đó
1
kk
f p p k
.
Nếu
1k
khi
k
fp
có thêm một ước nguyên t na thì
2 1,
k
f p k k
vô lý.
T đó
k
fp
là lũy thừa ca
gp
và nó có
1k
ước nên
.
k
k
f p g p
Gi s
n
là mt s nguyên dương,
p
là mt s nguyên t không chia hết cho
.n
Bây gi ta s đi chứng minh
,
kk
f n f p f np k
T
,1
k
np
nên ta có:
kk
n p np
Mt khác
k
k
g p f np
g p f n
Do vy mọi ước ca
fn
k
gp
chia hết cho
k
f np
và mọi ước ca
k
gp
và
fn
ước ca
.
k
f np
Li có
k k k
f n f p np f np
Nếu
k
f np
có ước khác với các ước ca
fn
k
gp
thì
,
kk
f n f p f np
vô lý.
Vy t đó ta có kết qu
k
kk
f np f n g p f n f p
T các nhn xét trên ta có hàm
f
đưc xây dựng như trên là duy nhất.
Vy tt c các hàm s tha mãn yêu cầu bài toán là các hàm như trên.
TÀI LIU THAM KHO
Dưới đây là các tài liệu mà ebook này có tham khảo và đồng thi có c nhng tài liu mà
bọn mình đề xut cho bạn đc
[1]. Nguyễn Văn Mậu (1997), 00 Phương trình hàm 00, NXB Giáo dục.
[2]. Nguyn Trng Tuấn (2004), “Bài toán hàm s qua các kì thi Olympic”, NXB Giáo
dc.
[3]. Nguyn Tài Chung, Lê Hoành Phò (2013), “Chuyên khảo phương trình hàm” Nhà
xut bản Đại hc quc gia Hà Ni.
[4]. J.Aczél (1966), “Lectures on functional equations and their applications”, ACADEMIC
PRESS New York San Francisco London.
[5]. Stevo Stevic (2004), “Periodic Character of a Class of Difference Equation”, Taylor &
Francis Group.
[6]. Valentine Boju, Luis Funar - The Math Problems Notebook.
[7]. Titu Andreescu, Razvan Gelca Birkhauser Mathematical Olympiad Challenges.
[8]. Edward Lozansky , Cecil Rousseau Winning Solutions.
[9]. The IMO Compendium. A Collection of Problems Suggested for The International
Mathematical Olympiads: 1959 2009 Djukic D., Vladimir Jankovic, Ivan Matic, Nikola
Petrovic Springer (2011).
[10]. Trần Nam Dũng, Dương Bửu Lc Chuyên đề Phương trình hàm trên tập s nguyên .
[11]. (Developments in Mathematics 39) Saïd Abbas, Mouffak Benchohra Advanced.
[12]. Functional Evolution Equations and Inclusions-Springer International Publishing (2015)
[13]. Aczel Lectures on functional equations and their applications Academic Press (1966).
[14]. An Introduction to the Theory of Functional Equationsand Inequalities Marek Kuczma.
[15]. Analytic Solutions of Functional Equations Sui Sun Cheng, Wenrong Li.
[16]. Functional Analysis, Sobolev Spaces and Partial Differential Equations Haim Brezis.
[17]. Topics in Algebra and Analysis Preparing for the Mathematical Olympiad Radmila.
[18]. Bulajich Manfrino, José Antonio Gómez Ortega, Rogelio Valdez Delgado-Birkhäuser
Basel (2015).
[19]. K yếu gp g toán hc và các k yếu t hi thảo chuyên đề các tnh.
[20]. The art of Mathematics.
[21]. 101 Problems in Algebra from the training of the USA IMO team T Andreescu, Z
Feng.
[22]. Problem Primer for the Olympiad C. R. Pranesachar, B. J. Venkatachala, C. S.
Yogananda
[23]. https://artofproblemsolving.com
TẠP CHÍ VÀ TƯ LIỆU TOÁN HC
HT
CHINH PHC OLYMPIC TOÁN
TẠP CHÍ VÀ TƯ LIU TOÁN HC
Thôn 6 Thch Hòa Thch Tht Hà Ni
Điện thoi: 0343763310; Email: tuangenk@gmail.com
Fanpage: https://www.facebook.com/OlympiadMathematical/
CHU TRÁCH NHIM NI DUNG
DOÃN QUANG TIN
NGUYN MINH TUN
TÔN NGC MINH QUÂN
BIÊN TP
NGUYN MINH TUN
TRÌNH BÀY BN THO
NGUYN MINH TUN
LA TH ĐÔNG PHƯƠNG
PHƯƠNG TRÌNH HÀM TRÊN TẬP RI RC
Đề ngh quý bạn đọc tôn trng bn quyn ca tác gi, không sao chép bn ph.
Mi ý kiến thc mắc đóng góp vui lòng gửi v địa ch đã cung cấp trên.
Phiên bản sách điện t đưc phát hành vào ngày 20/8/2019.
0343763310
tuangenk@gmail.com
Lovetoan.wordpress.com
Đại học FPT Hà Nội
CHINH PHC OLYMPIC TOÁN
NGUYN MINH TUN
MI Ý KIN THC MC XIN VUI LÒNG GI V ĐỊA CH
PHIÊN BN
ĐẶC BIT
| 1/98

Preview text:

CHINH PHỤC OLYMPIC TOÁN NGUYỄN MINH TUẤN DOÃN QUANG TIẾN TÔN NGỌC MINH QUÂN PHƯƠNG TRÌNH HÀM TRÊN TẬP RỜI RẠC
TẠP CHÍ VÀ TƯ LIỆU TOÁN HỌC Omaths Littited Chuyên đề Edition Bồi dưỡng Học sinh giỏi Phương trình hàm Trên tập rời rạc Chinh phục Olympic toán
TẠP CHÍ VÀ TƯ LIỆU TOÁN HỌC
TẠP CHÍ VÀ TƯ LIỆU TOÁN HỌC
Copyright © 2019 by Tap chi va tu lieu toan hoc.
All rights reserved. No part of this book may be reproduced or distributed in any form
or by anymeans, or stored in data base or a retrieval system, without the prior written the permission of the author. LỜI GIỚI THIỆU
Những bài toán phþơng trình hàm ngày nay đã trở nên rất phổ biến đối với các bän C
học sinh yêu Toán vì chúng đã xuất hiện thþờng xuyên trong các đề thi học sinh giỏi Ọ
các cấp cüng nhþ kì thi chọn đội tuyển quốc gia, VMO hay các kì thi khu vực và H
quốc tế mà ta đþợc biết đến. Đặc biệt, trong các lớp däng phþơng trình hàm, thì
däng phþơng trình hàm trên các tập rời räc là một mâng đþợc ít các học sinh chú ý
tới bởi độ khó và chþa đþợc tiếp xúc nhiều đồng thời ngoài việc sử dýng các kï thuật
xử lý phþơng trình hàm cơ bân chúng ta còn phâi sử dýng các tính chất số học rất
đặc sắc cûa tập rời räc nhþ là: tính chia hết, tính chất cûa số nguyên tố, cûa số U TOÁN Ệ
chính phþơng,... Trong ebook này chúng tôi sẽ mang tới cho bän đọc tuyển tập các
bài toán phþơng trình hàm trên tập rời räc và một số bài toán phþơng trình hàm LI
khác hay và khó với những lời giâi vô cùng đặc sắc nhằm giúp bän đọc có thể có
nhiều cách nhìn khác về mâng toán này đồng thời cüng nhþ chuẩn bð cho các kì học sinh giỏi, olympic. Í VÀ TƯ
CMình xin gửi lời câm ơn tới
Ọ1. Thầy Huỳnh Kim Linh – THPT chuyên Lê Quý Đôn – Khánh Hòa – Đã góp ý CH
H giúp bọn mình về phần nội dung. P
C2. Bän La Thð Đông Phþơng – Đäi học Hoa Sen – Đã giúp bọn mình chînh sửa bân Ạ Ọ T
H thâo đề hoàn thiện hơn.
Một lần nữa gửi lời câm ơn các bän, các thầy cô đã ûng hộ và theo dõi fanpage suốt
thời gian qua. Hy vọng ebook này sẽ giúp ích đþợc cho mọi ngþời. Thank you!
Nhóm tác giả
Nguyễn Minh Tuấn
Doãn Quang Tiến
Tôn Ngọc Minh Quân
PHƯƠNG TRÌNH HÀM TRÊN TẬP RỜI RẠC
TẠP CHÍ VÀ TƯ LIỆU TOÁN HỌC CHINH PHỤC OLYMPIC TOÁN
Phương trình hàm trên tập rời rạc Chuyên đề
PHƯƠNG TRÌNH HÀM TRÊN TẬP RỜI RẠC
Tạp chí và tư liệu toán học
Để giải quyết các bài toán phương trình hàm trên tập rời rạc mà có thể giải bằng các tính
chất số học thì nên lưu ý đến một số dấu hiệu sau:
 Nếu xuất hiện các biểu thức tuyến tính chứa lũy thừa, có thể nghĩ đến các bài toán
liên quan đến cấp của phần tử, các phương trình đặc biệt như phương trình Pell
hay phương trình Pythagore,nghiệm nguyên.
 Nếu hàm số đã cho là hàm nhân tính, ta thường hay xét đến giá trị hàm số tại các ÁN
điểm là số nguyên tố hoặc dãy vô hạn các số nguyên tố. O
 Sử dụng các đẳng thức và bất đẳng thức số học. IC T
 Và đặc biệt nhất, trong một số bài toán, hệ cơ số đếm có thể dùng để xây dựng P
nhiều dãy số có tính chất số học thú vị. Trong hệ cơ số 10 chúng ta có thể rất khó YM
nhận ra quy luật của dãy, nhưng nếu chọn được hệ cơ số phù hợp thì bài toán có OL
thể giải quyết đơn giản hơn rất nhiều. C g 2, g , Ụ Nếu  
với g là cơ số đếm, thì mọi số nguyên dương M đều biểu diễn PH duy nhất dưới dạng: n1 n
M a a ...a a g  2 a g
 ... a g a 1 a g 1;0 a g i n i 1, 2, . 1 2 n g 1 2 n1 n với         1
NH I Cơ số đếm mà hay được sử dụng trong các bài toán phương trình hàm trên tập rời rạc là 2 CH và 3.
Sau đây, chúng tôi sẽ đề cập đến các bài toán phương trình hàm mà sử dụng các tính chất
cũng như các phương pháp trong số học để giải, nhằm giúp bạn đọc hiểu rõ hơn và có một
cái nhìn mới mẻ hơn về các phương pháp khác để giải phương trình hàm, bên cạnh đó
chúng tôi cũng sẽ giới thiệu cho bạn đọc các bài toán phương trình hàm và khó trong tài
liệu này. Nào cùng bắt đầu nhé! Chinh phục olympic toán| 1
Bồi dưỡng học sinh giỏi I. ĐỀ BÀI
Câu 1. Tìm tất cả các hàm số f :
 thỏa mãn điều kiện sau:
3 f n  2 f f n  n,n
Câu 2. Tìm tất cả các hàm số f :
 thỏa mãn điều kiện sau
mnf  2 m  2
n   mf n  nf m,m,n 1 Câu 3. Cho hàm số * f  * :
thỏa mãn điều kiện sau:
f n    f f n n * 1 ,
Chứng minh rằng f n  n n  * , .
Câu 4. Tìm tất cả các hàm số * f  * :
thỏa mãn điều kiện sau: 2
x f y 2
f x  y x y  * , , 
Câu 5. Tìm tất cả các hàm số * f  * :
thỏa mãn điều kiện sau: C 2 2 2 * Ọ
f m  f n m n ,m,n * H
Câu 6. Tìm tất cả các hàm f :
 thỏa mãn tồn tại số k  và số nguyên tố p sao cho ÁN
với mọi n k, f n p   f n  và nếu m n thì f m  1 f n  1.
Câu 7. Cho p là số nguyên tố lẻ. Tìm tất cả các hàm f :
 thỏa mãn đồng thời các U TOỆ điều kiện: LI
i) f m  f n với m n mod p TƯ
ii) f mn  f mf n ,m,n  VÀ f :  0,  Í
Câu 8. Tìm số nguyên không âm n nhỏ nhất sao cho tồn tại hàm số   khác
hằng số thỏa mãn đồng thời các điều kiện: CH P
i) f xy  f xf y ,x, y  ẠT
ii) f  2 x  2 2
y   f x  f y0,1,..., 
n ,x, y
Với số n tìm được, hãy tìm tất cả các hàm số thỏa mãn.
Câu 9. Giả sử hàm số * f :
 thỏa mãn các điều kiện sau:   n  1  1  f
if n  2m     1   2 
f 1  1 và f n     n  1  f if n     2m   2 
Tìm các giá trị của n sao cho f n  2019.
Câu 10. Tìm tất cả các hàm số * f  * :
thỏa mãn các điều kiện sau:
2 | Tạp chí và tư liệu toán học
Phương trình hàm trên tập rời rạc
f 1  1, f 3  3 
f 2n  f n 
với mọi số nguyên dương n .
f 4n  1  2 f 2n  1   f n
f 4n 3  3 f 2n1  2 f nCâu 11. Cho hàm số   f : 
thỏa mãn đồng thời các điều kiện:
f n là ước của 2018 n với mọi n
f a. f b  f c với mọi  a,b,c  và 2  2  2 a b c
a) Chứng minh rằng nếu n lẻ hoặc n 4 thì f n  1
b) Gọi A là tập hợp giá trị có thể có của f 2  f 2018 . Tính A
Câu 12. Có tồn tại hàm số f : S S thỏa mãn điều kiện
f af b  f  2 2
a b ,a,bS,a b S  * \ 1 không, trong đó  ? * * f : ÁN
Câu 13. Tìm tất cả các hàm số  thỏa mãn điều kiện O
n  2  f nf f n  2
n n n * 1 , . IC T
Câu 14. Tìm tất cả hàm số f :
 thỏa mãn đồng thời hai điều kiện sau: P
i) x f y f x  y f x f y  với mọi x, y  ; YM
 f x  f y  OL
ii) Tập hợp I  
,x, y  ,x y là một khoảng C  x   y  Ụ 2
Câu 15. Tìm các hàm số * f  * : thỏa mãn 2
f m  f n  2
m n m n * , , PH
Câu 16. Cho hàm f x, y thỏa mãn các điều kiện: NH I
f 0, y  y  1; f x  1,0  f x,1 CH
f x  1, y  1  f x, f x  1,y      
Với mọi số nguyên không âm x, y . Tìm f 4,1981 ? Câu 17. Cho hàm   f : 
thỏa mãn các điều kiện sau: i) f n 1 f n    ;n
ii) f f n 
 3n,nZ .
Hãy tính f 2003.
Câu 18. Cho f n là hàm số xác định với mọi  * n
và lấy giá tị không âm thỏa mãn tính chất:  n m * ,
: f m n  f m  f n lấy giá trị 0 hoặc 1
f 2  0 và f 3  0 .
f 9999  3333 . Chinh phục olympic toán| 3
Bồi dưỡng học sinh giỏi Tính f 2000 .
Câu 19. Cho f , g là các hàm xác định trên thỏa mãn điều kiện
f x y  f x y  2 f x.g y ,x, y
Chứng minh rằng nếu f x  0 và f x  1,x  thì g y   a  1 0
Câu 20. Cho hàm số f :  thỏa 2 điều kiện
i) f x  1  x;x
ii) f x y  f x. f y ;x, y
Chứng minh rằng không thể tồn tại hai số a; b
f a. f b  0 2003
Câu 21. Cho f x, y 
cos 2x y  acosx y   với a,  . 2 2 2
Chứng minh rằng min  f x, y  max f x, y  2003. C 2 x 1 Ọ
Câu 22. Cho hàm số f x   ,x  0. 2x H
Giả sử f x  x
f x f f
x n * ,x   0 n n 1  0 và    . ÁN f x n   1 Chứng minh n   , x   1  ,0,1  1  n f x n 1   2  x  1  U TO f   Ệ  x  1  LI
Câu 23. Cho hàm số * f  *  * :
là hàm số thỏa mãn đồng thời các điều kiện sau: TƯ
i) f 1,1  2 VÀ
ii) f m n  f m n  m m n  * 1, , , , Í
iii) f m n    f m n  n m n  * , 1 , , , CH P
Tìm tất cả các cặp số  p, q sao cho f p,q  2019. ẠT
Câu 24. Tìm tất cả các hàm số f :
 thỏa mãn các điều kiện sau:
i)f x  2 0 x ,n
ii) f x  f y chia hết cho x y với mọi x, y  , x y
Câu 25. Tìm tất cả các hàm số * f  * :
*  x x    mà tập    0 thỏa mãn:   f xy
f x f y  xyf xy    * 2 f  , x, y 1 x y      Câu 26. Cho hàm  f :
 là một hàm số thỏa mãn với mọi n  1 thì có một số nguyên  n
tố p là ước của n sao cho: f n  f p  f   1 và  p
f  2018   f  2019   f  2020 3 5 7   2017.
4 | Tạp chí và tư liệu toán học
Phương trình hàm trên tập rời rạc
Hãy tính giá trị của biểu thức G f  2018   f  2019  f  2020 2018 2019 2020 
Câu 27. Tìm tất cả các hàm số * f  * : thỏa mãn: 3 f  2 m  2 n   2
f mf n  f m 2
f n m n * 2 , ,
Câu 28. Giả sử f :
 là hàm liên tục và giảm sao cho với mọi  x, y  ta có
f x yf f x  f y  f y f x
Chứng minh rằng f f x  x .
Câu 29. Cho song ánh f :
 . Chứng minh rằng tồn tại vô số bộ a,b,c với a,b,c
thỏa mãn a b c và 2 f b  f a  f c .
Câu 30. Có bao nhiêu hàm f : * 
* thoả mãn đồng thời các điều kiện sau
a) f 1  1 2
b) f nf n  2  9 f n  1  1997, n  * . ÁN * * O
Câu 31. Tìm tất cả các hàm số f :  sao cho. a) f 2  2
IC TP b) f  .mn f m.f nvới mọi m n * ,
, UCLN m,n  1 YM
c) f m  f n m n * , , m n . OL
Câu 32. Tìm tất cả các hàm số f :  thỏa mãn C Ụ
f m n  f mn  f mf n  1,m,n PH
Câu 33. Tìm tất cả các hàm số f :
 thỏa mãn f 0  2 và
f x f x  2y  f 2x  f 2y ,x, y  NH 1 I
Câu 34. Tìm tất cả hàm số f :
 sao cho f f n  f n  2n  3,n 1 CH
Câu 35. Chứng minh rằng tồn tại duy nhất hàm số f : *  * thỏa mãn
f m f n  n f m b ,m,n * b  i
Câu 36. Hãy xác định tất cả hàm số * f  * : thỏa mãn đẳng thức:
f n  f n  1  f n  2. f n  3  a 1
Với a là số tự nhiên thỏa mãn a  1 là số nguyên tố.
Câu 37. Tìm tất cả các hàm số * * f : 
thỏa mãn f n a
f n an t a k t    1.     với f n f f f n a t
k 2t  1  a  1 t  
 ...   với , là số tự nhiên tùy ý thỏa mãn   . t
Câu 38. Cho hàm số f :  thỏa mãn:
 f 2n1 f 2n1 f 2n1 f 2n1  31 2 f n  
  n  f n ,n f 2 Chinh phục olympic toán| 5
Bồi dưỡng học sinh giỏi
Tìm n sao cho f n  2009 .
Câu 39. Tìm tất cả các hàm số f :  thoả mãn: 1 f xy 1
f xz  f xf yz 1  , x  ,y,z . 3 3 9
Câu 40. Cho n  n  2 và hàm số f :  sao cho: f n x yn1
x f xf f y; x  ,y  *
a) Giả sử rằng f 2002  0. Tính f 2002. b) Tìm hàm số f .
Câu 41. Tìm tất cả các hàm số f :  thỏa mãn f  2 3
x y z   f x 2  f y 3  f zx  ,y,zCâu 42. Cho hàm số * * f : 
thỏa mãn đồng thời hai điều kiện:
a) f ab  f a,b f a,b với mọi * a,b
, a b ; trong đó a,b, a,b lần lượt là bội C
chung nhỏ nhất, ước chung lớn nhất của hai số nguyên dương a,b ; Ọ H
b) f p q r   f p  f q   f r  với mọi số nguyên tố p, q, r . ÁN
Tính giá trị của f 2013 ? Kí hiệu * là tập hợp tất cả các số nguyên dương.
Câu 43. Đặt F f : 0,  1  0, 
1 và n  2. Tìm giá trị nhỏ nhất của c thỏa mãn điều kiện U TO 1 1 nf
x dx c f x dx   0     0 LI
Với f F f là hàm liên tục. TƯ
Câu 44. Tìm tất cả các hàm f :  1
 ,1  liên tục, thỏa mãn: VÀ  2x  Í
f x  f  , x   1  ,1 2  1 x    CH
Câu 45. Có thể tồn tại hay không một hàm số f :
 , liên tục trên và thỏa mãn điều P Ạ f x f x  1 T
kiện: Với mọi số thực x , ta có   là số hữu tỉ khi và chỉ khi   là số vô tỉ.
Câu 46. Tìm tất cả các hàm số f :
 thỏa mãn điều kiện f x  f t  f y  f z với
mọi số hữu tỉ x y z t x, y, z,t theo thứ tự lập thành cấp số cộng.
Câu 47. Giả sử r , s
là hai số cho trước. Tìm tất cả các hàm số f :  thỏa mãn điều
kiện f x f y  f x r   y s, x  ,y  ?
Câu 48. Tìm tất cả các hàm số f :
 sao cho với tất cả các số nguyên a,b,c thỏa mãn
a b c  0 , đẳng thức sau là đúng:
f a2  f b2  f c2  2 f af b 2 f bf c 2 f cf a
Câu 49. Tìm tất cả các hàm f , g :  
có đạo hàm trên  thỏa mãn f x f xg x '   ; g x   '   x    x x
6 | Tạp chí và tư liệu toán học
Phương trình hàm trên tập rời rạc
Câu 50. Tìm tất cả các hàm * f :  
có đạo hàm trên * thỏa mãn
f xy  f x  f y * x  , y  1 
Câu 51. Tìm tất cả các hàm f :  thỏa mãn
f f n  n b n    1
trong đó b là số nguyên dương chẵn.
Câu 52. Tìm tất cả các hàm f :    thỏa mãn:
i) f xf y yf xx,y     1
ii) lim f x  0 x
Câu 53. Chứng minh rằng tồn tại song ánh f :    sao cho
f 3mn m n 4 f mf nf mf nm,n        
Câu 54. Tìm tất cả các hàm f :  thỏa: ÁN
3 f f f n  2 f f n  f n  6n, n   O
Câu 55. Tìm tất cả các hàm số f : 0;   0;  thỏa mãn điều kiện: IC TP
f f x  yf yf x x
 , y 0;  1 YM
Câu 56. Chứng minh rằng tồn tại duy nhất một hàm số f xác định trên tập các số thực
dương, nhận giá trị thực dương và thỏa mãn f f x  6x f x. OL C
Câu 57. Hàm số f :
 thỏa mãn đồng thời các điều kiện sau: Ụ
i : f f n  n, n   1 PH
ii : f f n 2 2  n, n   2
NH I iii: f 0 1 3 CH f 1995 , f 2007  Tìm giá trị    
Câu 58. Tìm f : 0, 1 
thỏa mãn f xyz  xf x  yf y  zf zx
 ,y,z0,1
Câu 59. Tìm tất cả các hàm f xác định trên
và thỏa mãn đồng thời các điều kiện sau:
2 f nf k n  2 f k n  3 f nf k,k n   f  1  1
Câu 60. Tìm tất cả các hàm số * * f : 
thỏa mãn đồng thời hai điều kiện sau:
f f n * *
n  2k, n , k     f
 n  1  f n * , n
Câu 61. Tìm tất cả các hàm số f :
 thỏa mãn đồng thời hai điều kiện sau:  f  2013  2016 
f f n  n  4, n   
Câu 62. Tìm tất cả các hàm số * f : 
thỏa mãn điều kiện sau: Chinh phục olympic toán| 7
Bồi dưỡng học sinh giỏi
f n  f n  1  f n  1. f n  3 , n   1
Câu 63. Tìm tất cả các hàm f :    thỏa mãn:
f x f y  f x y  f y
Câu 64. Tìm số nguyên dương m nhỏ nhất sao cho tồn tại hàm số * f :  \ 1  ;0;  1
thỏa mãn đồng thời các điều kiện sau
i) f m  f 2015 , f m  1  f 2016 ; f n  1
ii) f n m     f n , n 1,2,....  1
Câu 65. Xác định hàm số f x liên tục   
thỏa mãn đồng thời các điều kiện:
f 2x  2 f x với mọi x   , 1 
 3  fx    2 1   x f f x e
x e  1 f x với mọi x   , 2 C
f e  1  e  1 f 1 , 3 Ọ
f k là số nguyên dương với mọi số nguyên dương k , 4 H
Câu 66. Tìm tất cả các hàm f : 
thỏa mãn đồng thời hai điều kiện sau: ÁN
 Với mọi cặp a, b nguyên dương không nguyên tố cùng nhau, có f a. f b  f ab U TO
 Với mọi bộ a, b nguyên dương tồn tại một tam giác không suy biến có độ dài ba Ệ LI
cạnh là f a , f b và f a b  1 . f : 1;   TƯ
Câu 67. Tìm các hàm số   thoả mãn điều kiện:
f x  f y  y xf xy với mọi x, y  1 1 VÀ Í
Câu 68. Tìm tất cả các hàm * * f :  thỏa mãn đẳng thức: CH f  2 f m 2  f n 2 2 2
m  2n , với mọi * m,n . P Ạ
Câu 69. Tìm tất cả các số nguyên không âm n sao cho tồn tại một hàm f :  0; khác T
hằng thỏa mãn đồng thời 2 điều kiện sau
i) f xy  f xf y , x  , y ii) f  2 2
2 x y   f x  f yx,y   0;1;2;...;  n .
Câu 70. Tìm tất cả các hàm số f : *  * thoả mãn điều kiện:
f m n  3 2 2 2
f mf n 2
f nf m * 2 . . , m  ,n
Câu 71. Tìm tất cả các hàm số f :
 thoả mãn điều kiện:
f 0  c 3 f n  1
f n  1      1
f n , n * 3
8 | Tạp chí và tư liệu toán học
Phương trình hàm trên tập rời rạc
Câu 72. Tìm tất cả các hàm số f :  thỏa:
f 2a  2 f b  f f a b a  ,b
Câu 73. Có tồn tại hay không hàm số f :  sao cho
f m f n  f m  n, m  ,n 1
Câu 74. Cho hàm số f :
 là hàm số thỏa mãn các điều kiện sau:
i) f mn  f mf n , m  ,n
ii)m n là ước của f m  f n với mọi m, n
Chứng minh rằng tồn tại một số tự nhiên lẻ k sao cho   k
f n n , n   .
Câu 75. Tìm tất cả các hàm số * * f : 
thỏa mãn đồng thời các điều kiện sau:
i) f 0  0, f 1  1
ii) f 0  f 1  f 2  ... 2 2 2 2 * ÁN
iii) f x y   f x  f y , x  , y  O
Câu 76. Tìm tất cả các hàm số f :
 thỏa mãm các điều kiện sau: IC T
i) Nếu a b thì f a  f b P
ii) f ab  f  2 2
a b   f a  f b , a  ,b  YM
Câu 77. Tồn tại hay không hàm số f : 1, 2,..., 
n  thỏa mãn điều kiện: OL C
i) f là hàm đơn ánh Ụ
ii) f ab  f a  f b với mọi a,b 1, 2,..., 
n ab n PH
Câu 78. Giả sử Josephus có n  1 người bạn, n người này đúng thành một vòng tròn
NH I đánh số từ 1 đến n theo chiều kim đồng hồ, tự sát theo nguyên tắc, người thứ nhất cầm CH
dao đếm 1 rồi tự sát, người thứ hai đếm 2 rồi tự sát,
người. Gọi f n là hàm số biểu thị vị trí cùa người sống sót đó. Câu hỏi đặt ra là, hãy tính f n ?
Câu 79. Cho hai hàm số * * f , g : 
là hai hàm số thỏa mãn đồng thời các điều kiện:
i) g là hàm số toàn ánh ii) 2 f n 2 2 2 n
g n , n     
Nếu f nn 2019 n, n     
thì f có vô số điểm bất động.
Câu 80. Tìm tất cả các hàm số * * g : 
thỏa mãn điều kiện sau:
g gn  n  g n  1  3  n gn , n  
Câu 81. Cho ba số thực a, b,c không âm, phân biệt sao cho tồn tại hàm f , g :   thỏa  x
mãn af xy  bf    cf x  g y với mọi số thực dương x y .  y  Chinh phục olympic toán| 9
Bồi dưỡng học sinh giỏi
Chứng minh rằng tồn tại hàm h :   sao cho:    x
f xy f    2 f x  hy, x   y  0  y
Câu 82. Tìm tất cả hàm số f :  thỏa mãn:
n! f m! f n! f m! , m  ,n
Câu 83. Tồn tại hay không hàm số * * f : 
thỏa mãn điều kiện sau:
f f n  n f n * 3 2 , n  
Câu 84. Tìm tất cả các hàm số tăng thực sự * * f : 
thỏa mãn điều kiện sau:
f n f n  f n * 2 , n  
Câu 85. Tìm tất cả các toàn ánh f :
 sao cho với mọi m,n  thỏa mãn:
f mf n  m n C Ọ H ÁN U TOỆLI TƯ VÀ Í CH P ẠT
10 | Tạp chí và tư liệu toán học
Phương trình hàm trên tập rời rạc II. LỜI GIẢI.
Câu 1. Tìm tất cả các hàm số f :
 thỏa mãn điều kiện sau:
3 f n  2 f f n  n,nLời giải
Giả sử f là hàm số thỏa mãn điều kiện bài toán.
Đặt g n  f n  n,n  .
Khi đó, thì ta được 2g f n  2  f f n  f n  f n  n gn ,n 1 
Áp dụng liên tiếp 1 ta được
       2 2
2      ...  2m g n g f n g f f n
g f f ...f n... , trong đó có m dấu f . m
Như vậy thì g n chia hết cho 2 ,m  g n  0,n  hay f n  n,n  ÁN
Thử lại thì thấy hàm số f n  n,n  thỏa mãn yêu cầu đề bài. O
Vậy tất cả các hàm số thỏa mãn đề bài là: f n  n,n  . IC T
Nhận xét. Việc đặt hàm phụ g n  f n  n,n  giúp ta đưa phương trình hàm ban P
đầu về dạng mới đẹp hơn. Và khi đó ta phát hiện ra thêm được các tính chất của hàm mới YM
g n để từ đó ta áp dụng liên tiếp các tính chất ấy và kết hợp với các tính chất số học chia OL C
hết để suy ra được hàm thỏa mãn yêu cầu đề bài. PH
Câu 2. Tìm tất cả các hàm số f :
 thỏa mãn điều kiện sau
mnf  2 m  2
n   mf n  nf m,m,n 1 NH I Lời giải CH
Giả sử f là hàm số thỏa mãn điều kiện bài toán.
Kí hiệu P u, v là phép thế ,
u v vào 1 thì ta được:
Pn  nf  2 0,
n   nf 0 ,n Do đó f  2
n   f 0 ,n .
Đặt g n  f n  f 0 ,n .
Khi đó, ta thay vô 1 ta được m ng  2 m  2
n   mgn  ngm ,m,n 2
Hơn nữa, ta còn có g 0  0 và g  2
n   0,n
Kí hiệu Q u, v là phép thế m  ,n
u v vào 2 thì
Qn n  ng 2 , 2
2n   2ngn,n
Do đó ta được g  2
2n   gn ,n và
Chinh phục olympic toán| 11
Bồi dưỡng học sinh giỏi Q 2 2 n n   2 n g  4 n   2 2 , 3 5
n g n ,n
Từ đó suy ra g n  g  4
3 5n ,n
Từ đây ta áp dụng liên tục các tính chất trên, thì ta đó ta suy ra   k g nnk  * 3 , ,
Suy ra: g n  0,n  hay f n  f 0  const,n  .
Thử lại thì ta thấy hàm này thỏa mãn yêu cầu bài toán.
Vậy tất cả các hàm số thỏa mãn yêu cầu bài toán là f n  f 0  const,n  .
Nhận xét. Cũng tương tự như bài toán 1 ta nhìn phương trình hàm ban đầu dưới một hàm
phụ khác, bằng các phép thế cơ bản ta phát hiện ra được một số tính chất sơ khai ban đầu.
Và bằng phép đặt g n  f n  n,n ta được một phương trình hàm có dạng y chang
phương trình hàm ban đầu, nhưng ta lại được thêm các điều kiện ràng buộc là g 0  0 và g  2
n   0,n nên từ đó ta đã được thêm các ràng buộc, thuận lợi cho việc giải phương C Ọ
trình. Phép đặt này rất hay, nó vừa bảo toàn phương trình hàm có dạng y chang ban đầu H
và kèm theo là các điều kiện rang buộc mà phương trình hàm ban đầu không có. Từ đấy, ÁN
tương tự bài toán 1, ta phát hiện các tính chất của hàm g n và sử dụng liên tục chúng và
kết hợp cùng với các tính chất chia hết để suy ra hàm số cần tìm. U TOỆ LI Câu 3. Cho hàm số * f  * :
thỏa mãn điều kiện sau: TƯ
f n    f f n n * 1 ,
Chứng minh rằng f n  n n  * , . VÀ Í IMO 1977 CH Lời giải P Ạ
Giả sử f là hàm số thỏa mãn điều kiện bài toán. T Đặt d
f nn * min ,
,theo nguyên lý cực hạn thì d tồn tại và duy nhất. Gọi  * m
sao cho: f m  . d
Nếu m  1 thì d f m  f f m  1 , mâu thuẫn.
Do đó f n đạt giá trị nhỏ nhất duy nhất tại n  1
Lập luận tương tự thì ta có f   
f nn * 2 min , ,n   2
Và lập luận lại quá trình tương tự như trên ta được:
f 1  f 2  f 3...  f n  ...
Ta có f 1  1 nên f n  n n  * ,
Nếu tồn tại n  * f n n f n n 1. 0 mà    0 0 thì     0 0
12 | Tạp chí và tư liệu toán học
Phương trình hàm trên tập rời rạc
Suy ra f f n   f n  1 , 0 0  mâu thuẫn
Do đó, f n  n n  * ,
, thử lại thấy thỏa mãn yêu cầu bài toán.
Vậy tất cả các hàm số thỏa mãn yêu cầu bài toán là: f n  n n  * , .
Nhận xét. Đây là một bài toán phương trình hàm trong kì thi Toán Quốc Tế - IMO năm
1977, một bài toán phương trình hàm với điều kiện rang buộc là ở dạng bất đẳng thức, rất
lạ và mới. Làm ta nãy ra ý tưởng sử dụng nguyên lý cực hạn để đánh giá để có điều vô lý
và suy ra được hàm số thỏa mãn yêu cầu đề bài.
Câu 4. Tìm tất cả các hàm số * f  * :
thỏa mãn điều kiện sau: 2
x f y 2
f x  y x y  * , ,  Lời giải
Giả sử f là hàm số thỏa mãn điều kiện bài toán. ÁN
Trong  ta thế x y  1 ta được  f   2 1
1 f 1  1  f 1  1 O
Trong  ta thế x  1 ta được  f y 2
f    y   y y  *  y f y  y  * 1 1 1 , , 1
IC TP Trong  ta thế y 1 ta được YM 2
x f   2
f x  x y  *  2 x  2
f x  x y  *  f x  x x  * 1 1, , 1 1, , , 2 OL
Từ 1 và 2 ta suy ra f x  x x  * ,
, thử lại ta thấy thỏa mãn yêu cầu bài toán. C Ụ
Vậy tất cả các hàm số thỏa mãn yêu cầu bài toán là f x  x x  * , . PH
Nhận xét. Đây là một bài toán phương trình hàm trên tập rời rạc, mà cho dưới dạng chia
hết. Bằng các phép thế đơn giản cùng với các đánh giá số học không quá khó khan, ta có
NH I thể nhanh chóng đánh giá được biên của hàm f và để từ đó ta suy ra được hàm số thỏa CH mãn đề bài.
Câu 5. Tìm tất cả các hàm số * f  * :
thỏa mãn điều kiện sau:
f m  f n m n2 2 2 m n * , , *
IMO Shortlist 2004 Lời giải
Giả sử f là hàm số thỏa mãn điều kiện bài toán.
Trong * ta thế m n  1 ta được:
f    f     2 2 2 1 1 1
1  4  f 1  1, do f 1 và f 1  1
Trong * ta thế m  1 ta được:
f    f n   n2 m n
  f n   n2 2 2 * n * 1 1 , , 1 1 ,
Chinh phục olympic toán| 13
Bồi dưỡng học sinh giỏi
Trong * ta thế n  1 ta được:
f m  f   m  2 m
f m  m  2 2 2 * 2 2 m * 1 1 , 1 1 ,
Với p là một số nguyên tố bất kì thì:
Trong * ta thế m  1, n p  1 ta được:
1  f p  1  p
f p   2 1 1 p   1  
f p  1  2 p
Trường hợp 1. f p    2
p f p    2 1 1 1 p  1.
Ta thế m p  1, n  1 vào * ta được: f  2 2
p     p  2    p  2   p  2 2 2 1 1 1 1 1 1 1  1
Mà ta lại có đánh giá sau đây: 2 C
p  2   p 2 p 2  p p 2  p p2   p 2 2 2 2 1 1 1 1 1 1  1 , mâu thuẫn Ọ H
Do đó, ta phải xảy ra trường hợp còn lại. ÁN
Trường hợp 2. 1  f p  1  p f p  1  p  1, với mọi p là số nguyên tố
Hay tồn tại k sao cho f k  k. U TOỆ
Với mỗi k như thế và số tự nhiên n  0 bất kì thì ta có: LI
k f n k n2  k f n  p  2  f n p  2 2 2 2 1
1  2n f n f nn2 TƯ
Khi ta chọn k là một số đủ lớn thì ta bắt buộc phải có: f n  n n  * , , thử lại thỏa. VÀ Í
Vậy tất cả các hàm số thỏa mãn yêu cầu bài toán là: f n  n n  * , . CH
Nhận xét. Cũng tương tự như ở bài toán 4, đây là một bài phương trình hàm trên tập rời P
rạc có dạng chia hết. Cũng tương tự ở bài trên, ta cũng thế bằng các phép thế đơn giản để ẠT
phát hiện một số tính chất của đề bài. Nhưng ở bài toán 5 này khó hơn ở bài toán 4 rất
nhiều, vì từ các tính chất ta tìm được, ta không thể chặn được khoảng của hàm f để rồi
suy ra f n  n n  * ,
như ở bài toán trên được. Vì thế mà ta phải xét giá trị của hàm số
f tại các giá trị là số nguyên tố để xử lý bài toán và bằng một số kiến thức đơn giản về
giới hạn ta có thể suy ra được f n  n n  * ,
một cách dễ dàng, từ đó kết thúc bài toán.
14 | Tạp chí và tư liệu toán học
Phương trình hàm trên tập rời rạc
Câu 6. Tìm tất cả các hàm f :
 thỏa mãn tồn tại số k  và số nguyên tố p sao
cho với mọi n k, f n p  f n và nếu m n thì f m  1 f n  1. Iran TST 2005 Lời giải
Giả sử f là hàm số thỏa mãn điều kiện bài toán.
Giả sử n k p không chia hết cho n  1 thì khi đó tồn tại k sao cho n  1 n k . p
Suy ra ta được f nf n kp  1
Mặt khác ta lại có f n  f n kp nên f nf n  1  f n 1  f n  1
Với n  1 bất kì thì n  1 n  1 kp f nf n  1 kp  1  2
Do đó với n  1 thì ta có: f n 1,  2 .
Bây giờ ta sẽ xét hai trường hợp sau: ÁN
Trường hợp 1. f n  2,n k p n  1. O
Xác định n k p không chia hết cho n  1 khi đó tồn tại m sao cho: n  1 m p m  1.
IC TP Suy ra f nf m13 hay f n1
Ta xác định hàm f như sau: YM
f n  2,n k p n  1. OL C
f n  1,n k p không là ước của n  1. Ụ
f i  f i p ,i k. PH
Trường hợp 2. f n  1,n k p n  1.
NH I Trong trường hợp này f n 1,nk và nếu giả sử S a f a  2 thì sẽ không tồn tại CH
m, n S thỏa mãn m  1 . n
Ta xác định hàm f như sau f n  1,  2 ,n .
Với S là một tập con vô hạn của
sao cho không tồn tại m, n S thỏa mãn m  1 n và với
n  1 thì f n  2  nS; f x  1, với các giá trị x  1 còn lại và f 1 là một số bất kì
xác định bởi f 2 f 1  1.
Từ đây ta thử lại đề bài và thấy thỏa mãn nên ta hoàn thành bài toán.
Nhận xét. Đây là một bài toán phương trình hàm trên tập rời rạc khó và điều kiện ràng
buộc khá là khó chịu. Và bằng các phép thế để tìm ra các tính chất của hàm, cùng với các
kĩ thuật xử lý rất khó khan, chúng ta đã xử lý được bài toán. Đây là một bài toán khó, các
bạn đọc cần nghiên cứu và đọc thật kĩ.
Chinh phục olympic toán| 15
Bồi dưỡng học sinh giỏi
Câu 7. Cho p là số nguyên tố lẻ. Tìm tất cả các hàm f :
 thỏa mãn đồng thời các điều kiện:
i) f m  f n với m nmod p
ii) f mn  f mf n ,m,n USA TST Lời giải
Giả sử f là hàm số thỏa mãn điều kiện bài toán.
Với k  , thì ta có f pk  1  f pk  f p f k  1  f k  0
Bây giờ ta sẽ xét hai trường hợp sau
Trường hợp 1. f p  0
Dễ thấy nếu f 1  0 thì f n  0,n  , mâu thuẫn với f p  0.
Xét riêng khi f 1  1. C Ọ
Với mỗi x  và p khong chia hết cho x ta có y  sao cho xy  1mod p. H
Do đó ta có f xf y  f xy  f 1  1,x, y  ÁN
Suy ra: f n  1 và p không chia hết cho . n
Mặt khác ta lại có f  2 n   2
f n  1 với p không chia hết cho n nên f m  1, nếu m là U TOỆ
một số chính phương mod p p không chia hết cho . m LI
Nếu không tồn tại i, với p không chia hết cho i sao cho f i  1 thì ta có ngay TƯ
f n  1,n và p không chia hết cho . n VÀ Í
Xét i là một số không chính phương mod p k là một số không chính phương mod p
p không chia hết cho k bất kì thì ta suy ra ik là số chính phương mod p. CH P
Mặt khác ta lại có f k   f if k   f ik  1 ẠT Hay
f x  1, nếu x là một số chính phương mod p p không chia hết cho x
f x  1, nếu x là một số không chính phương mod p Xét số x f x 1. 0 sao cho     0
Bây giờ từ điều kiện ii) ta thay m x , n p 0 ta được:
f p  f px   f p f x f p 0
   0  hay    1 Suy ra:
f x  1, nếu x là số chính phương mod p
f x  1, nếu x là một số không chính phương mod p
Trường hợp 2. f p  0 suy ra f n  0,p n.
Khả năng 1. Nếu f 1  0 thì f n  0,n  .
16 | Tạp chí và tư liệu toán học
Phương trình hàm trên tập rời rạc
Khả năng 2. Nếu f 1  0
Giả sử tồn tại x f x 0 x 0 sao cho    0
p không chia hết cho 0
Suy ra f nx   0,n  0
Ta có dãy x , 2x ,...,p  1 x 0 0
 0 là một hệ thặng dư đầy đủ mod p
Suy ra f 1  0, điều này mâu thuẫn.
Vậy ta có f x  0  p x f x  1, với các giá trị x còn lại.
Từ các kết quả trên đây, ta thấy có 4 hàm số thỏa mãn yêu cầu bài toán: 0 if p n
f n  0,n
f n   1 if n p
1 if n la mot so chinh phuong mod p
f n  1,nf n    
 1 if n khong la mot so chinh phuong mod p ÁN
Vậy đây là tất cả các hàm thỏa mãn yêu cầu bài toán. O
Nhận xét. Đây là một bài toán khó, với điều kiện hàm rất khó xử lý, một bài toán khó
trong kì thi chọn đội tuyển IMO của Mỹ, và việc ứng dụng sâu sắc các kiến thức Số Học
IC TP tổng hợp trong lời giải, nó có vẻ khá phức tạp. Mong bạn đọc suy nghĩ và đọc thật kĩ, và YM
mong bạn đọc có một lời giải khác ngắn gọn và hay hơn cho bài toán. OL C
Câu 8. Tìm số nguyên không âm n nhỏ nhất sao cho tồn tại hàm số f :  0, khác Ụ
hằng số thỏa mãn đồng thời các điều kiện: PH
i) f xy  f xf y ,x, y  NH 2 f x  2 2
y f x f y  0,1,...,n ,x, y  I ii)         CH
Với số n tìm được, hãy tìm tất cả các hàm số thỏa mãn. Lời giải
Với n  1 xét hàm f được xác định như sau:
  0 if p x f x  
,với p là số nguyên tố có dạng 4k  3 1 if x p
Hiển nhiên hàm số trên thỏa mãn yêu cầu bài toán.
Giả sử với n  0 thì cũng tồn tại hàm số f thỏa mãn yêu cầu bài toán. Khi đó thì ta có: f  2 x  2
y   f x  f y  x y   f  2 x  2 2 0, , 2
y   f x  f y,x,y  
Từ điều kiện i) ta thế x y  0 ta được: f    2 0
f 0  f 0  0 hoặc f 0  1
Trường hợp 1. f 0  1
Chinh phục olympic toán| 17
Bồi dưỡng học sinh giỏi
Ta thế y  0 vào  thì ta được f  2
2 x   f x  f 0  f x  1,x Mà f  2 x   2
f x ,x  nên ta suy ra: 2
f x  f  2 2
2 x   f x  1,x  f x  1,x , do f x  0,x
Điều này lại trái với giả thiết f khác hằng số.
Trường hợp 2. f 0  0
Ta thế y  0 vào  thì ta được f  2
2 x   f x  f 0  f x ,x  Mà f  2 x   2
f x ,x  nên ta suy ra 2
f x  f  2 2
2 x   f x ,x
Suy ra với mỗi x  thì ta phải có f x  0 hoặc f x  1 . 2 1 Nếu tồn tại x f x . 0 sao cho    0 2
Ta thế x y x  0 vào   thì ta được: C Ọ
2 f 2x   2 f 2 f x   2 f  2 x  2 x f x f x 2 f x 0 0 0 0  
 0   0    0    H
Từ  ta thay x  1, y  0 thì ta được: f 1  0. ÁN
Từ  ta thay x  1, y  1 thì ta được: f 2  0. U TO
Từ đây ta thay f 2  0 vào   thì ta được: Ệ LI
f x  0,x  , điều này lại mâu thuẫn với f khác hằng số.
Vậy từ đây ta khẳng định được rằng n  1 là giá trị nhỏ nhất thỏa mãn yêu cầu bài toán. TƯ
Khi ta tìm được n  1 ta sẽ quay lại việc giải quyết bài toán đề bài. VÀ Í
Tìm tất cả các hàm số f :
 0, khác hằng số thỏa mãn đồng thời các điều kiện: CH P
i) f xy  f xf y ,x, y  ẠT ii) f  2 x  2 2
y   f x  f y0, 
1 ,x, y
Giả sử f là hàm số thỏa mãn điều kiện bài toán.
Ta dễ dàng chứng minh được rằng: f 0  0, f 1  1.
Trong i) ta thế y x thì ta được f  2 x   2
f x ,x
Trong ii) ta thế y  0 thì ta được f  2
x   f x  2 2
2 f x  f x0, 
1  f x0,  1
Trong i) ta thế x y  1 thì ta được 2
f 1  f 1  1  f 1  1
Trong i) ta thế x  1, y  x thì ta được:
18 | Tạp chí và tư liệu toán học
Phương trình hàm trên tập rời rạc
f x  f 1 f x ,x   f x  f x ,x
Trường hợp 1. Tồn tại số nguyên tố p sao cho f p  0.
Giả sử cũng tồn tại số nguyên tố q p sao cho f q  0.
Trong ii) ta thế x p, y q thì ta được f  2 p  2
q   f p  f q   f  2 p  2 2 0 q   0
Do đó với mỗi a, b  thì ta luôn có:
f a b f p q   f a b p q   f ap bq2  aq bp2 2 2 2 2 2 2 2 2 2 2 2 0 2 2
Lưu ý rằng.  2  2  2  2 a b p
q   ap bq  aq bp là đẳng thức Brahmagupta –
Fibonacci nổi tiếng, cũng đã được đề cập đến trong nhiều cuốn sách, mong bạn đọc lưu ý
chi tiết này để giải toán.
Vì  f x  f y  f  2 x  2 0 2
y  nên f aq bp  0. ÁN
Do  p,q  1 nên tồn tại a, b  sao cho aq bp  1. O
Suy ra được 1  f 1  f aq bp  0, điều này là vô lý.
IC TP Vậy tồn tại duy nhất số nguyên tố p sao cho f p0. 2 YM
Khả năng 1. Nếu p là số nguyên tố có dạng 4k  1, k  thì tồn tại a  sao cho p a  1 2 OL
hay f a  1  0. C Ụ
Lưu ý rằng. Kết quả này các bạn có thể tham khảo trong phần chuyên đề Thặng dư bình phương. PH
Mặt khác, trong ii) ta thế x  1, y a thì ta được: f  2  2
a   f    f a  f  2
a    f    f a   f  2 2 1 1 2 1 1 1 a  1  1,
NH I Điều này là mẫu thuẫn. CH
Vậy từ đấy chỉ xảy ra khả năng còn lại.
Khả năng 2. Nếu p là số nguyên tố có dạng 4k  3 thì
Từ đó ta có f x  0  p x f x  1 với các giá trị x còn lại.
Trường hợp 2. f p  1 với mọi số nguyên tố p.
Khi đó f x  1,x  \  0
Vậy từ đó có hai hàm số thỏa mãn yêu cầu bài toán là: 0 if p x
f x  
, trong đó p là một số nguyên tố bất kì có dạng 4k  3, k  . 1 if x p 0 if x 0  f x    1 if x   0
Nhận xét. Đây là một bài toán phương trình hàm trên tập rời rạc phải nói là rất rất khó, sử
dụng rất nhiều kiến thức trong Số Học cũng như kĩ năng phán đoán và biến đổi thuần
Chinh phục olympic toán| 19
Bồi dưỡng học sinh giỏi
thục. Sử dụng rất nhiều các mạng kiến thức liên quan đến số nguyên tố, thặng dư bình
phương hay các đẳng thức rất nổi tiếng trong Toán học. Thực sự đây là một bài hàm liên
quan đến số học tổng hợp, rất hay và thú vị, mong bạn đọc nghiên cứu thật kĩ càng và cẩn thận bài toán này.
Câu 9. Giả sử hàm số * f :
 thỏa mãn các điều kiện sau:   n  1  1  f
if n  2m     1   2 
f 1  1 và f n     n  1  f if n     2m   2 
Tìm các giá trị của n sao cho f n  2019. Lời giải
Từ cách xác định của hàm f ta dễ dàng tính được: C Ọ
f 2  f 3  2; f 4  f 5  f 6  f 7  3 H
Bây giờ ta sẽ viết dưới dạng nhị phân như sau: ÁN
f 1  f 1 1; f 2 f 10 2; f 3 f 11 2 2       2       2  
f 4  f 100 3; f 5 f 101 3; f 6 f 110 3;... 2       2       2   U TOỆ
Từ cách việt dưới dạng nhị phân như trên, ta dự đoán f n là số chữ số trong biễu diễn LI nhị phân của số . n
Ta sẽ chứng minh dự đoán này bằng quy nạp như sau. n 1,n 2. VÀ
Thật vậy, ta thấy khẳng định đúng với   Í
Giả sử khẳng định đúng đến .
n Ta sẽ chứng minh khẳng định đúng đến n  1. CH
Nếu n là số chẵn thì n a a ...a 0  f n k   1. k k 1 1 2   P ẠT
Khi đó thì ta có n  1  a a a k k ... 1 1 1 2 nn   n  Từ đấy ta có  a a ...af
k f n  1 fk  1, k k1 1 
tức là bằng số chữ số 2       2  2   2 
trong biểu diễn nhị phân của số . n
Nếu n là số lẻ thì bằng cách làm tương tự ta cũng được kết quả tương tự.
Vậy theo nguyên lý quy nạp ta suy ra f n là số chữ số trong biểu diễn nhị phân của . n
Ta đó ta suy ra nếu f n  2019 thì biểu diễn của n trong hệ nhị phân chứa đúng 2019 chữ số.
Vậy từ đó ta suy ra: 2018  n  2019 2 2 .
Nhận xét. Đây là một bài toán khá hay, với tư tưởng giải là đưa về hệ nhị phân. Bằng cách
biểu thị bình thường thì ta không thể tìm ra được tính chất của dãy, bởi điều kiện nó xen
kẽ với tính chẵn lẽ, rất khó chịu và phức tạp. Mà chỉ bằng cách đưa về hệ nhị phân ta đã
20 | Tạp chí và tư liệu toán học
Phương trình hàm trên tập rời rạc
nắm được quy luật của dãy số mà tác giá đã ẩn đi trong bài toán, và từ đó ta nảy ra ý
tưởng giải và chỉ việc đi triển khai, cuối cùng ta thu được kết quả của bài toán.
Câu 10. Tìm tất cả các hàm số * f  * :
thỏa mãn các điều kiện sau:
f 1  1, f 3  3 
f 2n  f n 
với mọi số nguyên dương n .
f 4n  1  2 f 2n  1   f n
f 4n 3  3 f 2n1  2 f nIMO 1988 Lời giải
Giả sử f là hàm số thỏa mãn điều kiện bài toán.
Một số nguyên dương k chỉ có thể có một trong bốn dạng sau:
k  4n, k  4n  1, k  4n  2, k  4n  3; k,n ÁN O
Do đó, từ giả thiết của bài toán, hàm số f được xác định một cách duy nhất. Ta sẽ sử
dụng biểu diễn cơ số 2 để tìm biểu diễn của hàm số f .
IC TP Ta có các nhận xét sơ bộ như sau: YM f 1 f 1 1 1 , f 10 f 2 1 01 2  
    2  2       2 OL f 11 f 3 3 11 , f 100 f 4 1 001 ,... 2  
    2  2       2 C Ụ
Từ đấy ta thấy được quy luật như sau: PH
Quy luật. Biểu diễn của f n trong hệ cơ số 2 chính là cách viết ngược lại của biểu diễn f a a a a a a a a k k ...   ... k k . 1 1 0 2   0 1 1  NH
của n trong hệ cơ số 2 tức là   I 2
Bây giờ ta sẽ chứng minh dự đoán này bằng quy nạp như sau. CH
Ch ứng minh.
Với n  1, 2, 3, 4 thì hiển nhiên đúng, do ta đã thử kiểm tra ở trên.
Giả sử tính chất đã đúng cho với mọi k  .
n Ta sẽ chứng minh tinh chất cũng đúng với . n
Trường hợp 1. Nếu n  2m thì theo giả thiết ta có f m  f n. Vì n  2m nên nếu m được
biễu diễn trong hệ cơ số 2 dưới dạng m  a a a a n a a a a k k ... 0 . k k ... 1 1 0  thì  1 1 0  2 2
Mà theo giả thiết quy nạp thì ta có: f a a a a f n f m f a a a a a a a a a a a a k k ... 0    k k ...  ...   k k 0 ... 1 1 0  2     
 1 1 02  0 1 1   2 0 1 k1 k 2
Từ đây, trong trường hợp này, tính chất được chứng minh.
Trường hợp 2. Nếu n  4m  1 với m  a a a a n a a a a k k ... 01 k k ... 1 1 0  thì  1 1 0  và 2 2
2m  1  a a a a k k ... 1 . 1 1 0  2
Mà theo giả thiết quy nạp thì ta có:
Chinh phục olympic toán| 21
Bồi dưỡng học sinh giỏi
f a a ...a a 01  f n f 4m  1  2 f 2m  1  f m k k1 1 0 2           2 f a a a a f a a a a k k ... 1   k k ... 1 1 0 2   1 1 02
 21a a ...a aa a a a k k ... 0 1 1   2 0 1 k1 k 2
 1a a ...a a a a a a k k 0  ... 0 1 1   2 0 1 k1 k 2
 10...0  a a ...a a 0  a a ...a a 2 0 1 k1 k   2 0 1 k1 k 2
 10...0  a a ...a a
  a a ...a a k k . 0 1 1  k k 10 2 0 1 1 2 2
Từ đây, trong trường hợp này, tính chất được chứng minh.
Trường hợp 3. Nếu n  4m  3 với m  a a a a n a a a a k k ... 11 k k ... 1 1 0  thì  1 1 0  và 2 2
2m  1  a a a a k k ... 1 . 1 1 0  2
Mà theo giả thiết quy nạp thì ta có: f a a a a f n f m f m f m k k ... 11   4  3  3 2  1   2 1 1 0 2          C Ọ
f 2m  1  2 f 2m  1  2 f m H
 1a a ...a aa a a a a a a a k k 1 ... k k0  ... k k0 0 1 1   2 0 1 1   2 0 1 1 2 ÁN
 1a a ...a ak k 10...0 0 1 1    2 2  U TO 11a a ...a a k k . 0 1 1 2 Ệ LI
Từ đây, trong trường hợp này, tính chất được chứng minh.
Vậy theo nguyên lý quy nạp thì quy luật của chúng ta đã được chứng minh. TƯ
Vậy tất cả các hàm f n thỏa mãn đề bài là: VÀ Í f a a a a a a a a k k ...   ... k k , 1 1 0  2   0 1 1 2 CH
trong đó n  a a a a k k ... 1
1 0  là biễu diễn của số n trong hệ cơ số 2. 2 P Ạ
Nhận xét. Cũng với ý tưởng tương tự như ở bài toán 9 là sử dụng hệ nhị phân để tìm ra T
quy luật của dãy số. Nhưng ở bài toán 10 thì khó hơn ở các bài trước rất nhiều, do điều
kiện của đề bài khá nhiều, làm ta khá hoang mang, và khá nhiều trường hợp cần xét của n
nên lời giải có vẻ phức tạp. Ở bài này, các bạn cần có một cách nhìn tổng quan để khám
phá ra quy luật và kiểm nghiệm sự chính xác của nó, và từ đó đi chứng minh phỏng đoán
đó bằng phương pháp quy nạp, bên cạnh đó cũng cần sự biến đổi điêu luyện và thật chính
xác thì mới thu được kết quả của bài toán.
22 | Tạp chí và tư liệu toán học
Phương trình hàm trên tập rời rạc Câu 11. Cho hàm số   f : 
thỏa mãn đồng thời các điều kiện:
f n là ước của 2018 n với mọi n
f a. f b  f c với mọi  a,b,c  và 2  2  2 a b c
a) Chứng minh rằng nếu n lẻ hoặc n 4 thì f n  1
b) Gọi A là tập hợp giá trị có thể có của f 2  f 2018 . Tính A Lời giải
n n  
a    a  2018 2 2 2 2 1 1 1 1
a) Nếu n  1mod 2 ta chọn a,b,c  n, ,
  f n| f    |   2 2   2   2   2018 2   n  1    2 2018 2018 2018 n  1 
Ta có f n|n
f n|gcdn ,     gcd n,
  1  f n   2  1       2  2018
Nếu n  0 mod 4 2 2 2 2
ta chọn a, b,c  4k, 4k  1, 4k  1  f 4k| f 4k  1  | 4k  1 ÁN 2018 f k kf k  2018 2 4 | 4 4 |gcd 4k , 4k  2018 1 1 O Ta có           2018 ff  0 1 2018 2 2 2 2 , 2 ,..., 2 IC T
b) Theo câu a, ta có       P
f 2018  f  2 1009  1  2 1009  2018  1 YM
Với a b c   2 , ,
2018,1018080,1009  1 ta có   f 2018 2018 2018 OL  C 2018 2 2018 0 1 2018 Ụ  f 2018 gcd
2018,1009 1  2  f 20182 ,2 ,...,2  PH  2 a  2 b  2 c *
Với a, b,c thoả 
gcda,b,c  1 NH I
 Nếu a,b lẻ, c chẵn. CH 2 2 2 Do 2 a  2 b  2
c  2a  2b  2c nên 1  f 2c  f 2af 2b . Mà   f : 
nên f 2a  f 2b  f 2c  1.
a c  1mod2  Nếu  . b   0mod2
Khi đó: f 2c  f 2af 2b  f 2a ,a,c  1 lẻ thỏa mãn (*).
Do đó f c
f a     b  2 a  2 b  2 2 2 * * :
c ,a,c  1. 2 2018 1018080 1009  1
Ta có f 2018 thỏa * * với bộ a,b,c     , ,
 mà f 2 không thỏa  2 2 2 
* * do a,c  1 nên f 2  f 2018
Dễ dàng chứng minh được 2a  2b  2c  2d với a, b  c, d bằng cách chia cả hai vế cho 
2 với   mina,b,c,  d nên T  2 C2019 .
Chinh phục olympic toán| 23
Bồi dưỡng học sinh giỏi
Câu 12. Có tồn tại hàm số f : S S thỏa mãn điều kiện
f af b  f  2 2
a b ,a,bS,a b không, trong đó S  * \  1 ? Lời giải
Giả sử tồn tại hàm số thỏa mãn yêu cầu bài toán. Ta sẽ tìm tính chất đặc biệt của hàm số f
Xét hai biểu thức sau f af df c , f af bf c .
Ta có         2 2      4 4 2 f a f d f c f a d f c f a d c  và
          2 2    2 4 4 f a f b f c f a f b c f a b c  .
Ta sẽ chọn c, d sao cho 4 4 2  2 4 4  2 4  4 2  2  2 a d c a b c a d b c ad b c 1.
Với c, d thỏa mãn (1) thì f  4 4 2
a d c   f  2 4 4
a b c   f af df c  f af bf c  f b  f d 2
Tất nhiên ta cần chọn d b c thỏa mãn 1 chẳng hạn d  2 b c  2 , ab . C
Nên từ 2 ta được f b  f  2
b ,bS (3). Ọ H
Từ tính chất 3 ta có 2 2 ÁN
f af b  f a b ,a,bS,a b f af b  f ab,a,b S,a b 4
Sử dụng tính chất 3 , 4 ta được f
  f  2   f    f  2 16 4 4 2   f 2 . U TOỆ
Mặt khác f 16  f 2.8  f 2 f 8 . LI
Từ hai đẳng thức trên ta được f 2 f 8  f 2  f 8  1 (vô lí.) TƯ
Vậy không tồn tại hàm số f thỏa mãn yêu cầu bài toán. VÀ Í
Câu 13. Tìm tất cả các hàm số * f  * : thỏa mãn điều kiện CH 2 P
n    f nf f n  2
n n n * 1 , . ẠT Lời giải
Với dạng toán mà giả thiết về bất đẳng thức thì việc dự đoán được nghiệm đóng vai trò
quan trọng trong việc định hướng lời giải. Việc dự đoán nghiệm thường ta thông qua việc
tính các giá như f 1 , f 2 , f 3 ,...
Thay n  1 vào phương trình ban đầu ta được 0  f 1 f f 1  2  f 1  1 .
Thay n  2 vào phương trình ban đầu ta được 1  f 2 f f 2  6 .
Từ bất đẳng thức này suy ra f 2  1 , nếu f 2  3 .
Từ điều kiện ban đầu ta thay n bởi f n thu được
f n 2  f f n f f f n  f n2  f n n * 1 , (1). 2 2
Từ (1), thay n  2 ta được  f 2  1  f f 2 f f f 2  f 2  f 2
24 | Tạp chí và tư liệu toán học
Phương trình hàm trên tập rời rạc
   f    2 4 2
1  f f 2 f f f 2  f f 2  2
Điều này mâu thuẫn với 1  f 2 f f 2  6 . Do đó f 2  2 .
Do đó ta dự đoán f n  n n  * ,
. Ta sẽ chứng minh bằng phương pháp quy nạp.
Giả sử f 1  1, f 2  2,..., f n  1  n  1 . Ta chứng minh f n  n
Với mỗi số nguyên dương n , đặt f n  m . Ta xét các trường hợp sau:  2
Nếu m n   f nf f n  mf m  2 1
m  n  1 vô lí.
 Nếu f n  m n   n   f f n  f nf f n  2 1 1
n n f f n  n
f f f n  f f n  f f n f f f n   f f n2  2 . n , 2 2
Kết hợp với f f f n f f n   f n  1 ta được 2
n   f n  1  f n  n  1 vô lí.
Do đó f n  n . Vậy f n  n n  * , . ÁN O
Câu 14. Tìm tất cả hàm số f :
 thỏa mãn đồng thời hai điều kiện sau: IC T
i) x f y f x  y f x f y với mọi x, y  ; P
 f x  f y  YM
ii) Tập hợp I  
,x, y  ,x y là một khoảng  x   y  OL C Lời giải
Phân tích. Điểm mấu chốt để giải bài toán chính là hiểu và tận dụng được giả thiết ii). PH
Tập hợp I là một khoảng có nghĩa là nếu a, b I , a b thì a, b  I . NH
Từ điều này ta suy ra tính chất quan trọng của tập hợp I là nếu có số thực a I thì hoặc I
x a,x I hoặc là x a,x I . CH
Khi đó ta có lời giải sau.
Để ý rằng nếu f x  f y   x y thì x y ( do i) ). Do đó 1  I .
Giả sử t I . Khi đó t  1 và tồn tại x, y  , x y sao cho f x  f y  t x y .
x y nên f y f x  f x f y và
f y f x  f x f y y x 1  .
y f x x f y  y x tx y     1  t 1 1 Do đó
I . Mà I là một khoảng nên mọi phần từ x nằm giữa t và cũng thuộc I 1  t 1  t 1 Nếu t  1 thì
 0 (mâu thuẫn vì khi đó ta lại có 1 I ) 1  t 1
Do đó t  1 . Tương tự ta cũng có
 1, tức là t  0 . 1  t
Chinh phục olympic toán| 25
Bồi dưỡng học sinh giỏi 1
Như vậy ta đã chứng minh được rằng nếu t I thì
I t  0 . 1  t 1 Mà khi t  0 thì
 0 nên ta có mâu thuẫn. 1  t
Như vậy I phải là tập rỗng mà rõ ràng I   , chẳng hạn  f 1  f 0 I .
Do đó không tồn tại hàm số thỏa mãn yêu cầu bài toán.
Ngoài ra ta cũng có thể giải bài toán này như sau. f x f y
Đặt F x, y     
với mọi x, y  , x y . x y
Tương tự cách đầu, ta có F x, y  1,x, y  , x  y
Nếu tồn tại x  y sao cho F x, y  0 thì F y f x , x f x  1 (vô lý).
Do đó F x, y0; 
1 ,x, y  , x y *
Suy ra f là đơn ánh. Do đó với mọi x, y  , x y ta có C Ọ
f x f y  f y f y  f y f x  f y f y  x y H
Hay F x f y , y f y  F y f x , y f yF x, y  1 ÁN
Từ đây, kết hợp với * và ii) ta suy ra F x, y  1,x, y  , x y U TO
Suy ra f đơn điệu tăng trên . Ệ LI
 Nếu x y thì f x  f y  x y hay y f x  x f y . TƯ
Suy ra f y f x  f x f y , kết hợp với i), ta được x y .
 Tương tự nếu x y ta cũng có mâu thuẫn. VÀ Í
Vậy không tồn tại hàm số thỏa mãn yêu cầu bài toán. CH P * * 2 2 2 * Ạ
Câu 15. Tìm các hàm số f : 
thỏa mãn f m  f n m n ,m,n T Lời giải
Giả sử tồn tại hàm f thỏa mãn yêu cầu bài toán Ta có 2
f 1  f 1 4  f 1  1 2
Thay m  1  1  f n 1  n ,n
f p  1  1  p
Thay m  1, n p  1p P  1  f p  1 2   p  
f p  1  1   2 p
Nếu p P f p    2 : 1 p  1 . 2 2 Thay m p n   2 1, 1
f p  1  1  | p  1  1 2 2 Suy ra  2 p     2 1
1 p  2p  2
26 | Tạp chí và tư liệu toán học
Phương trình hàm trên tập rời rạc Có 2 p   2
1 p  2p  2 nên có mâu thuẫn
Vậy f p  1  p  1,p P 2 2 2
Thay m p  1  p  1  f n p  1    n 2 2 2
Suy ra n  p  1   0modp  1  f n 2 2
Mà  f n  n  0 modp  1  f n 2 2
Suy ra  f n  n  0 modp  1  f n 2 2
Nếu f n  n , khi đó ta có  f n  n  f (n)  p  1 ,p P * 2
Mà tập các số nguyên tố là vô hạn nên f n   p  1   . 2
Mà  f n  n cố định với n xác định. ÁN
Do đó chỉ cần chọn p nguyên tố đủ lớn ta có mâu thuẫn * . O
Vậy f n  n n  * , (thử lại thỏa mãn) IC T P
Câu 16. Cho hàm f x, y thỏa mãn các điều kiện: YM
f 0, y  y  1; f x  1,0  f x,1 OL C
f x  1,y  1  f x, f x  1,y Ụ
Với mọi số nguyên không âm x, y . Tìm f 4,1981 PH Lời giải NH
f 1,n f 0, f 1,n  1   1  f 1,n  1 I Ta có    
f 1,n n f 1,0  n f 0,1  n  2 CH Do đó      
Ta lại có f 2, n  f 1, f 2,n  1  f 2,n  1  2
Do đó f 2, n  2n f 2,0  2n f 1,1  2n  3
Bây giờ f 3, n  f 2, f 3,n  1  2 f 3,n  1  3 Đặt u  2u
u f 3,0  3  f 2,1  3  0 n n1 và 0     Do vậy n3 u f n n   n   3 2 3, 2  3
f 4,nf 3, f 4,n 1
f 4,n1     3 2  3 
Ta có  f 4,0  f 3,1  4 2  3  13  f 4,2  224 2   3
Bằng qui nạp ta chứng minh được f n  22..24 4, 2  3
Trong đó số mũ chứa n  2 chữ số 2. Từ đó f    22.24 4, 1981 2
 3 với số mũ chứa 1983 chữ số 2.
Chinh phục olympic toán| 27
Bồi dưỡng học sinh giỏi Câu 17. Cho hàm   f : 
thỏa mãn các điều kiện sau: iii) f n 1 f n    ;n
iv) f f n 
 3n,nZ .
Hãy tính f 2003. Lời giải
Từ i ,ii  f 1  f f 1  3  f 1  2
Ta có f 2  f f 1  3.1  3
f 3  f f 3  3.2 f
  f f     2 2.3 3 3.3 3 .............
Suy ra f 2.3n n1  3 , n ; f 3n n    
 2.3 ;nZ n1 n n1 n1 n1 n1 n2 C
Nên có f 3   f f 2.3   2.3 và f 2.3   f f 3   3.3  3 Ọ
Do đó khẳng định đúng với mọi n H
Ta có 3n  1 số nguyên m nằm giữa 3n và 2. 3n và do giả thiết if n  1  f n nên có ÁN
3n 1 số nguyên m nằm giữa 3n f  và 2.3n f  suy ra 0   3n m
 3n    2.3n f m  3n . U TO 2.3n   3n   3 3n f m f f m  Ệ
Do giả thiết ii suy ra       m LI
Vậy 2.3n    33n f m
m với 0   3n m TƯ Suy ra n   6   f     6 2003 2.3 545 2003 3 3  545  3822 . VÀ Í
Câu 18. Cho f n là hàm số xác định với mọi  * n
và lấy giá tị không âm thỏa mãn CH P tính chất: Ạ * T
 n,m  : f m n  f m  f n lấy giá trị 0 hoặc 1
f 2  0 và f 3  0 .
f 9999  3333 . Tính f 2000 . Lời giải
f m n  f m  f n lấy giá trị 0 hoặc 1 nên ta suy ra f m n  f m  f n
f 2  2 f 1  f 1  0  f 3  1
Ta có f 6  f 3  f 3  2
f 9  f 6  f 3  3 .................
f 9999  f 9996  f 3  3333
28 | Tạp chí và tư liệu toán học
Phương trình hàm trên tập rời rạc
Vì giả thiết cho f 9999  3333 nên ta có dấu “=” ở các bất đẳng thức trên xảy ra, tức là
f 3n  n,n  1,2,...,3333  f 1998  666, f 2001  667
Mặt khác nếu a a b  * ,
a b f a  f b  f a b  f b .
 666  f 2000  667  f 2000  666 hoặc 667
Giả sử f 2000  667  f 4000  1334  f 6000  1334  667  2001 .
f 6000  2000 , mâu thuẫn. Vậy f 2000  666 .
Câu 19. Cho f , g là các hàm xác định trên thỏa mãn điều kiện
f x y  f x y  2 f x.g y ,x, y
Chứng minh rằng nếu f x  0 và f x  1,x  thì g y   a  1 0 Lời giải ÁN
Ta dùng phương pháp phản chứng O
Giả sử lại một điểm y
: gy   a  1 0 0 IC T Ta lấy x : f x 0 x k k 0,1, 2.... 0
 0  và xây dựng dãy   như sau: P x y f x y f x y k ,khi 0   k 0      k 0  YM xk1 
x y ,khi f x y f x y k 0   k 0     k 0  OL  C Ụ
Theo giả thiết ta có 2 f x
f x y f x y f x y f x y k1   k 0  k 0  k 0  k 0 PH  2 f x g y a f x k     2 0  k
NH I Nên f x a f x
a 1; k 1, 2, 3... k1   k với   CH k Do f x  đó ta có:  a f x k   . 0 
Nhưng vì f x   0 a k a f x 1 f x 0
và  1 nên có thể chọn k sao cho    0 dó đó  k  1
Mâu thuẫn với giả thiết.
Vậy g y  1,y R
Câu 20. Cho hàm số f :  thỏa 2 điều kiện
iii) f x  1  x;x
iv) f x y  f x. f y ;x, y
Chứng minh rằng không thể tồn tại hai số a; b
f a. f b  0 Lời giải
Ta sẽ chứng minh f x  0,x
Thật vậy! Với x  1 thì theo điều kiện i ta có ngay f x  0
Chinh phục olympic toán| 29
Bồi dưỡng học sinh giỏi
Với x  1 , trước hết ta sẽ chứng minh bất đẳng thức: 2n
f x   x   f
,x  ,n   n  1   2 
Với n  0 thì bất đẳng thức đúng! k   x 2
Giả sử 1 đúng với n k  0 tức f x   f k  2   2  k k k  1  x 2  x
x    x 2 .2  x 2 Ta có ff   f    f
tức 1 đúng với n k  1 k   k1 k1    k1   k1    2   2 2    2   2 
Theo nguyên lý quy nạp toán học bất đẳng thức 1 đúng. xx
Bây giờ chọn n đủ lớn để  2n x
, x  tùy ý, khi đó  1  fnn  0 2  2  2n   x  C Do đó f   
tức f x  0,x n  0 Ọ   2  H
Như vậy không thể tồn tại hai số a; b
f a. f b  0 . ÁN 2003
Câu 21. Cho f x, y 
cos 2x y  acosx y   với a,  . U TO 2 Ệ 2 2 LI
Chứng minh rằng min  f x, y  max f x, y  2003. TƯ Lời giải   f 0,0  f ;  2003 VÀ Ta có       2 Í  2 2  2 CH      2003 2 2003
Nên max f x, y  max  f 0,0 , f  ,  
x,y   max f x,y  P   2 2  2 2 ẠT     2003     2003 Ta lại có f ;    .
a sin , f  ;     . a sin       4 4  2  4 4  2         2003 Nên f ;  f  ;       .  4 4   4 4  2          2003
Suy ra min f x, y  min  f ; , f  ;       
x,y    4 4   4 4  2  
f x y2 2003 min , . 2 2 2
Do đó min  f x, y  max f x, y  2003.
30 | Tạp chí và tư liệu toán học
Phương trình hàm trên tập rời rạc 2 x 1
Câu 22. Cho hàm số f x   ,x  0. 2x
Giả sử f x  x
f x f f
x n * ,x   0 n n 1  0 và    . f x n   1 Chứng minh n   , x   1  ,0,1   f  x 1 2n n 1  x  1  f  x 1   Lời giải 1 2n 2n 1 2n 2n Đặt p x x x   q x x x x y n 1   1 , ,   n    1  1       và       2    2   
p x  2p x  2q x n1   n   n     Ta có: q x p x q x x y n 2 n n , ,   1         x p x f x x , x 0 0   0        ÁN  1 q x 0   O  p x k   2 IC T    1 2 2 P p x q x k   p x q x p x k      k   k1  
Giả sử f x    kf x    k1   k q x p x p x q x q x k  
2 k   k   k1   k   YM 2. q x k   OL C p x f x n n x n ,  ,  0 Ụ Do đó     q x n   PH
Ta có n  , x  1, 0, 1 thì có: n n n1 2 2 2 2n     1  NH
x  1 x 1
x  1 x 1 I f x n        f x  2n 2n   n 1 2 2n 1  n1     CH
x  1 x 1
x  1 x 1      n n x 12 x 12 2    n n  2 2  
2x  1 x  1  1 n n 1 n n 1 1   1   x n 1  2  x  12 1 x  1   2  x  12 1  x  1 2 f    x  1 
Câu 23. Cho hàm số * f  *  * :
là hàm số thỏa mãn đồng thời các điều kiện sau:
i) f 1,1  2
ii) f m n  f m n  m m n  * 1, , , ,
iii) f m n    f m n  n m n  * , 1 , , ,
Tìm tất cả các cặp số  p, q sao cho f p,q  2019.
Doãn Quang Tiến Lời giải
Áp dụng điều kiện ii) ta có:
Chinh phục olympic toán| 31
Bồi dưỡng học sinh giỏi f p qp p 1
,  f p  1,q  p  1  f p  2,q  p  2  p  1  ...  f 1,q     2 p p 1 q q 1 p p 1
f 1,q  1 q  1      ...  f 1,1        
 2019, áp dụng điều kiện 2 2 2 iii)
Từ điều kiện i) ta có f 1,1  4 nên từ đẳng thức trên ta suy ra:  q q p p q q p p f 1,1  1  1  1  1    2    2019 2 2 2 2
pp  1 q q  1  
 2017  p qp q  1  2.2017 2 2
Vì 2017 là số nguyên tố và  p q   p q  1 nên có thể xảy ra các trường hợp sau:
Trường hợp 1. p q  1 và p q  1  4034 thì từ đó ta được: p  2018, q  2017.
Trường hợp 2. p q  2 và p q  1  2017 thì từ đó ta được: p  1010, q  1008. C p,q  Ọ
Vậy tất cả các cặp số  p, q sao cho  p, q  2019 là: 
 2018,2017,1010,1008. H ÁN
Câu 24. Tìm tất cả các hàm số f :
 thỏa mãn các điều kiện sau:
i)f x  2 0 x ,n U TO
ii) f x  f y chia hết cho x y với mọi x, y  , x y Ệ LI Lời giải
Ta sẽ phải xét hai trường hợp dưới đây.
Trường hợp 1. f là hàm số hằng. VÀ Í
Giả sử f x  c  const,c là hằng số thuộc . CH
Cho x  0 thì từ i) ta suy ra: 0  f 0  0  f 0  0. Vậy f x  0,x  . P Ạ
Trường hợp 2. f không là hàm số hằng. T
f 0  0 nên khi cho y  0 thì từ điều kiện ii) ta được: x f x ,x  \  0 .
Với x  1 thì từ điều kiện i) ta suy ra: f 1  0 hoặc f 1  1.
Khả năng 1. f 1  0.
Cho y  1 thì từ điều kiện ii) ta suy ra: x  1 f x ,x  \  1 .
Mà ta đã có x f x ,x  \ 
0 và x,x  1  1
Nên x x   f x  x x    2 1 1
x x f x      2 f x x
f x  2 x x Mặt khác ta có    x f x 2 
 f x  x
Ta loại trường hợp    2 f x
x f 1  1  0.
32 | Tạp chí và tư liệu toán học
Phương trình hàm trên tập rời rạc
Vậy trong khả năng 1 này chỉ có hàm số f x  2
x x,x  thỏa mãn yêu cầu bài toán.
Khả năng 2. f 1  1.
Khi x f x ,x  \ 
0 và x  1 f x ,x  \ 
1 thì ta lấy k tùy ý sao cho f k  mk, với m tùy ý
Từ điều kiện ii) có k  1 mk  1  k  1 mk  1  mk m  m  1  k  1 m  1
Mà từ điều kiện i) ta được    2   2 f k k
mk k m k
 k  1 m  1 Mặt khác ta lại có 
k  1  m  1  k   m m   1  0
Do đó m  1  0 nên suy ra: m  1 hoặc m k.
Vậy từ đó ta được f x  2
x ,x  hoặc f x  x,x  .
Thử lại thì thấy các hàm số này thỏa mãn yêu cầu bài toán. ÁN
Vậy tất cả các hàm số thỏa mãn đề bài là O f x 
f x  x f x  2
x f x  2 0, , ,
x x,x  . IC T P
Câu 25. Tìm tất cả các hàm số * f  * :
*  x x    mà tập    0 thỏa mãn: YM   f xy
f x f y  xyf xy    x y  * 2 , , 1 OL
f x y    C Ụ Lời giải PH
Ta sẽ giải bài toán này thông qua ba bước sau đây.
Bước 1. Ta sẽ chứng minh f 1  1.
NH I Thật vậy, cho y  1 vào 1 và đặt f 1  a thì ta được CH   f x f x
f x a  2xf x     f x
f x 1    1    2 
1 2xf x    a 1 a 1 1 1
Từ đó ta suy ra f     f    4 2 , 3  , f 4  * 4a 4 5 5  4a 7  5a  2  4a a 4 f 4
Mặt khác, ta cho x y  2 vào 1 thì ta được 2 f 2  8 f 4    f    1 4 1 1
Mà từ * ta suy ra 2.  8.
 1  a  1  f 1  1. 4 7  5a  2 4a
Từ đây ta xong bước 1. f x
Bước 2. Ta sẽ chứng minh f x n     , n 1, 3 2 n  2nx      f x  1
Ta sẽ dùng phương pháp quy nạp để chứng minh khẳng định này.
Chinh phục olympic toán| 33
Bồi dưỡng học sinh giỏi
Từ 2 suy ra 3 đúng khi n  1. Giả sử đúng đến n k. f x k
Thì ta có f x k  1    
1  2x k f x k  1 f x
 2k 2kxf x1 f x    
1  2 x k f x
 k 12  2k 1xf x1   1 2
 k  2kx   f x  1  
Từ đây theo nguyên lý quy nạp ta suy ra điều phải chứng minh.
Vậy khẳng định ở bước 2 được chứng minh. f 1 1 1
Vậy từ đó ta suy ra f n  1    
hay f n  ,n  1, 2 n 2nf 1    1 n  12 2 n  1  1
Bước 3. Ta sẽ chứng minh: f  2 n  ,n  1,   4 2   C  n   1  Ọ    n  H  1  f ÁN    1   n
Thật vậy, trong 3 ta thay  1 x thì được f n   n   
n   2n 2  1  f    1 U TO  n  Ệ LI 1 1 Tiếp theo, ta thay  1 y
vào 1 thì được f x    f  2  x    x   1  f x  TƯ    x  1 1 1 VÀ 2 Í Vậy f n    f  2   n  2     n   1   1  f n    f   CH  n   n  P 1  1  Ạ
Mà ta có: f n  nên suy ra: f    2 n . T 2 nn
Nên từ đây bước 3 được chứng minh hoàn toàn.
Thử lại thì thấy hàm số này thỏa mãn yêu cầu bài toán. 1
Vậy tất cả các hàm số thỏa mãn yêu cầu bài toán là f n  ,n * . 2  n
34 | Tạp chí và tư liệu toán học
Phương trình hàm trên tập rời rạc Câu 26. Cho hàm  f :
 là một hàm số thỏa mãn với mọi n  1 thì có một số  n
nguyên tố p là ước của n sao cho: f n  f p  f   1 và  p
f  2018   f  2019   f  2020 3 5 7   2017.
Hãy tính giá trị của biểu thức G f  2018   f  2019   f  2020 2018 2019 2020 
Doãn Quang Tiến Lời giải f 1 Thay n p 1 f p f 1 f p f p
vào   thì ta được               2 Thay n bởi n
p thì ta được f n
p f n  1
p   f p
Bằng phương pháp quy nạp thì ta chứng minh được. f n
p   2  n     f 1 * ÁN  2  O
Thật vậy, thì * đúng với n  1. IC T
Giả sử * đúng với n k  1. P
Ta sẽ chứng minh * cũng đúng với n k  1 hay ta có YM k f k k k 2 1 2 1 1         OL
f p   f p   f p    f 1         f 1 C  2  2  2  Ụ
Từ đây ta suy ra * cũng đúng trong trường hợp n k  1. PH
Vậy theo nguyên lý quy nạp thì * đúng với mọi . n NH I Khi đó thì ta suy ra
f  2018   f  2019   f  2020 3 5 7   2017 CH
 2  2018  f 1  22019  f 1  2 2020     f 1        2017  2   2   2    6051 f     f     2 1 2017 1 . 2 3 f 1
Từ f     2 1 và f p   
suy ra: f p   1 .' 3 2 3  n  1
Khi f p   1 thì 1 được viết lại như sau f n  f    ,n 2 3  p  3  n
Cho k là số các thừa số nguyên tố của n, khi đó số lượng thừa số nguyên tố của   là  p  k  1.  1 1 2
Với k  1 thì ta chọn n  2 thì theo 2 ta được: f 2     . 3 3
Chinh phục olympic toán| 35
Bồi dưỡng học sinh giỏi
 Với k  2 thì ta chọn n  10  2.5 thì theo .2 ta được: . f    10   f  1  f 5 1 1 1 2       0  2 10    2  3 3 3 3 3
 Với k  3 thì ta chọn n  12  2.3.4 thì theo 2 ta được: f    12  1 f f 6 1  6       f
 1  1  f 2 1 1 1 1 1 1 3 
        2 12      2  3 3  3  3 3 3 3 3 3 3 3 3
Từ đó, bằng phương pháp quy nạp ta suy ra được:
f nk  
2 ,với k là số các thừa số nguyên tố của n ** 3 Mà lưu ý rằng    2
2018 2.1009, 2019 3.673, 2020 2 .5.101. Do đó suy ra: 2018 2018
có 4036 thừa số nguyên tố, 2019 2019
có 4038 thừa số nguyên tố, 2020 2020
có 8080 thừa số nguyên tố. C Ọ
Từ đấy theo công thức * * ta suy ra được: H G f  2018  f  2019  f
2020  4036  2 4038  2 8080        2  16148 2018 2019 2020 . 3 3 3 3 ÁN
Vậy từ đấy ta có kết quả của bài toán.
x  1, x, x  1, x  2 U TO
Nhận xét. Một điều thú vị là khi ta thay 2017, 2018, 2019, 2020 bằng   thì Ệ LI
ta được kết quả của f     2 1 không thay đổi. 3 TƯ
Câu 27. Tìm tất cả các hàm số * f  * : thỏa mãn: VÀ Í 3 f  2 m  2 n   2
f mf n  f m 2
f n m n * 2 , , CH P Lời giải ẠT
Giả sử tồn tại hàm f thỏa mãn yêu cầu bài toán.
Trường hợp 1. Nếu f là hàm số hằng.
Tức là f n  const c, với c là hằng số thì hiển nhiên thỏa mãn yêu cầu bài toán.
Trường hợp 2. Nếu f không là hàm số hằng.
Nếu tồn tại m n  * ,
sao cho f m  f n thì ta gọi a,b là hai số thỏa mãn:
f a  f b 
f m  f n m n * min , , 1
Giả sử f a  f b thì ta có 3 f b  2
f af b  f a 2 f b  3 2 2 f a
Suy ra     2  2       2  2 f b f a b f a f a
b   f b  f a  f b
Từ đó thì ta suy ra f a  f b  f a  f b  f  2 a  2
b   f b  f  2 a  2
b   f b 2
Rõ ràng thì ta thấy 2 mâu thuẫn với 1.
36 | Tạp chí và tư liệu toán học
Phương trình hàm trên tập rời rạc
Do đó chỉ có f là hàm số hằng thỏa mãn yêu cầu bài toán.
Vậy tất các hàm số thỏa mãn đề bài là: f n  const  c, với c là hằng số.
Câu 28. Giả sử f :
 là hàm liên tục và giảm sao cho với mọi  x, y  ta có
f x yf f x  f y  f y f x
Chứng minh rằng f f x  x Lời giải
Cho y x ta được: f 2x  f 2 f x  f 2 f x f x
Thay x bằng f x ta có f 2 f x  f 2 f f x  f 2 f f x  f f x
Trừ hai phương trình trên ta suy ra:
f 2 f f x  f 2x  f 2 f f x  f f x  f 2 f x f x ÁN
f f x   O Nếu
  x , vế trái của phương trình trên âm, do đó:
f f x  f f x  f x f x và f x  f f x  x f x
IC TP là điều mâu thuẫn. YM
Tương tự, ta cũng có điều mâu thuẫn xảy ra khi f f x  x OL
Vậy f f x  x , điều phải chứng minh. C Ụ PH
Câu 29. Cho song ánh f :
 . Chứng minh rằng tồn tại vô số bộ a,b,c với
a,b,c  thỏa mãn a b c và 2 f b  f a  f c NH I Lời giải CH
Ta xây dựng dãy an như sau:
Trong các số từ 0, 1, 2,..., m chọn số a
f a f i i  0; a m  1 sao cho  1   1   Chọn a a
f a f i ,i  0; a 2 1 sao cho  2   2 Chọn a a f a f i i a k ,  0; k k1 sao cho     k
Vậy ta có dãy a a  ...  a a
f a f a  ...  f a f a 1 2 k k1 và  1  2
k  k1 Trong đó a  
f a f j j  0; a i và  i    i
Vì f là song ánh nên f a
f a p, p * N k1   k Và c
để f c  f ap f a k1   k1 a a i k k i ,     1, 1 Mặt khác   f af i i a k ,   1, 1    n1
Chinh phục olympic toán| 37
Bồi dưỡng học sinh giỏi pa f a p k      k1  Nên c a    2 f af a f c k1 k 1 k
f c   f ap k1        
Do cách xây dựng, dãy a a,b,c
n là dãy vô hạn nên tồn tại vô số bộ 
 thỏa điều kiện đã nêu.
Câu 30. Có bao nhiêu hàm f : * 
* thoả mãn đồng thời các điều kiện sau
a) f 1  1 2
b) f nf n  2  9 f n  1  1997, n  * . Lời giải
Gọi D là tập hợp tất cả các hàm số f thoả mãn điều kiện bài toán.
Theo giả thiết b) ta có 2
f nf n  2   f n  1  1997 ; C Ọ  2
f n  1 f n  3   f n  2  1997 H 2 2
Suy ra f nf n  2   f n  1  f n  1 f (n  3   f n  2  1997 ÁN
f n  f n  2
f n  1  f n  3   n f n 1  f n 2 ,  *.   U TOỆ
f 1  f 3
f 2  f 4
f n  f n  2 LI Vì vậy ta có f    f    ...  f n 1  ... 2 3  TƯ
f 1  f 3 Đặt c
1 suy ra f n  2  cf n  1 f n,n *2 f 2 VÀ Í Ta chứng minh c
* . Thật vậy, nếu  p c
với p, q  và  p,q  1 thì từ 2 ta có CH q P
q f n f n  2  pf n  1 ,n Ạ        * T
Suy ra q f n  1 ,n  * hay 2
q f nf n  2 ,n * và n  2. 2 Vì
  f nf n  f n   2 1997 2 1 q .
Mà 1997 là số nguyên tố nên 2
q  1 hay q  1 suy ra c  * 2
Gọi f 2  a, do 1 ta có ac  1  f 3 suy ra ac  1  f 3  f 1 f 3   f 2  1997  ac   2
1 a  1997  ac a  1998
Ta được a 1998 , hay f 2 là một ước dương của 1998.
Ngược lại với mỗi ước dương a của 1998 ta xây dựng hàm f : *  * như sau
f 1  1; f 2  a 1998
f n    a bf n    f n n * 2 1 , ; trong đó b   *. a
38 | Tạp chí và tư liệu toán học
Phương trình hàm trên tập rời rạc
Ta chứng minh f thoả mãn điều kiện đề bài, nghĩa là f D . Thật vậy
f n  1 f n  3   f n  22  f n  1a bf n  2  f n  1   f n  22
 a bf n  1 f n  2   f n  12  f n  22
f n  2a bf n  1  f n  2  f n  12
f n  2 f n  f n  12 ,n * 2 2
Suy ra f n  1 f n  3   f n  2  f n  2 f n   f n  1 ...
f   f     f  2  f    f  2 3 1 2 3 2 2 2
Từ đó ta có f nf n  2   f n  1  f 3   f 2
 a bf    f    f  2  a ba   2 2 1 2 1 a ÁN
ab  1  1998  1  1997. O 2
Vậy ta được f nf n  1   f n  1  1997 hay f D
IC TP Ta có tương ứng, mỗi f D với một giá trị f 2 1998 là một song ánh giữa D và tập các ước dương của 1998 . YM
Do đó số phần tử của D là: D d    d 3 1998
2.3 .37  1 13  11 1  16. OL C
Vì vậy có tất cả 16 hàm số thoả mãn đề bài. Ụ PH
Câu 31. Tìm tất cả các hàm số * f  * : sao cho. NH a) f 2  2 I b) f  .
m n  f m. f n với mọi m n * ,
, UCLN m,n  1 CH
c) f m  f n m n * , , m n . Lời giải
Chọn n  1 , thay vào f  .
m n  f m. f n  f 1  1 .
Ta để ý rằng f 3. f 5  f 15  f 2. f 9  f 2. f 10  f 2. f 2 f 5 .
Vậy f 3  f 2. f 2  4 . Mà 2  f 2  f 3  4 nên f 3  3 .
Từ đó ta tính được f 4  4, f 5  5, f 6  6, f 7   7, f 8   8, f 9   9, f 10  10 .
Dự đoán f n  n với   * n .
Giả sử f k  k với k  * ,10  k n . Ta chứng minh điều khẳng định vẫn còn đúng với k n  1 .
Nếu k là số chẵn, ta xét hai trường hợp sau:   k
l    l * 2 2 1 ; , .
Chinh phục olympic toán| 39
Bồi dưỡng học sinh giỏi
Lúc này f k  f    l    f    f l     2 2 1 2 2 1
2 2l  1  k .   k    * 2 ; Lúc này f k
f    f   1  f   f  1            1 2 2 2 2 2 1 2 2 1 2 2  1  k  2 .
Mặt khác k  1  f k  
1  f k  f k  
1  f k  2  k  2 .
Do đó f k  k, f k  1  k  1 .
Nếu k là số lẻ thì k  1 là số chẵn, ta xét hai trường hợp sau:   k  
l   l * 1 2 2 1 ; , . Khi đó 
0  2  n,0  2l  1  n .
Theo giả thiết quy nạp f k    f    l    f    f l     1 2 2 1 2 2 1
2 2l  1  k  1 .
k  1  f k  1  f k  f k  1  k  1  f k  k .   k     * 1 2 ; . Lúc này   1  1  
f k      f     f  
   f   f      1 1 2 2 2 2 2 1 2 2 1 2 2  1  k  3 C Ọ
Mặt khác k  1  f k  1  f k  f k  1  f k  2  f k  3  k  3 . H
Do đó f k  k, f k  1  k  1, f k  2  k  2 . ÁN
Theo nguyên lí quy nạp f n  n với   * n . U TO Ệ
Câu 32. Tìm tất cả các hàm số f :  thỏa mãn LI
f m n  f mn  f mf n  1,m,n 1 TƯ Lời giải 2 VÀ
Thay m n  0 vào 1 ta được 2 f 0  f 0  1  f 0  1 Í
Thay m  1, n  1 vào 1 ta lại có f 1  f 1 f 1  1 CH P
Vậy f 1  0 . hoặc f 1  1 ẠT
 Xét f 1  1. Thay n  1 vào 1 ta có : f m  
1  f m  f m  1,m
Suy ra f (m  1)  1,m  hay f ( )
m  1,m  . Thử lại thỏa mãn.
 Xét f 1  0
Thay n  1 vào 1 ta được f m  
1  f m  1,m 2
Thay n  1 vào 1 ta lại được f m  1  f m  f 1 f m  1,m
Đặt a f 1 thì phương trình trên trở thành f m  
1  f m  af m  1,m 3
+ Với a  2 . Nếu a  1 thì dẫn tới f 1  1(trường hợp này đã giải ở trên)
Do đó ta xét a  1. 1  1 
Khi đó 3  f m   1   a   1 f m  ,m   2  a  2  a
40 | Tạp chí và tư liệu toán học
Phương trình hàm trên tập rời rạc f n 1 a  1 f n  1      1  ,n   2  a  2  a f n 1
a n1  1 f   1      1  ,n   2  a  2  a n1 1 a 1  f n      ,n 4 2  a n 1 1 b
Đặt b a  1 b  \ 0,  1 , f n      ,n . 1 b 1  m b   1 1 m b Thay vào 2 ta được   1,mm 1  1   1  m b b
 1  b ,m 1  b 1  b m 1   m b b
b  1,m 5 1 2
Thay m  2 vào 5 ta được 2
b   b  1  b  1 b  1  0  b  1 b  1 b n 1 ÁN 1 1 O
Từ đó suy ra f n      
,n , hay f n  0 khi n lẻ và f n  1 khi n chẵn. 2 Thử lại thỏa mãn.
IC TP + Với a2 thì f 12 . YM
Thay n  1 vào 1 ta được f m  1  f m  1,m 6 OL
Từ f 1  2 và 6 ta dùng phương pháp quy nạp toán học thì sẽ chứng minh được C Ụ
f n  n  1,n . PH
Thử lại thấy thỏa mãn. Vậy các hàm số thỏa mãn yêu cầu đề bài là n 1   1 NH 1 I
f n  1 , f n   
, f n  n  1 với n . 2 CH
Câu 33. Tìm tất cả các hàm số f :
 thỏa mãn f 0  2 và
f x f x  2y  f 2x  f 2y ,x, y  1 Lời giải
Thay x y  0 vào 1 ta được f f 0  2 f 0  f 2  4
Thay x  0 và y  1 ta được f f 2  2  f 2  f 4  6
Ta sẽ chứng minh bằng quy nạp rằng với x  thì f 2x  2x  2 2
Theo trên thì 2 đúng khi x  0 .
Giả sử 2 đúng tới x k k  , k  0 .
Thay x  0, y k vào 1 ta được f f 2k  f 0  f 2k  f 2k  1  2 k  1 2
Vậy 2 cũng đúng khi x k  1 , suy ra 2 đúng với x  , x  0.
Với x  , x  0, thay x, y  2x,x vào 1 ta được
Chinh phục olympic toán| 41
Bồi dưỡng học sinh giỏi
f 2x f 0  f 4x  f 2x ,x  ,x  0
f 2x 1  f 22x  f 2x,x ,x  0
 2x 1  2  22x  2  f 2x,x ,x  0
f 2x  2x  2,x ,x  0
f 2x  2x  2,xZ,x  0
f (2x)  2x  2,xZ
Dó đó (1) 1  f x f x  2y  2x  2y  4,x, y  3
Ta sẽ chứng minh nếu x là số nguyên lẻ thì f x cũng là số nguyên lẻ.
Thật vậy, nếu f x  2k , với k  , thay x x  2k (với x lẻ) và y k ta được :
f x  2k f x  2x  2k  2k  4,x ,x lẻ
f x  2x  2k  4,x ,x lẻ C
 2k  2x  2k  4,x ,x lẻ Ọ
Do đó 4k  2x  4, với mọi số nguyên lẻ x. H
Điều này vô lí vì 2x  4 không phải lúc nào cũng chia hết cho 4. ÁN
Như vậy nếu x lẻ thì f x lẻ. f x x f x  U TO
Từ đó nếu x lẻ thì x  2y lẻ, dẫn đến  2y lẻ, do đó  2y chẵn Ệ LI
Do đó f x f x  2y  x f x  2y  2 . TƯ
Kết hợp với 3 ta được x f x  2y  2  2x  2y  4  f x  2y  x  2y  2, x, y  , x lẻ
f x  x  2,x VÀ Í Thử lại thỏa mãn. CH P f :
f f n   f n  2n  3,n 1 Ạ
Câu 34. Tìm tất cả hàm số  sao cho   T Lời giải
Giả sử tồn tại hàm số f n thỏa mãn yêu cầu bài toán.
Cho n  0 , từ 1 có f f 0  f 0  3  0  f n  3 2
Nếu f 0  0 thì f f 0  f 0  0 mâu thuẫn 2 . Vậy f 0  0
Nếu f 0  2 thì từ 2 ta có
f f 0  1  f 2  f f 0  1  f 1  f f 2  2.2  3  f 2  6
f 6  f f 1  2.1 3  f 1  1 Suy ra f 6  , loại.
Như vậy f 0  2 . Tương tự cũng có f 0  3 .
42 | Tạp chí và tư liệu toán học
Phương trình hàm trên tập rời rạc
Do đó f 0  1 . Khi đó từ 2 ta có
f 1  f f 0  2  f 2  f f 1  2.1  3  f 1  3
Ta sẽ chứng minh hàm cần tìm là f n  n  1,n  bằng quy nạp toán học.
Thật vậy. Với n  0 thì f 0  1  0  1 .
Giả sử khẳng định đúng tới n k,(k  ). Tức là: f (k)  k  1
Với n k  1 ta có f k  1  f f k  2k  3  f k  2k  3  k  1  k  1  1
Vậy khẳng định đúng với n k  1
Theo nguyên lý quy nạp toán học, ta có f n  n  1,n  .
Thử lại hàm tìm được thỏa mãn yêu cầu bài toán. ÁN
Câu 35. Chứng minh rằng tồn tại duy nhất hàm số f : *  * thỏa mãn O
f m f n  n f m b ,m,n * b  i IC T Lời giải P
Giả sử tồn tại hàm số f n thỏa mãn yêu cầu bài toán. YM
Ta chứng minh f là đơn ánh. OL C
Thật vậy, giả sử f n   f n  ,n ,n  * 1 2 1 2 . Ụ
Từ i ta có f m f n   f m f n   n f m b  n f m b  n n 1 2 1 2 1 2 PH
Vậy f là đơn ánh.
NH I Với n * , ta có f f 1 f n nf f 1bn1 f bb f bf n1 CH
fn  1  b f n  f 1 , vì f là đơn ánh
f n  1  f n  f 1  b a , với a f 1  b .
Suy ra f n  f n  1  ...  f 2  f 1  f 1  b a,n  *
Từ đó: f n  b na f n  na b,n  * .
Lúc này f m f n  n f m b  f m na b  n  m ba b
 m na ba b n ma ba b  2 na n  2
a  1  a  1, vì nếu a  1 thì f n * khi n b .
Suy ra f n  n b,n  * b   .
Thử lại hàm vừa tìm được thỏa mãn yêu cầu đề.
Chinh phục olympic toán| 43
Bồi dưỡng học sinh giỏi
Câu 36. Hãy xác định tất cả hàm số * f  * : thỏa mãn đẳng thức:
f n  f n  1  f n  2. f n  3  a 1
Với a là số tự nhiên thỏa mãn a  1 là số nguyên tố Lời giải
Giả sử phương trình có nghiệm.
Thay n bởi n  1 lúc đó 1 trở thành f n  1  f n  2  f n  3 f n  4  a 2
Lấy 2  1 vế với vế ta được: f n  2  f n  f n  3 f n  4  f n  2 3
Thay n bởi 1 lúc này 3 trở thành: f 3  f 1  f 4 f 5  f 3
Thay n bởi 2 lúc này 3 trở thành: f 4  f 2  f 5 f 6  f 4
Thay n bởi 3 lúc này 3 trở thành: f 5  f 3  f 6 f 7  f 5
Thay n bởi 4 lúc này 3 trở thành: f 6  f 4  f 7. f 8  f 6
Từ đây ta đã nhận ra quy luật đặc biệt của bài toán này đó chính là nếu thay n bởi số lẻ thì C
f 3  f 1  f 4 . f 6 ... f 2n . f 2n  1  f 2n  1 4 Ọ
ta luôn biểu thị được:    
           H
Nếu thay n bởi các số chẵn ta sẽ được một đặc biệt khác
f 4  f 2  f 5 f 7... f 2n  1 f 2n  2  f 2n ÁN  5
 Nếu f 1  f 3 thì lúc này ta sẽ lậy tức có ngay f 2n  1  f 2n  1 lúc này sẽ có U TO
vô số số bé hơn f 1 mà f 1 là 1 số hữu hạn. Suy ra vô lí. Tức là f 3  f 1 . Ệ LI
Tương tự ta cũng sẽ có f 4  f 2 . TƯ
 Nếu f 1  f 3 và f 2  f 4 lúc này f 2n  1  f 2n  1 và f 2n  f 2n  2 .
Suy ra f 3  f 1 và f 4  f 2 sẽ có vô số ước số khác nhau (vô lí). VÀ Í
Từ đó chúng ta sẽ có 3 trường hợp CH
 f 3  f 1
 f 2n  1  f 2n 1
 f 1  f 2n 1 P
Trường hợp 1.    , lúc này  Ạ  f
 4  f 2  f
 2n  2  f 2n  f
 2  f 2n T
Thay bởi 1 vào phương trình 1 lúc này f 1  f 2  f 3. f 4  a
 f 3  f 1 Mà lại có 
f 1  f 2  f 2. f 1  a   f 1  1 f 2  1  a  1  f
 4  f 2
 f 1  2 
2 khi n  2k     f n f 2    a  
a khi n  2k  1
Mà lại có a  1 là số nguyên tố nên ta có   
 f 1  a      f na khi n 2k     f  2  2 
2 khi n  2k  1
 f 1  f 3
Trường hợp 2.
lúc này f 1  f 2n  1 với mọi  * n .  f
 2  f 4
44 | Tạp chí và tư liệu toán học
Phương trình hàm trên tập rời rạc
f 2n  2  f 2n 
Thay vào 5 ta được 
f 4  f 2  
f 1n1  f 2n 2 f 2n
Lúc này f 4  f 2 có vô số các ước số nguyên dương đôi một khác nhau, điều này
không thể xảy ra nên phải có f 1  1.
Thay n bởi 1 vào phương trình 1 thì ta sẽ có được f 1  f 2  f 3. f 4  a
Mà lại có f 2n  1  1
1 n  2k  1 
f 4  f 2  a  1  f 2n  2  f 2n  a  1  f n  
a  1n 2 x  n  2k  2 Với  * x
 f 2  f 4
Trường hợp 3. Nếu  .  f
 1  f 3 ÁN O
1 khi n  2k
Lập luận tương tự như trên f n  
a  1n 1 trong đó  * y . IC T y
khi n  2k  1 P  2
Câu 37. Tìm tất cả các hàm số * * f : 
thỏa mãn f n a
f n an t a k t    1.     YM
với f n f f f n a t
k 2t  1  a  1 t  
 ...   với , là số tự nhiên tùy ý thỏa mãn   . OL C tLời giải PH
Đặt f n  n k a a
n . Ta sẽ chứng minh n
0 với mọi số tự nhiên dương n bất kì Từ giả thiết ta suy ra NH I  an
t a k
n t a k
a  1 f n  an  t ak f n  1  1    n  1 CH a  1 a  1 a    1
Với n  2a  2  tk thì  
n t   a k n t  1  a  1 
k  2a  2  ak k
 2  k f n  n k  2 a  1
Như vậy với n  2a  2  tk f n   n k  1
Khi đó với n  2a  1  2kt k t thì f n  n k  1  2a  2kt  2k t  2a  2  tk lúc đó ta
cũng có f f f 2  f f n  k  1  f n  2k  1
Tiếp tục làm như thế cho đến lần thứ t  1 lúc đó ta sẽ có f
n n t  1 k  1  2a  2  tk t1       
Và cuối cùng f n  f f n f
n k  1  n t k  1 t t1   t1          
Lại có với n  t  1 k thì a  1 f n  f n  a  1 f n  n t k  1  a  1 f n t     
f n  an  t a  1 k  a  1 f n t     
nên bất đẳng thức trên cho ta
Chinh phục olympic toán| 45
Bồi dưỡng học sinh giỏi
a 1n k a an t a k n t k   n   a n   1
 1 97 2 n  2
tk n k 2t  1
Từ 1 ta có a  1 a a
k n t   a k a   
k t   a n  1  1    n 1  2 1 1 a  1 a  1 Mà từ 2 ta có  t
a  1 a a k t k   t a k a a t   a   a n  1  1  1  1 n n n 0 a  1
Từ đó ta sẽ có a n 0 .
Xét với n  t  1 k . Ta sẽ chứng minh phần này của bài toán bằng phương pháp quy nạp.
Giả sự bài toán đúng đến m n . Khi đó f
n t  1 k f
n t  2 k f
n t  3 k  ...  f n  2  n t1     t2     t3            
f n  f f                 n t k f n t k a n t k t a k a f n t k t 1 t 1 1 1 1 1 1                    
Theo giả thiết quy nạp f (n  (t  1)k)  n  (t  2)k C Ọ
Do đó f n  an at  1 k tk ak k an a t  2 k n  t  2 k n k H
Vậy f n  n k . Thử lại thấy thỏa mãn. ÁN
Câu 38. Cho hàm số f :  thỏa mãn: U TO
 f 2n1 f 2n1 f 2n1 f 2n1  31 2 f n Ệ  ,n LI  f
 2n  f n TƯ
Tìm n sao cho f n  2009 . Lời giải VÀ Í
Giả sử tồn tại hàm f thỏa mãn đề bài CH
Vì 31  2 f n là số nguyên dương lẻ f 2n  1  f 2n  1 là số nguyên dương lẻ P
f 2n  1  f 2n  2  f n Ạ T  f
 2n  1  f 2n  1  1
f 2n  1  f 2n  1  2 f n  1  
f 2n  1  f 2n  1  3  1  2 f n  f
 2n  1  f 2n  2
 f 2n  1  f 2n  2     2
  f 2n  1  2 3 f n  1  f
 2n  3 f n
Ta sẽ chứng minh f n  f n  1 (1)
Với n  1  f 1  f 0  2  f 0 
Giả sử f n  f n  1 đúng tới k (k N*)  f 0  f 1  f 2   <f k
 Nếu k chẵn. Đặt k  2mm *  f k  1  f 2m  1  f 2m  2  f 2m  f k
 Nếu k lẻ. Đặt k  2m  1m 
f k  1  f 2m  2  3 f m  1  3 f m  2  f 2m  2  f 2m  1  f k
Như vậy trong mọi trường hợp khẳng định (2) đúng
46 | Tạp chí và tư liệu toán học
Phương trình hàm trên tập rời rạc
Do đó f 0  3 f 0  f 0  0, f 1  2; f 2  3 f 1  6
f 3  f 2  2  8, f 13  f 12  2  9 f 3  2  74, f 27  3 f 13  2  224
f 53  9 f 13  2  668, f 107  3 f 53  2  2006, f 108  9 f 27  2016
f 107  2009  f 108
Do f n  2009  n 0;1; 2;...;10  7
Câu 39. Tìm tất cả các hàm số f :  thoả mãn: 1 f xy 1
f xz  f xf yz 1  , x  ,y,z . 3 3 9 Lời giải 2 1 1 1  1  1
Cho x y z  0 thì
f 0  f 0 2
f 0    f 0   0  f 0  . 3 3 9  3  3 2 1 1 1  1  1
x y z  2
f 1  f 1  f 1    f 1    0  f 1  ÁN Cho 1 thì           . 3 3 9  3  3 O 2 1
Cho y z  0 thì
f 0  f xf 0  . IC T 3 9 P Do f   1
0  nên f x 1  , x   . 1 YM 3 3 1 1 1 OL
Cho y z  1 , ta có
f x  f x  f xf 1  . C 3 3 9 Ụ Do f   1
1  nên f x 1  , x   . 2 PH 3 3
Từ 1 và 2 ta được f x 1  , x   . NH 3 I CH
Câu 40. Cho n  n  2 và hàm số f :  sao cho: f n x yn1
x f xf f y; x  ,y  *
a) Giả sử rằng f 2002  0. Tính f 2002. b) Tìm hàm số f . Lời giải a) Từ * ta được
 Với x  0; f y  f f y , y  
 Với x  1; y  0 : f f 0  0  f 0  0.
 Với x  1, y  : f 1  y  f 1 f y . 1
Do đó, chứng minh bằng quy nạp ta được f n  nf 1 , n   2
Từ 1 ta có f 0  f 1  f  1    f  1
   f 0  f 1   f t; f x  1  f x  f 1 .
Do đó, chứng minh bằng quy nạp ta được f n  nf 1 , n   3
Chinh phục olympic toán| 47
Bồi dưỡng học sinh giỏi
Từ 2 ,3 ta được f n  nf 1 , n   4 p Đặt f 1 *  ;p ,q và ta được * n  
n chia hết p nên nf 1  . q   2 2 f 1 0
Do đó ta được f f n  f n  n f  1  nf  1   f  1  f  1   .  f  1  1
Do đó, từ 4 ta được f 2002  1 hay f 2002  0 (loại).
Vậy f 2002  2002 .
b) Từ * ta được y f n x n1 0 :
x f x, x   1 f n x f n x
 Nếu n chẵn: x
  0 : f x  , f       xf  xn 1 n 1 xx
 Nếu n lẻ thì từ * và 1 ta được  n     n f x y
f x   f y 2
f xn   f n x n n
Suy ra f x   f x   0  f x     f x . n1    n 1 C xx
Do đó f x  f x , x   H
Từ 2 , chứng minh bằng quy nạp ta được  n    n f px pf x  , p   , x   ÁN Có *   :  n
    n    n p f px f px pf x n n U TO
Vậy f px   pf x  , p   , x   3 Ệ LI u n1  u   . u .vn  1 
Từ 3 ta có    * u , v  ta được 1 f    f    uv f v nn vvv      TƯ nu      f n 1 1 1 Mà f 1  
f    v f    f    VÀ n n n n Í  v   v   v vu   f 1 n 1   u CH Vậy f  . u v    f 1 4 n P  v v v ẠT
Ta có f 1  0 hay f 1  1 từ 4 suy ra f x  0, x
  hay f x  x, x  
Thử lại thỏa mãn * .
Vậy f x  0, x   .
Câu 41. Tìm tất cả các hàm số f :  thỏa mãn f  2 3
x y z   f x 2  f y 3  f zx  ,y,zLời giải
Đặt P u, v,t là phép thế x bởi u , y bởi v , z bởi t , ta có:
P0,0,0  f 0  0
Px,1,0  f x    f x  f  2 1 1
48 | Tạp chí và tư liệu toán học
Phương trình hàm trên tập rời rạc  f 1  0
P0, y, z    f  1  1
Với f 1  1, ta chứng minh f x 2
xf 1 , x  
Giả sử điều phải chứng minh đúng đến x k, k
  : f k 2  kf 1
Ta chứng minh đúng đến x k  1 k   , thực vậy:
f k    f k 2  f   2  kf   2
f    k   2 1 1 1 1 1 f 1
Với f 1  0 , ta chứng minh bằng quy nạp f x  0 x  
Thế vào giả thiết ban đầu, ta nhận được hai hàm thỏa đề: f x  0 và f x  x x  . Câu 42. Cho hàm số * * f : 
thỏa mãn đồng thời hai điều kiện:
a) f ab  f a,b f a,b với mọi * a,b
, a b ; trong đó a,b, a,b lần lượt là bội
chung nhỏ nhất, ước chung lớn nhất của hai số nguyên dương a,b ; ÁN O
b) f p q r   f p  f q  f r  với mọi số nguyên tố p, q, r . IC T
Tính giá trị của f 2013 ? Kí hiệu * là tập hợp tất cả các số nguyên dương. P Lời giải YM
Đặt f 2  a, f 3  b . Khi đó ta có các đẳng thức sau: OL
f 7  f 2  2  3  2 f 2  f 3  2a b C Ụ
f 8  f 2  3  3  f 2  2 f 3  a  2b PH
f 16  f 7  7  2  2 f 7  f 2  2 2a b  a  5a  2b
f 16  f 2 f 8  aa  2b . NH I Do đó ta có 2
5a  2b a  2ab 1. CH
Mặt khác ta có các đẳng thức sau:
f 12  f 2  3  7  f 2  f 3  f 7  3a  2b
f    f   f    af     2 12 2 6 2 2 2  3a Suy ra 2
3a  2b  3a 2 . 2
5a  2b a  2aba  2
Từ 1 ,2 ta có   ...  
f 7  7, f 8  8 2
3a  2b  3ab  3
Ta có 2003 là số nguyên tố nên
f 2013  f 2003  3  7  f 2003  f 3  f 7  f 2003  10 3
f 2025  f 2003  5  17  f 2003  f 5  f 17 4
f 9  3 f 3  9  f 5  2  2  f 5  2 f 2  f 5  5
f 17  f 7  7  3  2 f 7  f 3  17
Kết hợp với 4 ta được f 2003  f 2025  22 5
Chinh phục olympic toán| 49
Bồi dưỡng học sinh giỏi
Mặt khác f 2025  f 9.9.25  f 9 f 9.25  9. f 5.5.9  9 f 5 f 45
 9 f 5 f 3.15  45 f 3 f 15  45 f 3 f 7  5  3
 45.3 f 7  f 5  f 3  2025
Do đó f 2025  2025 , kết hợp với (5) ta được f 2003  2003 .
Do đó từ đẳng thức 3 ta được f 2013  2013 .
Câu 43. Đặt F f : 0,1  0,1 và n  2. Tìm giá trị nhỏ nhất của c thỏa mãn điều kiện 1 n f
x dx c f x dx   0   1   0
Với f F f là hàm liên tục. Lời giải 1 1 1 1 Ta có n nn f x dx n y f y dy n x   
f x dx n f x dx x  0,1    
, vậy c n . 0   1   1        0 0 0 C np  1 Ọ
Với p  0 , ta chọn hàm   p
f x x , khi đó: c np H np  1 ÁN Do đó: c  lim  n p p nn  U TO Vậy c n, lại có
2 nên giá trị c cần tìm là 2. Ệ LI
Câu 44. Tìm tất cả các hàm f :  1
 ,1  liên tục, thỏa mãn: TƯ    2x f x f  , x   1  ,1 2  1 x    VÀ Í Lời giải CH 2x P
Đặt g x 
f x f g x  với mọi 1   x  1 . 2 
. Bài toán trở thành:     Ạ 1 x T
Ta chứng minh bài toán nhỏ: Gọi dãy số a g a n
n1 . Khi đó, với mọi giá trị dương 0  a  1 a  1 , ta có: lim n 1 n 2a
Chứng minh. Với mọi số thực 0  a  1 , ta có: a a g a . 2 1     a
Lại có, với mọi 0  a  1 , ta thấy 0  g a  1 . Vì vậy a
nn là dãy tăng nghiêm ngặt và có giới hạn. 1 2L
Đặt lim a L L g L   L  1 n . Ta có:   n 2 1  L
Do L  0 nên lim a n 1 n
50 | Tạp chí và tư liệu toán học
Phương trình hàm trên tập rời rạc
f là hàm liên tục nên: lim f 1  h  f 1 , nói cách khác với mọi  0 luôn tồn tại  h0
sao cho với mọi 0  h   : f 1    f 1 
Đặt x  1   , từ chứng minh trên ta có các hàm x g x 2 ,
, g x , đều tiến đến 1.
Vậy tồn tại số nguyên dương k sao cho k
g x  1   . Điều này đồng nghĩa:  k
f g x  f 1  Vì     k f x
f g x nên f x  f 1  , do có thể chọn giá trị vô cùng bé, ta được:
f x  f 1  0  f x  f 1 với mọi 0  x  1.
Chứng minh tương tự, ta được f x  f  1   với mọi 1   x  0
f là hàm liên tục nên lim f x  f 0  f 1  f  1    f 0 x0
Vậy f x  c với c là hằng số bất kỳ. ÁN O
Câu 45. Có thể tồn tại hay không một hàm số f :
 , liên tục trên và thỏa mãn IC T
điều kiện: Với mọi số thực x , ta có f x là số hữu tỉ khi và chỉ khi f x  1 là số vô tỉ. P Lời giải YM
Giả sử tồn tại hàm số liên tục f :
 thỏa mãn điều kiện: OL x
  : f x  f x  1 \ * C Ụ
Xét các hàm số g x  f x  1  f x , h x  f x  1  f x PH
Khi đó g h là những hàm số liên tục trên
. Ta có g h không thể đồng thời là
hàm hằng. Thực vậy, giả sử g x  C , h x C 1   . Khi đó: NH 2 I
2 f x  C C f x C 2 1   (C là hằng số) CH
Vì t hế với   thì f   f   1  C , điều này mâu thuẫn với * .
Giả sử h không là hàm hằng, không mất tính tổng quát, khi đó tồn tại x , x , x x 1 2 1 2 sao
cho h x h x 1   2 .
Lúc này tồn tại số hữu tỉ r  h x , h x   1   2  . Ta có h
 x r h x r  0 1     2  
Lại có h x  r là hàm liên tục với mọi x thuộc
nên phương trình h x  r  0 có
nghiệm, tức là tồn tại x h x r
f x  1  f x r 0 sao cho  0  , từ đó  0   0  . Mà r
nên f x  1 , f x 0
  0  hoặc đồng thời hữu tỉ hoặc đồng thời vô tỉ, điều này mâu thuẫn với * .
Vậy không tồn tại hàm số thỏa đề.
Chinh phục olympic toán| 51
Bồi dưỡng học sinh giỏi
Câu 46. Tìm tất cả các hàm số f :
 thỏa mãn điều kiện f x  f t  f y  f z
với mọi số hữu tỉ x y z t x, y, z,t theo thứ tự lập thành cấp số cộng. USAJMO 2015 Lời giải
Do x, y, z,t theo thứ tự lập thành cấp số cộng nên x t y z , kết hợp với phương trình đã
cho ta được f x  f y z x  f y   f z 1 với mọi x y z và x, y, z theo thứ tự
lập thành cấp số cộng.
Thay x  0 vào (1) ta được f 0  f y z  f y   f z với mọi 0  y z .
Đặt g x  f x  f 0 ta được g y z  g y  g z 0
  y z 2
Với số nguyên dương n  3 , ta sẽ tìm cách biểu diễn g n theo g 1 , g 2 .
Ta có g 3  g 1  g 2 , g 4  g 3  g 1  g 2  2g 1 , dùng quy nạp ta chứng minh
được g n  g 2  n  2 g 1 3 . C Ọ
Lấy 2  y z là các số nguyên dương ta được: H
g y  g2  y  2 g1 , gz  g 2  z  2 g 1 , g y z  g 2  y z  2 g 1 , ÁN Thay vào 2 ta được
g 2  y z  2 g1  g2  y  2 g 1  g 2  z  2 g 1  g 2  2g 1. * U TOỆ
Kết hợp với 3 ta được g n  ng 1 LI
Ta biểu diễn g nx theo g x , g 2x . TƯ
Bằng quy nạp ta dễ dàng chứng minh g nx  g 2x  n  2 g x 4 . VÀ Í
Lấy 2  n m là các số nguyên dương, thực hiện các thao tác như * ta được
g 2x  m n  2 gx  g 2x  n  2 g x  g 2x  m  2 g x  g 2x  2g x CH P
Kết hợp với 4 ta được g nx  ng x 5 ẠT  m   m m
Vậy với m, n nguyên dương: g m  ngg
g1  gx     
xg1 , với x    n   n n
Do f x  f t  f y  f z  g x  g t  g y  g zx
  y z t lập thành CSC
Với x  0 bất kỳ, ta xét CSC x  0  y z ta được
g x  gt  g 0  g z  g x  tg 1  g 0  zg 1  g x  g 0  zg 1 tg 1  xg 1
Vậy g x  xg 1 , x
   f x  f 0  xf 1  f 0  ax b với x, a,b .
Thử lại ta thấy thỏa mãn.
52 | Tạp chí và tư liệu toán học
Phương trình hàm trên tập rời rạc
Câu 47. Giả sử r , s
là hai số cho trước. Tìm tất cả các hàm số f :  thỏa mãn
điều kiện f x f y  f x r   y s, x  ,y Romania 2006 Lời giải
Thay x  0 vào phương trình ban đầu ta được
f f y  f r  y s, y   1
Thay y bởi f y thu được:
f x f f y  f x r  f y  s, x  ,y
f x f r  y s  f x r  f y  s, x  ,y
f x r y f r  r s  f x r  f y  s, x  ,y
f x y f r  r s  f x  f y  s, x  , y  ÁN
Đặt a f r   r s thay vào phương trình trên ta được: O
f x y a  f x  f y  s, x  , y  2 IC T y
f x a f x f 0  s, x   3 P Thay
0 vào phương trình (2) ta được:        .
Từ 2 ,3 ta được: YM
f x y  f 0  s f x  f y  s, x  , y  OL C
f x y  f 0  f x  f y , x  , y  Ụ
f x y  f 0  f x  f 0  f y  f 0 , x  , y  PH
Đặt f x  f 0  g x , x
  và thay vào phương trình trên ta được NH I
g x y  gx  g y , x  , y  4 CH    Từ 4 , g x bx x b
theo kết quả về phương trình hàm Cauchy ta được   , ,
Từ cách xác định hàm số g ta được f x  bx f 0  bx c 5
Từ 5 thay lại vào phương trình ban đầu ta được:
bx by c  c bx r  y s c, x  ,y  2
bx b y bc c bx y br s c, x  , y  
b r s 2  b  1     b  1 
bc c br s c   c   r    s
Vậy bài toán có hai nghiệm f x  x r s, f x  x r s, x   .
Chinh phục olympic toán| 53
Bồi dưỡng học sinh giỏi
Câu 48. Tìm tất cả các hàm số f :
 sao cho với tất cả các số nguyên a,b,c thỏa
mãn a b c  0 , đẳng thức sau là đúng:
f a2  f b2  f c2  2 f af b 2 f bf c 2 f cf aIMO 2012 Lời giải
Lời giải 1. Tôn Ngọc Minh Quân Giả sử hàm f : 
thỏa mãn điều kiện đề bài.
Cho a b c  0 , ta được f 0  0 .
Cho a n, b  n,c  0 n   ta được f n  f n . Đặt f 1  t t   .
Cho a  2, b  1  ,c  1
 ta có f 2  0 hoặc f 2  4t .
Trường hợp 1. f 2  0  f 3  t 2 2 2
Ta có  f 4   f 2   f (2)  2 f 2 f 4  2 f 2 f 4  2 f 2 f 2  f 4  0 C Ọ
Giả sử f 2i  0, f 2i  1  t 1  i k H
  f k  2   f k2   f  2 2 2 2 2
 0  f 2k  2  0 ÁN 2 2 2
Ta có  f 2k  3   f 2k   f 3  2 f 3 f 2k  3 f 2k  3  f 3  t U TO
Vậy f 2i  0, f 2i  1  t, i
  N f 2i  0, f 2i  1  t, i   Ệ LI
Trường hợp 2. f 2  4t t  ,t  0 2 2 2 TƯ
Ta có  f 3   f 2   f 1  2 f 1 f 2  2 f 1 f 3  2 f 2 f 3
Suy ra f 3  t hoặc f 3  9t VÀ Í
a) f 3  9t , f 2  4t , f 1  t . Ta chứng minh f n 2 *  n t, n   CH
Thật vậy mệnh đề đúng với n  1, 2, 3 . Giả sử mệnh đề đúng đến n  3 P 2 2 2 Ạ
Ta có  f n  1   f n   f 1  2 f 1 f n  2 f 1 f n  1  2 f nf n  1 T
  f n  2  t 2
n   f n   2  t  2 1 2 1 1 n  12  0
f n    tn  2 1
1 hoặc f n    t n  2 1 1 2
Giả sử f n  1  t n  1  f n  1 2 2 2 2
Ta có  f n  1   f 2   f n  1  2 f 2 f n  1  2 f 2 f n  1  2  f n  1   f  2 2
 2 f 2 f n  1  f n  1
t t n  2 t t t n  2 2 2 2 16 8 .2 1 16 16 1 .
Đó là điều vô lý (vì n  3 ). Vậy f n 2 *  n t n    f n 2 ,  n t, n  
b) f 3  t, f 0  0, f 1  t, f 2  4t
54 | Tạp chí và tư liệu toán học
Phương trình hàm trên tập rời rạc
f  2  f  2  f  2 4 2 2
 2 f 2 f 2  2 f 2 f (4)  2 f 2 f 4
f 4  0 hoặc f 4  16t 2 2 2
Giả sử f 4  16t . Ta có  f 4   f 3   f 1  2 f 1 f 4  2 f 3 f 4  2 f 1 f 3 2 2 2 2 2
 256t  2t  32t  32t  2t 2
 192t  0 (vô lý).
Vậy f 4  0 . 2 2 2
Ta có  f 5   f 4   f 1  2 f 1 f 5  f (5)  f (1)  t ,
f  2  f  2  f  2 6 4 2
 2 f 2 f 6  f 6  f 2  4t ,
f  2  f  2  f  2 7 4 3
 2 f 3 f 7  f (7)  f (3)  t ,
f  2  f  2  f  2 8 4 4
 0  f 8  0
Bằng phương pháp quy nạp toán học ta chứng minh được
f 4i  1  t i N ; f 4i  3  t i  ÁN O
f 4i  0 i N ; f 4i  2  4t i
Thật vậy giả sử f 4k  0, f 4k  1  t, f 4k  2  4t, f 4k  3  t k N  IC TP 2 2 2
Ta có  f 4k  1   f 4k   f 1  2 f 1 f 4k  2 f 4kf 4k  1  2 f 1 f 4k  1 YM
f 4k  1  f 1 OL
f k  2  f k2  f  2 4 2 4 2
 2 f 2 f 4k  2 f 4kf 4k  2  2 f 2 f 4k  2 C Ụ
f 4k  2  f 2  4t PH
f k  2  f k2  f  2 4 3 4 3
 2 f 3 f 4k  2 f 4kf 4k  3  2 f 3 f 4k  3 NH
f 4k  3  f 3  I t 2 2 2 CH
f 4k  4  f 4k  f 4  2 f 4 f 4k2 f 4kf 4k  42 f 4 f 4k  4
f 4k  4  f 4  0 .
Suy ra f 4i  0, f 4i  1  t, f 4i  2  4t, f 4i  3  t t  ,t  0 i  
Ngược lại, giả sử hàm f :
 thỏa mãn f 2i  0, f 2i  1  t t   với mọi i
Giả sử a, b,c  , a b c  0 . Suy ra trong 3 số a,b,c có ít nhất một số chẵn
+) Nếu a,b,c cùng chẵn thì f a  f b  f c  0
  f a2   f b2   f c2  2 f af b  2 f bf c  2 f cf a
+) Nếu a chẵn và b,c lẻ thì f a  0 , f b  f c  t
  f a2   f b2   f c2 2  2t
f af b f af c f bf c 2 2  2t   f a 2
( )   f b2   f c2  2 f af b  2 f bf c  2 f cf a
Chinh phục olympic toán| 55
Bồi dưỡng học sinh giỏi
Tương tự nếu b chẵn a,c lẻ hoặc c chẵn a,b lẻ thì ta cũng có:
f a2  f b2  f c2  2 f af b 2 f bf c 2 f cf a Vậy hàm f :
 sao cho f 2i  0 , f 2i  1  t t   với mọi i  thỏa mãn điều kiện đề bài.
 Xét hàm số f :  thỏa mãn f n 2
n t t  ,t  0 n  
Giả sử a, b,c  thỏa mãn a b c  0 Ta có f a 2
a t f b 2
b t f c 2 , ,  c t 2 2 2
Suy ra              4 4 4    2 f a f b f c a b c t Có 2 2 2
a b c  0  a b c  2ab  2bc  2ca 4 4 4 2 2 2 2 2 2 2 2 2 2 2 2
a b c  2a b  2b c  2a c  4a b  4b c  4a c  8abc a b c 4 4 4 2 2 2 2 2 2
a b c  2a b  2b c  2a c
  f a2   f b2   f c2 2 2 2 2 2 2 2 2 2
 2a b t  2b c t  2a c t C Ọ
 2 f af b  2 f bf c  2 f cf a H Vậy hàm f :
 sao cho f n 2
n t t  ,t  0 n
  thỏa mãn đề bài. ÁN
 Xét hàm f :  thỏa mãn
f 4i  1  t, f 4i  2  4t, f 4i  3  t, f 4i      U TO 0 t ,t 0 i
Giả sử a, b,c  sao cho a b c  0 LI
+ Nếu a  4i i   b c  0 mod 4 TƯ
+ Nếu b,c đều chia hết cho 4 thì f a  f b  f c  0 VÀ 2 2 2 Í
  f a   f b   f c  2 f af b  2 f bf c  2 f cf a 0 CH
+ Nếu b  2 mod 4 và c  2 mod 4 thì f a  0, f b  4t, f c  4t P 2 2 2 2 Ạ
  f a   f b   f c  32t T
f af b  f bf c  f cf a 2 2 2 2  32t
  f a2   f b2   f c2  2 f af b  2 f bf c  2 f cf a
+ Nếu b  1mod 4 và c  3mod 4 thì f b  t, f c  t
  f a2   f b2   f c2 2  2t
f af b  f bf c  f cf a 2 2 2 2  2t
  f a2   f b2   f c2  2 f af b  2 f bf c  2 f cf a
+ Nếu a  1mod 4 , b  0 mod 4 và c  3mod 4 , tương tự như trên ta cũng có:
f a2  f b2  f c2  2 f af b 2 f bf c 2 f cf a
+ Nếu a  1( mod 4) , b  3mod 4 và c  0 mod 4 , tương tự như trên ta cũng có:
56 | Tạp chí và tư liệu toán học
Phương trình hàm trên tập rời rạc
f a2  f b2  f c2  2 f af b2 f bf c2 f cf a
+ Nếu a  1mod 4 , b  2 mod 4 và c  1mod 4
f a  t, f b  4t, f c  t 2 2 2
  f a    f b    f c  2 ( ) ( ) ( )  18t
f af b  f bf c  f cf a 2 2 2 2 2 2 2
 8t  8t  2t  18t
  f a2   f b2   f c2  2 f af b  2 f bf c  2 f cf a
+ Nếu a  1mod 4 , b  1mod 4 và c  2 mod 4 , tương tự như trên ta cũng có:
f a2  f b2  f c2  2 f af b 2 f bf c 2 f cf a
+ Nếu a  2 mod 4 , b  0 mod 4 và c  2 mod 4 hoặc a  2 mod 4 , b  1mod 4 và
c  1mod 4; hoặc a  3mod 4, b  0mod 4 và c  1mod 4hoặc a  3mod 4,
b  1mod 4 và c  0mod 4 , tương tự như trên ta cũng có:
f a2  f b2  f c2  2 f af b 2 f bf c 2 f cf a ÁN O
+ Nếu a  3mod 4 , b  3mod 4 , c  2 mod 4 thì f a  f b  t, f c  4t 2 2 2 IC T
  f a   f b   f c 2
 18t ; f af b  f bf c  f cf a 2 2 2 2  18t P
  f a2   f b2   f c2  2 f af b  2 f bf (c) 2 f cf a YM
Vậy tất cả các hàm f :
 thỏa mãn đề bài là: OL C
f :  : f 2i  0, f 2i  1  t t   i   Ụ
f :  : f n 2
n t t  ,t  0 n   PH
f :  : f 4i  0, f 4i  1  t, f 4i  2  4t, f 4i  3  t t  ,t  0 i   . NH I Cách 2.
Thay a b c  0 vào phương trình ban đầu ta được: CH
f  2  f  2  f  2  f   f    f   f    f   f    f  2  f  2 0 0 0 2 0 0 2 0 0 2 0 0 3 0
6 0  f 0  0
Thay b  a,c  0 vào phương trình ban đầu ta được:
f a2  f a2  f  2
0  2 f af a  2 f af 0  2 f 0 f a
f a2  f a2  f af a  f a2  f af a  f a2 2 2
 0  f a  f a
Ta có thể viết lại phương trình ban đầu dưới dạng:
f c2  2 f c f a  f b   f a  f b2  0
2  f a  f b  4 f a  f b2  4 f a  f b2
f c  f c
   f a b  2
f a b  f a  f b  2 f af b
Nếu f b  0 : f a b  f a  f amodb
Trường hợp 1. f 1  0  f x  0 x  .
Chinh phục olympic toán| 57
Bồi dưỡng học sinh giỏi
Trường hợp 2. f 1  0, ta có f 2  f 1  f 1  2 f 1 f 1  f 2  0 or f 2  4 f 1
Ta xét hai trường hợp nhỏ:
Trường hợp 2.1: f 1  0, f 2  0  f x  f x mod 2  f x  f (1) nếu x lẻ và f x  0 nếu x chẵn.
Trường hợp 2.2: f 1  0, f 2  4 f 1  f 3  f 2  f 1  2 f 2 f 1
f 3  5 f 1  4 f 1  f 3  f 1  9 f 1
 Nếu f 1  0, f 2  4 f 1, f 3  f 1 :
f 4  f 1  f 3  2 f 1 f 3 và f 4  f 2  f 2  2 f 2 f 2
f 4  f 1 hoặc 0 và f 4  16 f 1 hoặc 0
f 4  0  f x  f x mod 4.
 Nếu f 1  0, f 2  4 f 1, f 3  9 f 1 : C
f 4  f 1  3  f 1  f 3  2 f 1 f 3  16 f (1) f 1 Ọ hoặc 4   H
f 4  f 2  f 2  2 f 2  16 f 1 hoặc 0.  f 4  16 f 1 ÁN
 Nếu x  4 , khi đó f x  f   2 1 x
Dùng quy nạp, ta chứng minh: f x  f   2 1 x x  U TOỆ
 Nếu . x m ., khi đó f x  f   2 1 x x
 , đúng với một số giá trị m . LI
Giả sử điều phải chứng minh đúng với m k : TƯ
f k    f k  f   
f kf    f  k  2 1 1 2 1 1
1 hoặc f  k  2 1 1 VÀ Í
f k    f k    f   
f k   f    f  k  2 1 1 2 2 1 2 1
1 hoặc f  k  2 1 3 CH
f k    f  k  2 1 1 1 . P Ạ
Vậy điều phải chứng minh đúng với m k  1 . T
Vì nó vẫn đúng với m  4 , theo quy nạp toán học ta có thể kết luận f x  f   2 1 x x  .
Câu 49. Tìm tất cả các hàm f , g :  
có đạo hàm trên  thỏa mãn f x f xg x '   ; g x   '   x    x x Lời giải Ta có  . x
  f x  g x'  x
f 'x g'x f x gx
gxf x   x 
  f x  gx  0, x   0  x x   x f
 x  gx  a x   0 
f x  gx  a x  0 1 x
58 | Tạp chí và tư liệu toán học
Phương trình hàm trên tập rời rạc
f x  gx Tương tự ta có '  0, x    
0  f x  gx  bx, x
  0  x  0 2  x   a   a
Từ 1 ,2  f x 1   
bx ; gx 1    bx , x
  0, a,b 2 x    2  x
Câu 50. Tìm tất cả các hàm * f :  
có đạo hàm trên * thỏa mãn
f xy  f x  f y * x  , y  1  Lời giải
Lấy đạo hàm hai vế 1 lần lượt theo biến x, y ta có
yf xy  f x * ' ' x  , y  
xf xy  f y * ' ' x  , y    .
x f 'x  y. f 'y * x  , y   ÁN
x. f '(x)  a * x  
f x  .
a ln x b * x   R O  
Thử lại  b  0 .
IC TP Vậy f x .alnx * x    . f  YM
Câu 51. Tìm tất cả các hàm : thỏa mãn
f f n  n b n    1 OL C
trong đó b là số nguyên dương chẵn. Ụ Lời giải PH
Giả sử f thỏa mãn đề bài. Dễ thấy f đơn ánh NH f f
  f n   f n I
Trong 1 thay n bởi f n ta được    b  CH
Suy ra f n b  f n  b n  
Vậy nếu m qb r , 0  r b  1 thì f m  f qb r   f q  1b r   b f r   qb
Bây giờ ta chỉ cần xác định hàm f trên tập A  0,1, 2,..., b   1 .
Xét x A , đặt f x  y thì f y  x b
Giả sử y qb r thì f y  f r   qb suy ra x b f r   qb qb x b  2b nên q  0 hoặc q  1 .
Do đó nếu x A thì
i) f x  r b,r A, f r   x
ii) f x  r,r A, f r   x b
f qb r  qb f r 
Vậy hàm f được xác định như sau  f a b
a b A i i
với  i  i .  f
 b a b i i
Chinh phục olympic toán| 59
Bồi dưỡng học sinh giỏi
Câu 52. Tìm tất cả các hàm f :    thỏa mãn:
i) f xf y yf xx, y     1
ii) lim f x  0 x IMO 1983 Lời giải
Thế x  1 ta được f f y  yf 1
Nếu f a  f b , thì af 1  f f a  f f b  bf 1  a b f 1 tiến tới số nguyên
dương. Vì vậy hàm f đơn ánh.
Thế x y ta được f xf x  xf (x)
Vậy xf x là điểm bất động của hàm f .
Khi đó, thay y  1 thu được f xf 1  f x  f 1  1 vì x khác 0 nên 1 là điểm bất C
động của hàm f. Ta chứng minh 1 là điểm bất động duy nhất của hàm f . Ọ f a a f b b H
Giả sử tồn tại a,b là điểm bất động của f , ta có   và   .
Khi đó, thế x a, y b vào i ta được f ab  f af b  bf a  ab ÁN
Suy ra ab cũng là điểm bất động của f . U TO 1  1   1   1   1  1 Ệ
Thế x  , y a thu được 1  f 1  fa f
f a  aff          LI aa   a   a   a a 1 TƯ
Vậy là điểm bất động của f . a n
Nếu f a  a với a  1 , khi đó f a  là điểm bất động của f , mâu thuẫn ii Í  1  1 CH
Nếu f a  a với 0  a  1 , khi đó f  
là điểm bất động của f , mâu thuẫn iin n P  a a ẠT
Từ các kết quả trên ta thu được điểm bất động duy nhất là 1, suy ra xf x   f a 1 1  . x
Câu 53. Chứng minh rằng tồn tại song ánh f :    sao cho
f 3mn m n 4 f mf nf mf nm,n        
IMO Shortlist 1996 Lời giải x   Xét hàm g : 3  1 4   
 1 thỏa mãn: gx 1  4 f  
 1 thì ta có g là song ánh và  3 
g 3m 13n 1 g3m 1 g3n 1 m,n         .
Thật vậy g 3m  13n  1  g 33mn m n  1  4 f 3mn m n  1
 44 f mf n  f m  f n  1  4 f m  14 f n  1  g3m  1 g3n  1
Vậy với mọi x, y 3  
 1 , ta có gxy  gx.gy
60 | Tạp chí và tư liệu toán học
Phương trình hàm trên tập rời rạc g 3x  1  1
Như thế, ta chỉ cần chỉ ra một song ánh g là đủ, vì khi đó f x    thỏa mãn 4 đề bài. Xét P , P k k Q ,Q 1
2 là tập các số nguyên tố dạng 3 1, 3
2 tương ứng và 1 2 là tập các số
nguyên tố dạng 4k  1, 4k  3 tương ứng, ta xét song ánh h : P P Q Q 1 2 1 2
Sao cho h P Q , h P Q 1  1  2 
2 và xác định g như sau:  g 1  1
 nguyên tố thì gn  hpi
Rõ ràng song ánh g như vậy thỏa mãn đề bài.
Câu 54. Tìm tất cả các hàm f :  thỏa:
3 f f f n  2 f f n  f n  6n, n   ÁN Lời giải O Đặt k
a f n f f f n k  
 ...   (lặp k lần).
IC TP Ta lặp được dãy 3a a    a a k 2 k k 6 1 2 k3 với mọi n YM
Xét tập S  a , a , a ,... 0 1 2
 vì đây là tập các số nguyên dương nên sẽ tồn tại một phần tử có OL
giá trị nhỏ nhất. Đặt j là số sao cho aj là giá trị nhỏ nhất trong tập S . C Ụ
Ta có bất đẳng thức 3a a     a a a k 2 k k 6 k 6 1 2 3 j PH
Đẳng thức xảy ra khi a a    a a k k 1 k 2 j
Lại có khi đặt k j  3 thì đẳng thức xảy ra, vậy ta có a a    a a . NH j j 1 j 2 j3 I
Tịnh tiến giá trị ta thu được a ak j với mọi k j . CH 3a  2a a
Thực hiện tương tự cho tịnh tiến lùi, lưu ý rằng k 3 k 2 k 1 a     a a k , ta thu được 6 k j
với mọi k j . Vậy a a k k j với mọi 0 .
Từ chứng minh trên dẫn đến a a
f n n   1 0 hay   , n .
Câu 55. Tìm tất cả các hàm số f : 0;   0;  thỏa mãn điều kiện:
f f x  yf yf x x
 , y 0;  1 Lời giải f u
Với mọi t 0;  , ta chọn tùy ý một x u f x v tf x t  0 cố định và  0  ,  0  thì f v 1 1
f x 
Ta thay trong 1 y bởi
ta có f f x  f   f y
f y  f y   
Chinh phục olympic toán| 61
Bồi dưỡng học sinh giỏi
f x  Hay f     f y
f y. f f x  2    1 1
Trong 1 , thay y bởi f f x    ta được    f 1 x   0; 3 f x
f x      
f x  f y
Từ 2 ,3 suy ra f     f  yf 1   .  f  x
f u  f v Do đó f     f  vf 1   .  f  u 1
Do vậy, f t  f 1. t  0; . t a
Thử lại, hàm cần tìm là f x  x
 0; ở đó a  0 là hằng số. x C Ọ
Câu 56. Chứng minh rằng tồn tại duy nhất một hàm số f xác định trên tập các số thực H
dương, nhận giá trị thực dương và thỏa mãn f f x  6x f x. ÁN Putnam 1988 Lời giải U TO
Với mỗi số thực dương x cố định, ta xây dựng dãy  fn như sau: Ệ 0 n1 LI
f x , f f x , f   f f x n n . 1 0 2  0  1   0 TƯ
Khi đó, từ đẳng thức ở giả thiết ta suy ra dãy  fn thỏa mãn phương trình truy hồi n1 f    f f n 6 n n , 2 1 VÀ Í Hay f     f f n n 6 n 0. 2 1 CH
Đến đây, giải phương trình đặc trưng của dãy  fn , ta được hai nghiệm là 2 và 3  . P n1 Ạ n n T
Do đó, f a   b  f , f n 2
 3 , trong đó, các hằng số a, b tìm được phụ thuộc vào 1 2 .
Tuy nhiên, nếu b  0 , thì tồn tại n đủ lớn sao cho f n
0 (ta có thể thấy được dễ dàng bằng
cách chọn n chẵn đủ lớn nếu b  0 , và chọn n lẻ đủ lớn nếu b  0 ).
Do vậy, b  0 . Thành ra f a  2n n
Suy ra f f x  3  .2 a , f x  2  .2 a 0 0
, thay hai giá trị này vào đẳng thức
f f x  6x f x 0  0  0 
Ta được 2a x f x  2x 0 . Dẫn đến  0  0
Và vì điều này đúng với mọi x f x x x   0 dương nên   2 , 0 .
62 | Tạp chí và tư liệu toán học
Phương trình hàm trên tập rời rạc
Câu 57. Hàm số f :
 thỏa mãn đồng thời các điều kiện sau:
i : f f n  n, n   1
ii : f f n 2 2  n, n   2
iii : f 0  1 3
Tìm giá trị f 1995 , f  2007  
Olympic Ukraine 1995 Lời giải
Cũng nhận xét và lý luận như các ví dụ trước, ta đưa đến f n phải có dạng: f n  an b
Khi đó điều kiện i trở thành: 2
a n ab b n, n   2 a  1
a  1 a  1 
Đồng nhất các hệ số, ta được     
ab b  0
b  0 b  0 a  1 f n n ii ÁN Với  ta được  
. Trường hợp này loại vì không thỏa mãn   b   0 O a  1 
f n  n b IC T Với  ta được   b   0 P
Từ điều kiện iii cho n  0 ta được b  1 YM
Vậy f n  n  14 OL C
Hiển nhiên hàm số này thỏa mãn điều kiện bài toán. Ụ
Ta phải chứng minh f n  n  1 là hàm duy nhất thỏa mãn điều kiện bài toán PH
Thật vậy giả sử tồn tại hàm g n khác f n cũng thỏa mãn điều kiện bài toán.
NH I Từ iii suy ra f 0  g0 1 CH
Từ iii suy ra f 1  g 1  0
Sử dụng điều kiện i ,ii ta nhận được g g n  g g n  2  2 n
Dno đó g ggn  gggn  2  2 n
Hay g n  g n  2  2 n  Giả sử n f n g n 5
0 là số tự nhiên bé nhất làm cho  0   0  
Do f n cũng thỏa mãn 4 nên ta có
gn  2  g n  2  f n  2  f n  2 0   0   0   0 
gn  2  f n  2 0   0 
Mâu thuẫn với điều kiện n0 là số tự nhiên bé nhất thỏa mãn 5
Vậy f n  g n , n
Chứng minh tương tự ta cũng được f n  g n với mọi n nguyên âm.
Chinh phục olympic toán| 63
Bồi dưỡng học sinh giỏi
Vậy f n  1  n là nghiệm duy nhất.
Từ đó tính được f 1995 , f  2007   .
Câu 58. Tìm f : 0, 1  thỏa mãn:
f xyz  xf x  yf y  zf zx
 ,y,z0,1 Lời giải
Chọn x y z ta được f  3
x   3xf x
Thay x, y, z bởi 2
x ta được f  6 x  2  x f  2 3 x  Mặt khác f  6
x   f  2 3
x x x   xf x 2  x f  2 x  3  x f  3 . . x  Hay 2 x f  2
x   xf x 2  x f  2 x  4 3
 3x f x 3 2   x f  2
x   xf x 4 2  3x f xxf  2 x  3 1 
f x , x   C 2 Ọ 9  9  6 3x 1 3 2 2 3x 1 H Thay x bởi 3
x ta được f x  
f x , x
   3x f x  
3xf x , x   2 2 ÁN 3 9   2 3x 1  x
f x 3x 1 3 
3xf x , x
   f x  0, x   0 2 2 U TO
Vậy f x  0 với mọi x 0;1 . Ệ LI TƯ
Câu 59. Tìm tất cả các hàm f xác định trên
và thỏa mãn đồng thời các điều kiện sau:
2 f nf k n  2 f k n  3 f nf k,k n VÀ Í   f  1  1 CH Lời giải P 2 2 Ạ
Cho k n  0  2 f 0  2 f 0  3 f 0  f 0  0  f 0  2  T
Nếu f 0  0 chọn n  0 ta được: 2
f k  0 do đó f k  0 với mọi k
Chọn k  1 ta được f 1  0 mâu thuẫn với giả thiết. Vậy f 0  2 
Chọn n  1 ta được phương trình 2 f 1 f k  1  2 f k  1  3 f 1 f k , k
 2 f k  1  2 f k  1  3 f k , k  Đặt x f k 2x     x x k 3 k 2 k 0 k
 ta có phương trình sai phân 1 1
Phương trình đặc trưng là 2 1
2  3  2  0    2     2 n  1 n
Vậy f n  c 2  c  1 2  . 2   
64 | Tạp chí và tư liệu toán học
Phương trình hàm trên tập rời rạc Ta tìm c ,c f 0  2  , f 1  1 1 2 từ điều kiện     .
Dễ tìm được c  0,c  2  1 2 n   Vậy f n 1  2    . 2   
Câu 60. Tìm tất cả các hàm số * * f : 
thỏa mãn đồng thời hai điều kiện sau:
f f n * *
n  2k, n , k     f
 n  1  f n * , n Lời giải
Giả sử có hàm f thoả mãn các điều kiện 1 ,2 .
Ta chúng minh hàm f là một đơn ánh. Thật vậy, với mọi * m,n
m n , nếu có f m  f n thì do 1 suy ra ÁN O
f f m  f f n  m  2k n  2k m n (vô lý)
Suy ra f là đơn ánh. Do 2  f n  1  f n với mọi * n  suy ra IC TP
f n  1  f n  1 với mọi * n  YM
f f n  1  f f n  1  1 f f n với mọi * n  OL
f f n  1  1 f f n với mọi * n  C Ụ
n  1  2k  1  n  2k với mọi * n  PH
f f n  1  f f n  1 với mọi * n
f n  1  f n  1 NH với mọi * n
( do f là đơn ánh) I
f n  f n  1  1 với mọi * n  2 ; n  CH Truy hồi ta được
f n  n  1  f 1 với mọi * n
f f n  f n  1 f 1  n  2  2 f 1 với mọi * n
n  2k n  2  2 f 1 với mọi * n
f 1  k  1 với mọi * n
Suy ra f n  n  1  k  1 với mọi * n
f n  n k với mọi * n
Thử lại f n  n k với mọi * n
thỏa mãn các điều kiện 1 ,2
Vậy các hàm f cần tìm thỏa mãn đề bài là f n  n k với * n  .
Chinh phục olympic toán| 65
Bồi dưỡng học sinh giỏi
Câu 61. Tìm tất cả các hàm số f :
 thỏa mãn đồng thời hai điều kiện sau:  f  2013  2016 
f f n  n  4, n    Lời giải
Giả sử có hàm f thoả mãn các điều kiện 1 ,2 .
Từ điều kiện 1 ta dễ dàng chứng minh được f là một đơn ánh.
Từ 1 ta suy ra f f f n  f n  4 với mọi n
f n  4  f n  4 với mọi n (3)
Với n  4k r với k  ; r 0, 1, 2,  3
Từ 3 ta suy ra f 4k r   4  4k f r  với mọi k
 Tính f 1 . Do 2013  4.503  1 nên f 2013  2012  f 1  2016  f 1  4 C
 Tính f 0 . Ta có f f 0  4  f 1  f 0  1 (do f là đơn ánh) Ọ H
 Tính f 2 và f 3 . Giả sử f 2  4m r với m và r 0,1,2,  3 1
f f 2  6  2  4  f f 2  f 4m r  4m f r  6 ÁN
Do   mà ta có         ;
f r   0  m  0 hoặc m  1 U TO
+ Với m  0 , thì f r   6 và f f 2  f r   f 2  r Ệ LI
+ Với m  1 , thì f r   2 và f 2  4  r TƯ  f  r  6
Trường hợp 1. Xét m  0   f 2  r  r  0,1,2,  3  VÀ Í  f 0  6 CH
+ Với r  0 thì 
vô lý do f 0  1  f  2  0 P ẠT  f 1  6
+ Với r  1 thì 
vô lý do f 1  4  f  2  1  f 2  6
+ Với r  2 thì  vô lý  f  2  2  f 2  6
+ Với r  2 thì  vô lý  f  3  2
Vậy khi m  0 ta có f 0  1; f 1  2; f 2  3; f 3  6  f  r  2
Trường hợp 2. Xét m  1  
f 2  r  4  r  0,1,2,  3 
+ Với r  0 thì f 0  2 vô lý do f 0  1
+ Với r  1 thì f 1  2 vô lý do f 1  4
66 | Tạp chí và tư liệu toán học
Phương trình hàm trên tập rời rạc  f 2  2
+ Với r  2 thì  vô lý  f  2  6  f 2  7
+ Với r  2 thì  vô lý  f  3  2
Vậy khi m =1 ta có f 0  1; f 1  4; f 2  7; f 3  2
n  1 , n  0 mod 4 
n  3 , n  1 mod 4
Suy ra f n   n  5 , n  2  mod 4 n1 , n  3  mod 4
Thử lại: f n thỏa mãn đồng thời các điều kiện (1) và (2) nên f n là các hàm cần tìm.
Câu 62. Tìm tất cả các hàm số * f : 
thỏa mãn điều kiện sau: ÁN O
f n  f n  1  f n  1. f n  3 , n   1 Lời giải
IC TP Giả sử có hàm f thoả mãn các điều kiện 1. YM Ta lập dãy a a f n 1
n  với mỗi n  ta đặt n
  khi đó   trở thành OL a a     a a n 2 n n n n , 2 1 3   C Ụ
Thay n n  2 trong phương trình 2 ta được a      a a a n 3 n n n n , 2 4 3 5   PH
Trừ từng vế của 2 ,3 ta được a       a a a a n n n n n n , 4 3  5 1  .
a a a a a 4 0 3  5 1  NH I 
a a a a a 5 1 4  6 2 
Thay n lần lượt bởi 0,1, 2, 3,...... ta có  CH
a a a a a 6 2 5  7 3 
. . . . . . . . . . . . . . . .  
Suy ra a a a a a .....a  ,n  .  a a 4 4 0 3 4 5 n 2  n 4 n   
Ta chứng minh rằng a  ,n
f n  4  f n ,n .  a 5 n 4 n  
Hay f là hàm tuần hoàn với chu kỳ 4.
Thật vậy. Giả sử tồn tại số * n a    a n n n n ; 0 mà 0 4 0 0 Do * a    a    a 1 n với n nên suy ra 0 n 4 0 n
a a a a a .....a    a a a a a a n n n ..... 4 0 3 4 5 0 2  0 4 0  3 4 5 0 n 2 Do a a   
a a a a n , n , n ,  a a n n 2 n 1 n3 với n
, nên với 4 số liên tiếp phải có ít nhất 0 1 0 0 1 0 n 2 một số lớn hơn 1.
Do đó khi n   thì a a a .....a   a     a 3 4 5 suy ra (vô lý) 0 n 2 0 n 4 0 n
Chinh phục olympic toán| 67
Bồi dưỡng học sinh giỏi
Suy ra a a  0  a a   
f n  4  f nr  0,1,2,3 n n 0 4 0 4 với n với   trong đó f
là hàm tuần hoàn chu kỳ 4.
Hàm f được xác định khi ta tính được f 0  a ; f 1  a ; f 2  a ; f 3  a 0
  1   2   3 bởi vì từ 
a a a a
a a a a 2 ta có 0 2 3 4 0 2 1 3   
a a a a a a a a a    a 1 3 4 2 0 2 0 2 1 3 
 a 1 a 1  a a a a  1 0  2  0 2  0 2  Mặt khác 
a  1 a  1  a a a a  1  1  3  1 3  1 3
Suy ra a  1 a  1  a  1 a  1  2 6 0
 2   1  3   
Do đó có các khả năng xảy ra. 
 a 1 a 1  0       0  2  a 2 v a 3 a 1 v a 5 Khả năng 1. 1 1 0 0      
a  1 a  1  2  a  3 v a  2 a  1 v a  5 1  3  3 3 2 2
Suy ra  f 0 ; f 1 ; f 2 ; f 3   a ;a ; a ; a  1; 2; 5; 3 , 1; 3; 5; 2 , 5; 2;1; 3 , 5; 3;1; 2 0 1 2 3          C Ọ
Ta tìm được 4 hàm cần tìm với f n với f n xác định bởi H
f 0 ,n  0mod 4  ÁN
f 1 ,n  1mod 4
f n   7 f
 2 ,n  2 mod 4 U TO   Ệ f
 3 ,n 3mod 4 LI
đều thỏa mà điều kiện 1. TƯ
a 1 a 1  1 0  2 
Khả năng 2. 
a a a a  2  a ; a ; a ; a  2;2;2;2 0 1 2 3  0 1 2 3   
a  1 a  1  0  1  3  VÀ Í
Suy ra f (n)  2 với mọi n  CH
a 1 a 1  2 0  2  P
Khả năng 3.  Ạ
a 1 a 1  0  1  3  T
Lập luận tương tự như khả năng 1 ta được
f 0; f 1; f 2; f 3  a ;a ;a ;a  2;1;3;5 , 3;1;2;5 , 2;5;3;1 , 3;5;2;1 0 1 2 3         
Ta được 4 hàm f n cần tìm và cả 4 hàm đều thỏa mãn điều kiện 1.
Với f n xác định bởi 7.
Vậy có 9 hàm f thỏa mãn là nghiệm của phương trình 1.
Câu 63. Tìm tất cả các hàm f :    thỏa mãn:
f x f y  f x y  f yLời giải
Ta để ý rằng: f a f b  f c  fa f b  c f c  f a b c  f b  f c
68 | Tạp chí và tư liệu toán học
Phương trình hàm trên tập rời rạc
Thế y bởi y f z ta được:
f x f y f z  f x f y z  f z  f x y  2z  f y z  f z
f x y f z  f y f z  f x y z  f z  f y z  f z
Thu được: f x y  2z  f x y z  f z với mọi x, y, z  0
Với giá trị của y , ta chọn x sao cho x y, f y , từ đó ta có kết quả f f y  2 f y
Thế y x vào phương trình ban đầu f x f x  f 2x  f x
Dễ nhận thấy f x  x , vì vậy: f x  f f x  f 2x  f x , f f x  f 2x
Theo phương trình hàm Cauchy, ta được f x  2x với mọi x   .
Câu 64. Tìm số nguyên dương m nhỏ nhất sao cho tồn tại hàm số * f :  \ 1  ;0;  1
thỏa mãn đồng thời các điều kiện sau
ÁN i) f m  f 2015, f m1  f 2016; O f n  1
ii) f n m    , n  1, 2,.... IC T f n  1 P Lời giải YM 1
Ta có f n  2m        f n
f n 4mf n * , n OL C Ụ
Với m  1 , ta có f n    f n  f n k  f n * 4 4 , k  ,n PH 1 f n  1
Ta có f n  2 *
   ; f n1      , n   f n f n  1 NH I
f    f    f     f   1 1 2015 4.503 3 3   : vô lý. f 1 CH
Với m  2 , ta có f n    f n  f n k  f n * 8 8 , n  , k  1 f n  1
f n  4 *
   ; f n2      , n   f n f n  1 1
Ta có f 2  f 2015  f 251.8  7  f 7   f   ; 3
f    f    f     f   1 3 2016 251.8 8 8   f 4 
f    f   f 2 1 2 4 
  f 2 2   f     1 2  1
Điều mâu thuẫn trên dẫn đến m  3.
Với m  3, ta xây dựng được vô số hàm f thỏa yêu cầu bài toán như sau
Chinh phục olympic toán| 69
Bồi dưỡng học sinh giỏi 1  a 1 f n  1 Cho a  \ 1  ;0; 
1 , đặt f 1  a; f 2  ; f 3   ; f n  3  , n   1 1  và     a a f n  1
Khi đó, chứng minh quy nạp thì hàm số xác định trên * và f n    * \ 1;0;1 , n   1
Hơn nữa theo chứng minh trên f n  6   , f n
k  f n * 12 , n  , k f n 1 1  f 2 1
Khi đó f 2015  f 167.12  11  f 11        f
f 5 1  f 2 3 a   f    f     f   1 1 f 3 a 1 2016 167.12 12 12       f
f 6 1  f 3 4 a  1
Vậy hàm số thỏa mãn yêu cầu bài toán.
Câu 65. Xác định hàm số f x liên tục   
thỏa mãn đồng thời các điều kiện:   C
f 2x 2 f x với mọi x   , 1 Ọ 
 3  fx    2 1   x f f x e
x e  1 f x với mọi x   , 2 H
f e  1  e  1 f 1 , 3 ÁN
f k là số nguyên dương với mọi số nguyên dương k , 4 U TO Lời giải Ệ LI Với a, b  
f a  f b , suy ra f a f b 3 f a   e   3 1  f b   e 1. Do đó TƯ f  3
f af a
e    f  3 1
f bf b e 1 2 a 2 b 2 a 2 b VÀ Í
Hay a e  1 f a  b e  1 f b . Vì f a  f b  0 nên ta suy ra a e  1  b e  1 . 2 x x x CH
Xét hàm số h x  x e  1 trên  , ta có h'x  2x e   2
1  x e  0 với mọi x   . P 2 x
Do đó hàm số h x  x e  1 đồng biến trên  . T
Do đó từ 2  a   2 1   b a e
b e  1, ta suy ra ha  hb hay a b .
Vậy f x là đơn ánh. Kết hợp với f x liên tục ta suy ra f x là hàm đơn điệu thực sự.
Mặt khác, theo giả thiết f 2  2 f 1  f 1 nên ta suy ra f x là hàm tăng thực sự trên tập  .
Từ 2 ta cho x  1 thì f  3
f 1 f 1
e 1 e1 f 1.
Kết hợp với 3 ta suy ra f  3
f 1 f 1
e 1  f e1.
f x là hàm tăng thực sự trên  nên ta suy ra 3
f 1 f1
e 1 e1. Xét hàm số   3   x g x
x e  1 trên  , ta có 'x 2   x 1 3 3 x g x e
x e  0 với mọi x   . Do đó hàm số   3   x g x
x e  1đồng biến trên  .
70 | Tạp chí và tư liệu toán học
Phương trình hàm trên tập rời rạc Do đó từ 3
f 1 f1
e 1 e1, ta suy ra gf 1 g1 hay f 1  1.
f 2x  2 f x với mọi x  
f 1  1 nên theo quy nạp ta có 2n  2n f  với mọi số tự nhiên n .
Với mọi số tự nhiên n , ta có
n f n   f n    f n    f n n
   f n1  n1 2 2 2 1 2 2 2 2 1 2  2
Vì điều kiện 4 nên 2n  1 , 2n  2 ,, 2n  2n f f f
 1 đều là các số nguyên dương.
Do đó ta suy ra 2n  1  2n  1, 2n  2  2n  2,, 2n  2n  1  2n  2n f f f  1.
Vậy f n  n với mọi số nguyên dương n .
Từ f 2x  2 f x với mọi x  
. Ta suy ra 2n   2n f x
f x với mọi x   . m nm  Cho với mọi x
với mọi m, n là số nguyên dương ta suy ra f m  2 f   . 2n  2n  ÁN  m   m m O
Do đó m  2n f  hay f  
mọi số nguyên dương m, n . 2n  2n      2n IC T m P Với mỗi x  
tùy ý cho trước đều tồn tại dãy số q q
k  , k có dạng
hội tụ đến x . 2n YM
f x là hàm liên tục nên x  lim q f q f q f x k lim  k    lim k k k k    OL C
Thử lại ta thấy hàm số f x  x thỏa mãn mọi điều kiện của bài ra. Ụ PH
Câu 66. Tìm tất cả các hàm f : 
thỏa mãn đồng thời hai điều kiện sau: NH
 Với mọi cặp a, b nguyên dương không nguyên tố cùng nhau, có I
f a. f b  f ab CH
 Với mọi bộ a, b nguyên dương tồn tại một tam giác không suy biến có độ dài ba
cạnh là f a , f b và f a b  1 . Lời giải
f a  f b  f a b  1 ; 
Từ điều kiện 2 , với mọi bộ a, b nguyên dương, ta có  f a  f a b  1  f b ;  f
 a b  1  f b  f a ; 2
Nếu a b  2 : f 4  f 2 ; 2 f 2  f 3.
Nếu a  3; b  2 : f 2  f 3  f 4 2
Ta có f 2  f 4  f 2  f 3  f 2  2 f 2  3 f 2  f 2  1 or f 2  2.
Nếu f 2  1 . Do 2 f 2  f 1  f 1  1.
Chinh phục olympic toán| 71
Bồi dưỡng học sinh giỏi
Quy nạp chứng minh f n  1 với mọi n nguyên dương.
Cho a n, b  2 : f n  1  f n  f 2  2  f n  1  1 .
Nếu f 2  2 , bằng quy nạp chứng minh được       1 2 2 . 2
 ....  2k k k  2k f f f f .
Do f 4  f 2  f 3  2 f 2  f 3  3
Quy nạp chứng minh f n  n, n   2
Cho a n  1, b  2 : f n  f n  1  f 2  n  1  f n  . n
Lấy r là số nguyên lớn nhất sao cho 2r không vượt quá n .
Nếu 2r n thì theo chứng minh trên có f n  n . Nếu  2r n
s với 1   2r s .
Với   2r  ;  2r a n s bs  1. Ta có
2r  1  2r  1   2r  1  2r  2r f s s f n f s f ss  1  1 C
    r1  r    r1 2 2
1  2  2r   1  2r f n f f s s
s  1  n  1  f n  n Ọ H
Vậy f n  n, n   2 . ÁN
Do f 1  2 f 2  4 nên f 1 bằng 1, 2 hoặc 3 .
Vậy f n  1 với mọi n nguyên dương hoặc f n  n, n
  2 ; f 11;2;  3 . U TOỆ LI
Câu 67. Tìm các hàm số f : 1;   thoả mãn điều kiện: TƯ
f x  f y  y xf xy với mọi x, y  1 1 Lời giải VÀ Í
Với mọi t > 1, thay x; y  t; 2 ,t; 4 và 2t; 2 vào 1 ta được: CH
f t  f 2  2 tf 2t P  Ạ
f t  f 4  4  tf 4t
f 4  t  3 f 2  t 2t  5 f 4t , t   1 2 T  f
 2t  f 2  2  2tf 4t 5 1 Lấy t
f 4  f 2 2 2  5 
Thay vào 2 ta được t
f 2  t 2t    5 f 4t  2  5 f 2
Do đó với mọi t  1, t   f 4t    2 2t 2 f 2 5
Từ 1 ta có f t  f 4  4  tf 4t   
với t  1, t  . t 2 5 5 Với t
, từ 1 thay x  , y  2 ta có: 2 2
72 | Tạp chí và tư liệu toán học
Phương trình hàm trên tập rời rạc  5  2 f 2     1 f f f
f 2  f 5 4 2 2 2     f t    , t   1  2  2 5 5 t 2 c
Đặt c  2 f 2  f x  với x  1 . x
Thử lại thỏa mãn điều kiện 1 .
Vậy hàm số cần tìm là   c f x  . x
Câu 68. Tìm tất cả các hàm * * f :  thỏa mãn đẳng thức: f  2 f m 2  f n 2 2 2
m  2n , với mọi * m,n . Lời giải m ,m f m f m Nếu * 1 2 sao cho  1   2   f  2
f m  2 f n f f m  2 f n m  2n m  2n 1  2    2  2 2   2 2 2 2 ÁN 1 2 , O Suy ra m m 1
2 hay f là đơn ánh. 2 2 2 2 IC T
Từ đó f m  f n  f p  f q 2 2 2 2 2 2
m  2n p  2q 1 P 2 2 2 2 Dế thấy với mọi * n
,n  3 ta có n  2  2n  1  n  2  2 n  1 2 YM
Chú ý. Điều này vẫn đúng nếu ta nhân cả 2 vế với cùng một thừa số OL 2    C
Đặt f 1 a
f 3a  3 . Theo 1 suy ra: Ụ
f a 2  f a 2  f a 2  f a 2  f a 2 2 2 2 2 2 5 2 3 2 3 3 3  27 PH Vì phương trình 2 2
x  2y  27 chỉ có nghiệm nguyên dương là x; y  3,3 hoặc 5,1
NH I nên ta có f  2a  f  2 1, 5a   5 . CH 2 2 2 2 2 2 2 2
Cũng từ 1 ta có 2 f 4a   2 f 2a   f 5a   f a   24 . Vì phương trình 2 2
x y  12 chỉ có nghiệm nguyên dương là x, y là 4,2 nên f  2
a   f  2 4 4, 2a   2 2 2 2 2
Từ 1 ta có f k   2
a   f k   2
a   f k   2
a   f  2 4 2 3 2 1
ka  , suy ra từ khai triển 2
Vì vậy theo các kết quả trên và phép quy nạp ta suy ra  2
f ka   k , với mọi k là số nguyên
dương. Do đó f  3
a   a f 1 mà f đơn ánh nên 3
a  1  a  1 .
Vậy f n  n với mọi n nguyên dương. Thử lại thỏa mãn bài toán.
Chinh phục olympic toán| 73
Bồi dưỡng học sinh giỏi
Câu 69. Tìm tất cả các số nguyên không âm n sao cho tồn tại một hàm f :  0;
khác hằng thỏa mãn đồng thời 2 điều kiện sau
i) f xy  f xf y , x  , y ii) f  2 2
2 x y   f x  f yx,y   0;1;2;...;  n . Lời giải
Với a  bất kì, bằng cách thay k *
x y a ; k  vào i) được
2  k  2  k f a f
f a  1 0;1;2;...;  n   1  k
Nếu f 2  0 thì 2
f a 0;1;2;...; 
n f a  0
 Nếu f 2  0 thì ta thấy f a  0 hoặc f a  1. k k
Thật vậy, nếu f a  1 thì bằng cách cho k   , ta thấy 2 f a  f 2 f a  1     .
Nên 1 không thể xảy ra, còn nếu 0  f a  1 thì với k đủ lớn ta có C Ọ
2  k  2  k f a f f a  1  0 H  
Nên 1 cũng không thể xảy ra. ÁN
Thành thử, ta đã chứng minh được với mọi a thì f a  0 hoặc f a  1. U TO Từ đó suy ra  f   2 2
2 x y   f x  f y 0;1;  2 ; x  , y  2 Ệ    LI Do đó, n  2. 2 2 TƯ
Nếu n  0, thì 2 f x y   f x  f y ; x  , y  .
f khác hằng nên tồn tại x  sao cho f x  0 0  . VÀ 0 Í
Khi đó f x f x
f 1  f 1  1 0   0      . CH P
Do f khác hằng nên tồn tại x f x  1 1 sao cho  1  . ẠT
Từ i), ta có f 0  f x f 0  f 0  0 1     
Bây giờ, sử dụng 2 ta được  f  2 2
2 2 1  0   f 1  f 0  1 .
Điều vô lí này chứng tỏ n  0 không thỏa mãn. 0 if x
 Nếu n  1 thì hàm số f x 0   . 1 if x  0
Thỏa mãn đề bài. Do đó n  1 thỏa mãn đề bài.
 Nếu n  2 thì ta thấy không thể tồn tại 2 số p,q  ; p,q  1 sao cho f  2 2
p q   0 .
Thật vậy, nếu trái lại, thì x  , y  ta có
        
       2   2 2 2 2 2 2 2 2 2 0 f p q f x y f p q x y f xp yq xq yp
74 | Tạp chí và tư liệu toán học
Phương trình hàm trên tập rời rạc
Kết hợp với 2 suy ra f xp yq  f xp yq  0. Thế nhưng, do  p,q  1 nên tồn tại
x, y  để xp yq  1. Do đó 1  f xp yq  0.
Điều vô lí này chứng tỏ f  2 2
x y   1; x
 , y  ; x, y  1.  p x 0 if   q x
Bây giờ, ta xét hàm số f x  
, trong đó p, q là 2 số nguyên tố phân biệt có  p x 1 if    q x
dạng 4k  3. Ta sẽ chứng minh hàm f x xây dựng như trên thỏa mãn 2 điều kiện:
i) f xy  f xf y , x  , y ii) f  2 2
2 x y   f x  f yx,y   0;1;2;...;  n .
 Kiểm tra điều kiện i) ÁN p xy O Nếu 
thì hiển nhiên f xy  0  f xf y . q xy  IC TP pq x Nếu 
thì f xy  1  f xf y pq y YM
 Kiểm tra điều kiện ii) OL C
f x 0;  1 nên  f  2 2
2 x y   f x  f yx,y   0;1;  2 Ụ 2 f  2
1  p   f 1  f p  1 PH 
Dễ thấy 2 f  2 2
p q   f p  f q  2 nên  f  2 2
2 x y   f x  f yx,y   0;1;  2 NH I 2 f
0  f 0  f 0  0  CH
Vậ y n  1, n  2 là tất cả các giá trị thỏa mãn đề bài.
Câu 70. Tìm tất cả các hàm số f : *  * thoả mãn điều kiện:
f m n  3 2 2 2
f mf n 2
f nf m * 2 . . , m  ,nLời giải
Giả sử tồn tại hàm số f thỏa mãn các yêu cầu của đề bài.
Nếu f n  c , với c là hằng số thì hiển nhiên thỏa mãn điều kiện của bài toán.
Nếu tồn tại m n * ,
sao cho f m  f n thì ta gọi a,b là 2 số thỏa mãn
f a  f b 
f m  f n * min , m,n 1
Giả sử f a  f b . Ta có 3 f b 2
f af b 2
f bf a 3 2 . .
 2 f a . Vậy     2 2
       2 2 f b f a b f a
f a b   f b  f a  f b .
Chinh phục olympic toán| 75
Bồi dưỡng học sinh giỏi Từ đó
           2 2        2 2 f a f b f a f b f a b f b
f a b   f b 2
Rõ ràng 2 mâu thuẫn với 1 .
Do đó f n  c , với c là hằng số là tất cả các hàm cần tìm.
Câu 71. Tìm tất cả các hàm số f :
 thoả mãn điều kiện:
f 0  c 3 f n  1
f n  1      1
f n , n * 3 Lời giải f n 1   
f n  tan f 0  tan 3 6
Từ 1 ta có f n  1    f   6 1  1   1 
f n 1  f ntan 1  f 0tan C 3 6 6 Ọ    H
Do đó ta đặt f 0  c  tan  thì f 1  tan    6    ÁN      f 1  tan tan      tan      f 2 6 6 6 2    tan     U TO       6  Ệ 1  f 1tan 1  tan     tan 6 LI  6  6    TƯ
Ta chứng minh quy nạp công thức f n  tan    ,n  2  6  VÀ
Thật vậy, với n  0,1, 2 công thức 2 đúng. Í  n  CH
Giả sử f n  tan     6  P  Ạ   n   T
f n  tan tan      tan 6  6  6   
Ta có f n  1  
 tan   n   1       f n  6 1 tan 1 tan   tan      6  6  6
Hay 2 đúng với n  1.   
Nghiệm của bài toán là f n  tan    ,n  .  6 
Câu 72. Tìm tất cả các hàm số f :  thỏa:
f 2a  2 f b  f f a b a  ,b
International Mathematical Olympiad 2019
Lời giải – Trần Bá Đạt
Lần lượt thay a  0, b  0 ta được:
76 | Tạp chí và tư liệu toán học
Phương trình hàm trên tập rời rạc
f 0  2 f b  f f b , f 2a  2 f 0  f f a , với a,b
Do đó f 2a  2 f a  f 0 , với mọi a
Thay vào phương trình đề bài ta được 2 f a  2 f b  f 0  f f a b
Cho a  0, b a b trong phương trình trên ta được: f 0  2 f a b  f f a b
Do đó: f a  f b  f a b  f 0
Đặt g x  f x  f 0 suy ra g a  g b  g a b  g x  cx hay f x  cx d x   .
Thay vào phương trình ban đầu và cân bằng hệ số cho a b ta thu được: c d  0 hoặc c  2 .
Vậy f  0 hoặc f x  2x d .
Câu 73. Có tồn tại hay không hàm số f :  sao cho
f m f n  f m  n, m  ,n 1 ÁN O Lời giải
Giả sử tồn tại hàm f thoả mãn đề bài.
IC TP Từ 1cho m0 ta có f f n f 0n 2.  f f nf f n 1    2  YM Với n , n f n f n 1 2 mà  1   2  thì   2
f 0  n f 0  n , n n OL Từ   suy ra   1   2 do đó 1
2 nên f là đơn ánh. C n  1
f m f 0   f m  m f 0  m Ụ Cho 0 từ   ta có   
f f n      PH
Từ đó ta được f 0 0 thay vào 2 có   n, n 3 .
Từ 1 thay m bằng f m và áp dụng 3 ta được f f m  f n  m   n.
NH I Xét m,n,p,q là các số nguyên sao cho mn pq, khi đó CH
f f m  f n  m n  p q f f p  f q
Theo chứng minh trên f là đơn ánh, nên suy ra f m  f n  f q  f p
Do đó với mọi n  ta có
f n  1  f n  1  f n  f n  f n  1  f n  f n  f n  1
f n  1  f n  f n  f n  1  ...  f 2  1  f 1  f 0
Nên  f n là một cấp số cộng với số hạng đầu là U f 0  0 d f 1 1   và công sai   suy
ra f n  U       U nd nd n n , 0. 1 1
Ta xét với hai số n  0, m  0 sao cho m nd  0 thay vào 1 được
f m f n  f m nd  f m  n f m nd  md n  m ndd md n Từ đó có 2
d  1 , điều này vô lý do vậy không tồn tại hàm f thoả mãn yêu cầu của đề bài.
Chinh phục olympic toán| 77
Bồi dưỡng học sinh giỏi
Câu 74. Cho hàm số f :
 là hàm số thỏa mãn các điều kiện sau:
i) f mn  f mf n , m  ,n
ii)m n là ước của f m  f n với mọi m, n
Chứng minh rằng tồn tại một số tự nhiên lẻ k sao cho   k
f n n , n   .
India National Olympiad 2018 Lời giải
Gọi P x, y là phép thế m x, n y vào điều kiện i)
Q x, y là phép thế m x, n y vào điều kiện ii)
Thế P 1, 1 thì ta được f 1.1  f 1 f 1  f 1  1 vì f
Thế Q 2, 2 thì ta được:
2  2  2 2  2 2  2  2k f f f f
q, k  ,2,q  1 C
Giả sử ta xét với q  1 thì tồn tại một số nguyên tố p sao cho p q suy ra p là một số Ọ nguyên tố lẻ. H Từ 2  2k f
q, k  ,2,q  1 nên ta suy ra: p f 2. ÁN  p  1    p  1   p  1  Thế P  2, thì ta được f 2.   
f p  1    f 2 f 2        2   2  U TOỆ
Từ p f 2 nên ta suy ra: p f p  1. LI
Thế Q 1, p  1 thì ta được: TƯ
1  p  1 f 1  f p  1  p f 1  f p  1  p 1  f p  1  p 1 VÀ Í
Điều này là hoàn toàn vô lý, do đó ta phải có  1  2  2k q f . CH
Thế Q 2, 1 thì ta được: P k
2  1 2  1  3 2k  1  2k f f
 1  0mod3   1
   1  0mod3  k phải là số lẻ. T
Từ điều kiện i) thì ta được:
2.2...2  2 2... 2  2k.2k...2k f f f f
, trong đó m lần số 2
 2   2 m m k  2k . m f
Từ đây, ta thế  , 2m Q n  thì ta được:
 2m    2m    2m    2km n f n f n f n 1
Mà ta biết rằng:     k k
x y x y  khi k là số lẻ. Từ đó ta suy ra:  2  2 k m k m n n 2
Từ 1 và 2 thì ta được:
78 | Tạp chí và tư liệu toán học
Phương trình hàm trên tập rời rạc
 2m     2km   k  2km ,    2m   k n f n n m n
f n n , m   3
Khi đó, với m là một số tự nhiên đủ lớn thì 3 xảy ra khi:   k
f n n , n
  , k là số tự nhiên lẻ.
Vậy từ đây ta suy ra được điều phải chứng minh.
Câu 75. Tìm tất cả các hàm số * * f : 
thỏa mãn đồng thời các điều kiện sau:
i) f 0  0, f 1  1
ii) f 0  f 1  f 2  ... iii) f  2 2 x y  2  f x 2  f y * , x  , y Baltic MO Lời giải Ta có: ÁN O
f    f  2 2   2  f   2  f   
f    f  2 2   2  f   2 2 1 1 1 1 2, 5 1 2
1  f 2  5,..., f x x n n      IC T Ở đây 2 x 1, x x n n n 1, 0 1 P
Hiển nhiên thì ta có: lim x   n x YM
Từ đây suy ra nếu f m  f m  1 thì: OL 2 2 2 2 2 2 2 C
f m  1  1   f m  1  f 1  1 f m 1  1 f m  f m  1 Ụ  f  2
m k  f  2 m  1, k   1,2m  2 PH
Quy nạp lên thì ta có tồn tại vô hạn số m sao cho f  2
m k  f  2 m  1, k   1,2m  2
NH I Ta chọn m đủ lớn sao cho tồn tại n để 2 2 a , a        m 1,m 2m 2 n n 1   CH
Khi đó thì a a n
n1 điều này hoàn toàn vô lý nên suy ra f là hàm số tăng thực sự.
Từ đó thì hiển nhiên ta có: f n *  n, n  
, thử lại thì thấy thỏa mãn.
Vậy tất cả các hàm số thỏa mãn yêu cầu bài toán là f n *  n, n   .
Câu 76. Tìm tất cả các hàm số f :
 thỏa mãm các điều kiện sau:
i) Nếu a b thì f a  f b
ii) f ab  f  2 2
a b   f a  f b , a  ,b
Mathlinks Contest Lời giải
Trước hết, ta có một nhận xét nhỏ sau đây.
Nếu f x là một nghiệm hàm thì f x  c cũng là một nghiệm hàm thỏa mãn yêu cầu bài toán.
Chinh phục olympic toán| 79
Bồi dưỡng học sinh giỏi
Do đó ta có thể giả sử rằng f 1  0. Chú ý rằng từ 1 n thì f n  0, n   .
Ta sẽ giải bài toán này thông qua các bước sau đây.
Bước 1. Từ f 1.1  f 1  1  f 1  f 1 suy ra f 2  f 1 hoặc f 2  0.
Bước 2. Gọi n là số nguyên sao cho 1 là số chính phương modulo . n Do đó tồn tại số a thỏa mãn: 2
a  1  k . n Suy ra:
f a  f  2
a    f a  f    f  2 1 1
a  1  f kn  f 1  0
Nhưng f n  f kn  f  2
a  1 và f n  f 1 nên f n  f 1  0.
Do đó nếu tồn tại u sao cho 2 u  1
 modn thì f n  0.
Bước 3. Từ bước 2 dễ thấy f p  0 với mọi p là số nguyên tố và p  1mod 4.
Bước 4. Giả sử f a  f b  0 và f ab  f a  f b  0 thì f  2 2
a b   0, vô lý.    C
Do đó nếu f a
f b 0 thì f ab 0. Ọ
Bước 5. Gọi a,b là hai số nguyên thỏa mãn a,b  1, khi đó gọi p là một ước của H 2 2
a b thì ta có: 2 2
a b  0mod p. ÁN
Mà ta có một bổ đề quen thuộc sau. Nếu p là một số nguyên tố có dạng 4k  3 thì với mọi U TO
bộ số a, b thỏa mãn 2 2
p a b thì ta sẽ có p a p b. Ệ LI
Vì a,b  1 nên nếu 2 2
p a b thì p chỉ có dạng 4k  1. TƯ
Từ bước 4 thì ta có: 2 2
a b là tích của các số nguyên tố p f p  2 2 f a b  0. i thỏa mãn  i  0 nên   VÀ Í
Mà từ f ab  f  2 2
a b   f a  f b , a
 ,b  ta có a,b  1  f ab  f a  f b. CH
Bước 6. Cho a bc vào phương trình đã cho thì ta được: P Ạ 2 2 2     T
f b cf b c 1
f bcf b
Nhưng do f b  f  2 b  2
c  1 và     2 f bc f b c
Do đó thì ta có: f bc  f  2 b c *
Thế c  1 vào * thì được: f b  f  2 b .
Thế c b vào * thì được: f  2
b   f  3 b .
Từ đấy, bằng phép quy nạp thì ta được  k
f b   f b , k   1
Bước 7. Sử dụng bước 5bước 6 thì ta có: f i np   f p p i
i  ở đây i là các số nguyên tố
Xét hàm số f x xác định bởi:
* f 1  0, f 2  0
80 | Tạp chí và tư liệu toán học
Phương trình hàm trên tập rời rạc
* f p  0 với các số nguyên tố p sao cho p  1mod 4 và p  2.
* f p  a  0 a p
với mọi số nguyên tố p còn lại, ở đây p là các số nguyên không dương. * f i np   f p p i
i  ở đây i là các số nguyên tố.
Ta có thể chứng minh f x thỏa mãn các điều kiện:
Hiển nhiên nếu a b thì f a  f b.
f 1.1  f 1  1  f 1  f 1  0
f a   f  2 .1
a  1  f a  f 1
Ta có mọi ước nguyên tố p của 2
a  1 đều thỏa mãn p  1mod 4.
Với hai số nguyên a, b  1 bất kì, ta gọi: p b
i là các ước nguyên tố của a không chia hết cho .
qi là các ước nguyên tố của b không chia hết cho . a ÁN r b
i là các ước nguyên tố của a và . O
f a   f p   f r i   i  IC T
f b   f q   f r i   i  P
f ab   f p   f q   f r i   i   i  YM 2 2     OL a b f  2 2
a b    f r   f s s A       . i
i ,ở đây là các ước nguyên tố của C i
 a,b   a,b      Ụ
Nhưng tương tự bước 5 ta có: PH
Các ước nguyên tố của A là các số nguyên tố thỏa mãn s f A i
1mod 4 và do đó   0. NH I Suy ra f  2 2
a b    f ri  CH 2 2
Hay ta có f ab  f a b   f a  f b , a  ,b
Và ta có nghiệm của phương trình hàm là:
Cho M là một số nguyên, hàm f được xác định như sau:
* f 1  M
* f 2  M
* f p  M với mọi số nguyên tố p thỏa mãn p  1mod 4.
* f p  M a a
p với mọi số nguyên tố p còn lại, ở đây p là các số nguyên không dương. * f i n
p M  f p M p i    i
 ở đây i là các số nguyên tố.
Chinh phục olympic toán| 81
Bồi dưỡng học sinh giỏi
Câu 77. Tồn tại hay không hàm số f : 1, 2,..., 
n  thỏa mãn điều kiện:
i) f là hàm đơn ánh
ii) f ab  f a  f b với mọi a,b 1, 2,..., 
n ab n Lời giải
Ta có thể chỉ ra hàm số f như sau:
Kí hiệu các số nguyên tố bé hơn hoặc bằng n theo thứ tự tăng dần là: p , p ,..., pk. 1 2 k k i Khi đó nếu i
a  p     n
f p n  1 , f a   n  1 i , i , i 1 thì  i      i   i1 i1
Ta sẽ chứng minh hàm số này thỏa mãn yêu cầu bài toán. k k Thật vậy, với i a  pi b  p   i
i có ít nhất một giá trị i j khác nhau. i1 i1 k k
Thì f a   n
f b   n i 1 i  1 và     i1 i1 C Ọ
Hiển nhiên ta chỉ có thể biểu thị f a , f b một cách duy nhất sang hệ cơ số n  1 và vì H
thế f a  f b. ÁN k k k k  
Mặt khác ta có f ab i i
f p      f p   f p  f p f a    f b i
i i   i ii ii       i1  i1 i1 i1 U TOỆ
Hay ta có: f ab  f a  f b , đúng theo giả thiết đề bài. LI
Vậy từ đó hàm số xây dựng như trên thỏa mãn yêu cầu bài toán. TƯ
Câu 78. Giả sử Josephus có n  1 người bạn, n người này đúng thành một vòng tròn VÀ Í
đánh số từ 1 đến n theo chiều kim đồng hồ, tự sát theo nguyên tắc, người thứ nhất cầm CH
dao đếm 1 rồi tự sát, người thứ hai đếm 2 rồi tự sát,…Quá trình dừng lại khi còn một P Ạ
người. Gọi f n là hàm số biểu thị vị trí cùa người sống sót đó. Câu hỏi đặt ra là, hãy T
tính f n ?
Bài toán cổ Josephus Lời giải
Ta sẽ xét hai khả năng của n là chẵn và lẻ.
Khả năng 1. Khi số n là số chẵn, tức n  2k. Sau vòng 1 thì còn người ở vị trí lẻ. Số người
này đánh lại thành 1, 2,..., k.
Nếu lượt trước người đó có số 2i  1 thì sau đó mang số i. Người sống sót có số cũ là
f 2k sau mang số mới là f k. Vậy từ đây ta có f 2k  2 f k  1
Khả năng 2. Khi số n là số lẻ, tức là n  2k  1. Sau vòng 1 ta ngầm hiểu rằng có 2k  2
người bằng cách tính trùng người thứ 1 thành 2k  2, còn lại những người số 3, 5,..., 2k  1
đánh số lại là 1, 2,..., k.
82 | Tạp chí và tư liệu toán học
Phương trình hàm trên tập rời rạc
Nếu lượt trước người đó có số 2i  1 thì sau đó mang số i. Người sống sót có số cũ là
f 2k  1 sau mang số mới là f k. Vậy từ đây ta có f 2k  1  2 f k  1
Như vậy thì f 1  1, f 2k  2 f k  1, f 2k  1  2 f k  1
Ta chứng minh bằng quy nạp rằng.
Nếu biểu diễn trong cơ số 2 của n là: n  a a    a a a i 0,1 k k ... , k 1, 1 1  với i k và   2
Thì f n  a a a k ... k . 1 1  Thật vậy: 2
Với n  1 thì ta thấy hiển nhiên đúng.
Giả sử với mọi k n thì mệnh đề đúng. Ta sẽ xét hai trường hợp sau đây:
Trường hợp 1. Nếu n là số chẵn, đặt n  2 . m
Khi đó nếu như: m  b b   b 2m b b b k k ... 0 k k ... 1 1  thì  1 1  2 2
f 2m  2 f m  1  2 b       2k b b b k ... .2 1 1 k ... 01 1 1   1 1  2
Vậy trong trường hợp 1 thì mệnh đề đúng. ÁN O
Trường hợp 2. Nếu n là số lẻ, đặt n  2m  1.
Khi đó nếu như: m  b b    b 2m 1 b b b k k ... 1 k k ... 1 1  thì  1 1  IC T 2 2 P
f 2m  1  2 f m  1  2 b       2k b b b k ... .2 1 1 k ... 11 1 1   1 1  2 YM
Vậy trong trường hợp 2 thì mệnh đề đúng. OL
Từ đó, theo nguyên lý quy nạp thì mệnh đề ban đầu đúng và từ đó ta suy ra điều phải C Ụ chứng minh. PH
Câu 79. Cho hai hàm số * * f , g : 
là hai hàm số thỏa mãn đồng thời các điều kiện:
NH I i) g là hàm số toàn ánh 2 2 2 CH
ii) 2 f nn
g n , n     
Nếu f nn 2019 n, n     
thì f có vô số điểm bất động. Lời giải
Đầu tiên ta có định lý Dirichlet về số nguyên tố thì dãy số  p p i  với
i là các số nguyên tô
có dạng 8k  3 là một dãy vô hạn.
Từ đó với mọi n, theo công thức của kí hiệu Legendre ta có: 2 2    n p     1   1 8  1   pn
Sử dụng điều kiện i) thì ta tìm được dãy x g x p n     n  sao cho  n n , . n1 Ta có 2
f x x p f x x p n  2 2 2 n nn  2 2 2 n mod n   2  p f n n   Vì    1  nên suy ra:   pn  p xn n
Chinh phục olympic toán| 83
Bồi dưỡng học sinh giỏi x a p n n n
Suy ra tồn tại hai dãy số nguyên dương a b
n  và  n  sao cho   f  x b p n n n
Từ điều kiện ii) thì ta được 2 2 2b a  1 n n
Cuối cùng, sử dụng giả thiết: f nn 2019 n, n      thì ta có: 2019 f x 2 a n 1 n b   1 n   1  lim  2  lim a n 1 x x a x x a  n n n n Suy ra tồn tại N a b n   N n n 1, 0 sao cho 0
Vậy từ đó f p p n   N n n , . 0
Và từ đây ta suy ra điều phải chứng minh.
Câu 80. Tìm tất cả các hàm số * * g : 
thỏa mãn điều kiện sau:
g gn  n  g n  1  3  n gn , n   C Ọ
Doãn Quang Tiến H Lời giải ÁN
Đặt g n  f n *  n, n  
thì thay vào phương trình hàm ban đầu ta được:
f f n  f n  f n  1  n  1  3  n f n *  n, n   U TO * Ệ
f f n  f n  1  n  2, n   1 LI
Từ đây, ta đã chuyển bài toán ban đầu thành một bài toán khác có vẻ gọn đẹp hơn rất TƯ nhiều.
Thay n  1 vào 1 thì ta được f f 1  f 2  3 VÀ Í
Từ đó ta suy ra: f 2  2 và f f 1  2. Từ đó ta xét hai trường hợp sau: CH
Trường hợp 1. f 2  1 và f f 1  2. P ẠT
Bây giờ ta đặt f 1  k f k  2
Thay n  2 vào 1 thì ta được f f 2  f 3  4
Từ đây suy ra f 3  4  f 1  4  k
Từ f 3  1 nên suy ra: k  3.
Nếu k  1 thì ta có: 2  f f 1  f k  f 1  1, điều này cũng mâu thuẫn.
Nếu k  2 thì ta cũng có: 2  f f 1  f k  f 2  1, điều này cũng mâu thuẫn.
Nếu k  3 thì ta có: 2  f f 1  f k  f 3  4  k  4  3  1, điều này cũng mâu thuẫn.
Vậy tóm lại không có giá trị nào của k thỏa mãn nên trường hợp 1 không xảy ra.
Trường hợp 2. f 2  2 và f f 1  1.
Thay n  2 vào 1 thì ta được f f 2  f 3  4
Từ đó thì ta dễ thấy f 3  2 và ta tính toán được các giá trị sau:
84 | Tạp chí và tư liệu toán học
Phương trình hàm trên tập rời rạc
f 4  5  f f 3  5  f 2  3
f 5  6  f f 4  6  f 3  4
f 6  7  f f 5  7  f 4  4
Từ đấy ta dự đoán được rằng, hàm số f n được xác định như sau: 1  5 f n *    n   n  1, n   trong đó   2
Bây giờ ta sẽ chứng minh rằng, hàm số này là hàm số thỏa mãn 1 để rồi từ đó suy ra
công thức của hàm g n và từ đó ta hoàn tất bài toán.
Mà trước tiên, để chứng minh nhận định đó, ta cần phải có hai bổ đề sau: n
Bổ đề 1. Với mỗi số * n  thì ta có     n   n  1    n   1 Chứng minh ÁN Trước hết ta có     n 
n  1  n n  1    n  1  n    n  2 O  Và     n 
n  1  n  1  n  1  1  n  1  1  n  1  IC TP 1  5 1   5 Do lưu ý rằng:      1 
nên suy ra:    1  1. YM 2 2
Vậy từ đó ta thấy bổ đề 1 được chứng minh. OL C   n   2 if     n 
n  1  n Ụ 
Bổ đề 2. Với mỗi số * n  thì ta có 
 n  1      n   1 otherwise PH Chứng minh
NH I Hiển nhiên thì ta có n1   bằng   n   1 hoặc   n   2 CH           Giả n 1 n 1 sử   thì từ đó ta có:     n 
n  1  
  n  1  n
  n11n1 n111 n
Và như trên thì ta có     n 
n  1  n n  1    n  1  n    n  2 
Từ đó thì ta suy ra được:     n 
n  1  n  1.  Giả sử 
 n  1     n   2 thì từ đó ta có:     n 
n  1  
  n  1  n  1 
  n1n1  n1 
Tử đó thì theo bổ đề 1 thì ta thu được:     n   n  1  . n
Vậy từ đó thì bổ đề 2 được chứng minh.
Quay trở lại với việc giải bài toán
Ta sẽ sử dụng phương pháp quy nạp để chứng minh kết quả ban đầu.
Với n  1 thì f 1   .1     1  1  1.
Chinh phục olympic toán| 85
Bồi dưỡng học sinh giỏi
Với n  2 thì f 2   .2     2  1  2.
Giả sử kết quả đúng với 1  j  .
n Sử dụng 1 thì ta có:
f n  1  n  2  f f n  n  2  f   n 
n  1  n  2    n   n  1     n   n  1  1 Mà từ   n 
n  1  2n n  1  n  1 ta có f n  1    n   2      n   n  1
Giả sử n thỏa mãn:     n 
n  1  n  thì từ đó ta có: 
 n  1     n   2
Và do đó ta suy ra được f n  1  
 n  1  n
Nếu n không thỏa mãn     n 
n  1  n
thì tức là chỉ có thể xảy ra:     n 
n  1  n  1. 
Và theo bổ đề 2 thì ta được: 
 n  1     n   1
Và từ đó ta suy ra được:
f n  1    n   2      n 
n  1     n 
 2  n  1    n   1  n  
 n  1  n C  Ọ
Vậy từ đó theo nguyên lý quy nạp thì mệnh đề 1 được chứng minh hoàn toàn. H
Từ đây suy ra tất cả các hàm số thỏa mãn 1 là ÁN 1  5 f n *    n   n  1, n   trong đó   . 2 U TOỆ
Hay từ đây ta suy ra được hàm g n mà chúng ta cần tìm là: LI 1  5 TƯ
g n  f n *  n    n 
n  1  n    n   1, n   trong đó   . 2 VÀ Í
Câu 81. Cho ba số thực a, b,c không âm, phân biệt sao cho tồn tại hàm f , g :   CH  x  P
thỏa mãn af xy  bf    cf x  g y với mọi số thực dương x y . Ạ  y  T
Chứng minh rằng tồn tại hàm h :   sao cho:    x
f xy f    2 f x  hy, x   y  0  y Iran TST 2019 Lời giải x
Đặt P x, y là phép thế cho phương trình: af xy  bf    cf x  g y  y
Trường hợp 1: Nếu a b c  0 : g 1  x
Ta có P x,1  f x   
là hằng số, vậy: f xy  f    2 f x
a b cy
Suy ra h y  0 y  , thỏa mãn.
86 | Tạp chí và tư liệu toán học
Phương trình hàm trên tập rời rạc
Trường hợp 2: Nếu a b c  0 :   x  
Ta có af xy  f x  bf    f x  g y 
và đặt phép thế là Q x, yy     
Lấy Q 1, y  Qx, y ta được
           x     a f xy f x f y f
bf    f x 1 1
f    f 1  0 1  y y        1 Thế y  vào 1 ta được: y   x    
af    f x 1
f    f 1  bf xy  f x  f y  f 1  0  y y       
Do a b , a b c  0 2 2
a b  0 , vì vậy:
f xy  f x  f y  f 1 , x  , y  0 ÁN  x   1  O
f    f x  f    f 1 , x  , y  0  y   y  IC T     P x 1
Cộng hai phương trình với nhau f xy  f    2 f x  f y  f    2 f 1  y   y  YM  1 
h y f y f    2 f 1 y  OL Vậy       , thỏa mãn.  y C  Ụ PH
Câu 82. Tìm tất cả hàm số f :  thỏa mãn:
n! f m! f n! f m! , m  ,n NH I
BMO Shortlist 2018 CH Lời giải
Đặt phép thế P m, n cho phương trình ban đầu
P 1,1  1  f 1! f 1  f 1!  f 1! 1 f 1  1, từ đây hiển nhiên f 1  1
P1,n  n! 1 f n! 1  f n!  n!  f n  n
Gọi p là số nguyên tố tùy ý, ta có P 1, p  1   p  1! 1 f p  1! 1 .
Theo định lý Wilson trong số học ta được p p  1! 1 , suy ra p f p  1! 1
Lưu ý rằng nếu f p  1  p  1 thì f p  1! là tích của ít nhất p thừa số nguyên dương và
là một số chia hết cho p , do đó f p  1! 1  1mod p - mâu thuẫn chứng minh trên.
Vậy f p  1  p  1 , lại có f n  n nên f p  1  p  1 với mọi số nguyên tố p .
Lại có P m, p  1   p  1! f m! f m!  p  1!  p  1! f m! f m!  f m!
Chinh phục olympic toán| 87
Bồi dưỡng học sinh giỏi
Với giá trị m bất kỳ, ta chọn p đủ lớn để thu được: f m!  f m! , sử dụng kết quả này ta
được: n! f m! f n! f m! tương đương n! f m! f n! n!
Thay m p  1 với p đủ lớn vào phương trình trên ra được f n!  n! với mọi n .
Vậy f n  n là hàm số cần tìm.
Câu 83. Tồn tại hay không hàm số * * f : 
thỏa mãn điều kiện sau:
f f n  n f n * 3 2 , n   Lời giải
Giả sử tồn tại hàm số f thỏa mãn yêu cầu bài toán. Với mỗi * i
ta xây dựng dãy số như sau a
a i, a   f a n  sao cho: 1 n 1  n n1 Khi đó thì ta có a        f a f f a f a a a a n n n 2 n 3 n 2 n 3 1     1  1 1 n1 Hay ta có a       a a n n 4 n 3 n 0, 1 C 4 1 Ọ Do a n   n 0,
1 nên đẳng thức không thể xảy ra. H
Nên từ đó ta kết luận rằng không tồn tại hàm số f thỏa mãn yêu cầu bài toán. ÁN
Câu 84. Tìm tất cả các hàm số tăng thực sự * * f : 
thỏa mãn điều kiện sau: U TOỆ
f n f n  f n * 2 , n   LI Lời giải
Do f là hàm số tăng thực sự nên ta có:
f n  1  f n  1 hay f n  1  n  1  f n  n VÀ Í
Suy ra: f n  n là hàm số tăng. CH
Mặt khác ta đặt a  1, a    a f a 0 n 1 nn  P Ạ
Từ đó ta suy ra a a  ... f a   f a n 2 0 1 và  1   n  T Do đó f a       a f a a n nnn  * n , 1 1
Suy ra có vô hạn bộ số m, n sao cho: f m  m f n  n
Suy ra f n  n c, với c là hằng số.
Vậy tất cả các hàm số thỏa mãn yêu cầu bài toán là: f n *
n c, n   ,c là hằng số.
Câu 85. Tìm tất cả các toàn ánh f :
 sao cho với mọi m,n thỏa mãn:
f mf n  m n Lời giải Kí hiệu P
là tập tất cả các số nguyên tố. k k
Xét đơn ánh g : P P thì nếu i n  pi
f n   g p  . i thì    i i1 i1
88 | Tạp chí và tư liệu toán học
Phương trình hàm trên tập rời rạc
Kí hiệu n là số ước nguyên dương của . n
Mà ta có nhận xét sau: n  f n do f là toàn ánh.
Với mỗi số nguyên tố p thì f p chỉ có đúng hai ước nguyên tố nên nó cũng là số nguyên tố.
Xác định hàm g như trên thì từ đó ta có: f p  g p nên ta sẽ chứng minh g là song ánh.
Thật vậy, do f là toàn ánh nên g là toàn ánh nên g là song ánh.
Tiếp theo, ta sẽ chứng minh     k k f p
g p với k là số nguyên dương bằng quy nạp
Ta thấy rằng, với k  1 thì hiển nhiên đúng.
Giả sử mệnh đề đúng với k  1. 2 k1 Ta có  k
f p  chia hết cho 1, gp, gp ,..., gp và ngoài ra không chia hết cho số nguyên dương nào khác.
Do đó   k    k f p
p   k  1. ÁN k k f p  k
f p   2k k  1, O Nếu
1 khi   có thêm một ước nguyên tố nữa thì   vô lý. k Từ đó  k
f p  là lũy thừa của gp và nó có k  1 ước nên  k
f p   gp .
IC TP Giả sử n là một số nguyên dương, p là một số nguyên tố không chia hết cho .n YM
Bây giờ ta sẽ đi chứng minh    k    k f n f p f np , k   OL Từ  , k
n p   1 nên ta có:   k    k n p np  C Ụ k Mặt khác    k g p
f np  và gpf n PH
Do vậy mọi ước của f n và  k
g p chia hết cho  k
f np  và mọi ước của  k
g p f n là NH k I
ước của f np . 
k    k     k f n f p np f np  CH Lại có     Nếu  k
f np  có ước khác với các ước của f n và  k g p thì
    k     k f n f p f np  , vô lý. k
Vậy từ đó ta có kết quả  k           k f np f n g p f n f p
Từ các nhận xét trên ta có hàm f được xây dựng như trên là duy nhất.
Vậy tất cả các hàm số thỏa mãn yêu cầu bài toán là các hàm như trên.
Chinh phục olympic toán| 89 TÀI LIỆU THAM KHẢO
Dưới đây là các tài liệu mà ebook này có tham khảo và đồng thời có cả những tài liệu mà
bọn mình đề xuất cho bạn đọc
[1]. Nguyễn Văn Mậu (1997), 00 Phương trình hàm 00, NXB Giáo dục.
[2]. Nguyễn Trọng Tuấn (2004), “Bài toán hàm số qua các kì thi Olympic”, NXB Giáo dục.
[3]. Nguyễn Tài Chung, Lê Hoành Phò (2013), “Chuyên khảo phương trình hàm” Nhà
xuất bản Đại học quốc gia Hà Nội.
[4]. J.Aczél (1966), “Lectures on functional equations and their applications”, ACADEMIC
PRESS New York San Francisco London.
[5]. Stevo Stevic (2004), “Periodic Character of a Class of Difference Equation”, Taylor & Francis Group.
[6]. Valentine Boju, Luis Funar - The Math Problems Notebook.
[7]. Titu Andreescu, Razvan Gelca – Birkhauser Mathematical Olympiad Challenges.
[8]. Edward Lozansky , Cecil Rousseau – Winning Solutions.
[9]. The IMO Compendium. A Collection of Problems Suggested for The International
Mathematical Olympiads: 1959 – 2009 – Djukic D., Vladimir Jankovic, Ivan Matic, Nikola Petrovic – Springer (2011).
[10]. Trần Nam Dũng, Dương Bửu Lộc – Chuyên đề Phương trình hàm trên tập số nguyên .
[11]. (Developments in Mathematics 39) Saïd Abbas, Mouffak Benchohra – Advanced.
[12]. Functional Evolution Equations and Inclusions-Springer International Publishing (2015)
[13]. Aczel – Lectures on functional equations and their applications – Academic Press (1966).
[14]. An Introduction to the Theory of Functional Equationsand Inequalities – Marek Kuczma.
[15]. Analytic Solutions of Functional Equations – Sui Sun Cheng, Wenrong Li.
[16]. Functional Analysis, Sobolev Spaces and Partial Differential Equations – Haim Brezis.
[17]. Topics in Algebra and Analysis Preparing for the Mathematical Olympiad –Radmila.
[18]. Bulajich Manfrino, José Antonio Gómez Ortega, Rogelio Valdez Delgado-Birkhäuser Basel (2015).
[19]. Kỷ yếu gặp gỡ toán học và các kỷ yếu từ hội thảo chuyên đề các tỉnh. [20]. The art of Mathematics.
[21]. 101 Problems in Algebra from the training of the USA IMO team – T Andreescu, Z Feng.
[22]. Problem Primer for the Olympiad – C. R. Pranesachar, B. J. Venkatachala, C. S. Yogananda
[23]. https://artofproblemsolving.com
TẠP CHÍ VÀ TƯ LIỆU TOÁN HỌC HẾT CHINH PHỤC OLYMPIC TOÁN
TẠP CHÍ VÀ TƯ LIỆU TOÁN HỌC
Thôn 6 – Thạch Hòa – Thạch Thất – Hà Nội
Điện thoại: 0343763310; Email: tuangenk@gmail.com
Fanpage: https://www.facebook.com/OlympiadMathematical/
CHỊU TRÁCH NHIỆM NỘI DUNG DOÃN QUANG TIẾN NGUYỄN MINH TUẤN TÔN NGỌC MINH QUÂN BIÊN TẬP NGUYỄN MINH TUẤN
TRÌNH BÀY BẢN THẢO NGUYỄN MINH TUẤN LA THỊ ĐÔNG PHƯƠNG
PHƯƠNG TRÌNH HÀM TRÊN TẬP RỜI RẠC
Đề nghị quý bạn đọc tôn trọng bản quyền của tác giả, không sao chép bản phụ.
Mọi ý kiến thắc mắc đóng góp vui lòng gửi về địa chỉ đã cung cấp ở trên.
Phiên bản sách điện tử được phát hành vào ngày 20/8/2019. CHINH PHỤC OLYMPIC TOÁN
MỌI Ý KIẾN THẮC MẮC XIN VUI LÒNG GỬI VỀ ĐỊA CHỈ NGUYỄN MINH TUẤN 0343763310 tuangenk@gmail.com Lovetoan.wordpress.com Đại học FPT Hà Nội PHIÊN BẢN ĐẶC BIỆT